PrepU Neurological System

Pataasin ang iyong marka sa homework at exams ngayon gamit ang Quizwiz!

A client presents to the emergency department status post-seizure. The physician wants to know what the pressure is in the client's head. What test might be ordered on this client? Lumbar puncture Nerve conduction studies EMG Echoencephalography

Correct response: Lumbar puncture Explanation: Changes in CSF occur in many neurologic disorders. A lumbar puncture (spinal tap) is performed to obtain samples of CSF from the subarachnoid space for laboratory examination and to measure CSF pressure.

A nurse is caring for a client with a diagnosis of trigeminal neuralgia. Which activity is altered as a result of this diagnosis? swallowing smelling chewing tasting

Correct response: chewing Explanation: Trigeminal neuralgia is a painful condition that involves the fifth (V) cranial nerve (the trigeminal nerve) and is important to chewing.

A client with a neurologic impairment reports having problems with constipation. Which foods might the nurse recommend? white rice vegetables ice cream meat

Correct response: vegetables Explanation: Vegetables are high in fiber. Fiber increases fecal bulk and pulls water into the feces, promoting regular bowel movements. Ice cream, meat, and white rice are low in fiber.

A nurse is caring for a client who reports throbbing headache after a lumbar puncture. Which of the following actions is most likely to facilitate resolution of the headache? a. Darken the client's room and close the door. b. Administer pain medication. c. Increase fluid intake d. Elevate the head of the bed to 30'

Increase fluid intake

A client with Parkinson's disease asks the nurse what their treatment is supposed to do since the disease is progressive. What would be the nurse's best response? "Treatment for Parkinson's is only palliative; it keeps you comfortable." "Treatment really doesn't matter; the disease is going to progress anyway." "Treatment aims at keeping you independent as long as possible." "Treatment aims at keeping you emotionally healthy by making you think you are doing something to fight this disease."

Correct response: "Treatment aims at keeping you independent as long as possible." Explanation: Treatment aims at prolonging independence. Treatment does matter, it is not palliative, and it is not aimed at keeping you emotionally healthy.

Which of the following is an early sign of increasing intracranial pressure (ICP)? Headache Vomiting Loss of consciousness Decerebrate posturing

Correct response: Headache Explanation: A headache that is constant or increases in intensity is considered an early sign of increasing intracranial pressure (ICP). Loss of consciousness, projectile vomiting, and decerebrate posturing are all later signs of increasing ICP. Chapter 61: p. 2009.

The nurse reviews the physician's emergency department progress notes for the client who sustained a head injury and sees that the physician observed the Battle sign. The nurse knows that the physician observed which clinical manifestation? A bloodstain surrounded by a yellowish stain on the head dressing An area of bruising over the mastoid bone Escape of cerebrospinal fluid from the client's ear Escape of cerebrospinal fluid from the client's nose

Correct response: An area of bruising over the mastoid bone Explanation: Battle sign may indicate a skull fracture. A bloodstain surrounded by a yellowish stain on the head dressing is referred to as a halo sign and is highly suggestive of a cerebrospinal fluid (CSF) leak. Escape of CSF from the client's ear is termed otorrhea. Escape of CSF from the client's nose is termed rhinorrhea. Chapter 63: pp. 2056-2057.

The nurse is caring for a client diagnosed with Guillain-Barre syndrome. His spouse asks about recovery rates. The nurse can correctly relate which of the following? No one with Guillain-Barre syndrome recovers completely. Only a very small percentage (5% to 8%) of clients recover completely. Usually 100% of clients recover completely. Approximately 60% to 75% of clients recover completely.

Correct response: Approximately 60% to 75% of clients recover completely. Explanation: Results of studies on Guillain-Barre syndrome indicate that 60% to 75% of clients recover completely.

Which term refers to the inability to perform previously learned purposeful motor acts on a voluntary basis? Agnosia Agraphia Apraxia Perseveration

Correct response: Apraxia Explanation: Verbal apraxia refers to difficulty forming and organizing intelligible words although the musculature is intact. Agnosia is a failure to recognize familiar objects perceived by the senses. Agraphia refers to disturbances in writing intelligible words. Perseveration is the continued and automatic repetition of an activity or word or phrase that is no longer appropriate.

Level of consciousness (LOC) can be assessed based on criteria in the Glasgow Coma Scale (GCS). Which of the following indicators are assessed in the GCS? Select all that apply. Verbal response Muscle strength Eye opening Intelligence Motor response

Correct response: Eye opening Verbal response Motor response Explanation: LOC can be assessed based on the criteria in the GCS, which include eye opening, verbal response, and motor response. The patient's responses are rated on a scale from 3 to 15. Intelligence and muscle strength are not measured in the GCS. Chapter 63: Page 2062

Which is a nonmodifiable risk factor for ischemic stroke? Smoking Gender Atrial fibrillation Hyperlipidemia

Correct response: Gender Explanation: Nonmodifiable risk factors include gender, age, and race. Modifiable risk factors include atrial fibrillation, hyperlipidemia, and smoking.

Cranial nerve IX is also known as which of the following? Hypoglossal Spinal accessory Vagus Glossopharyngeal

Correct response: Glossopharyngeal Explanation: Cranial nerve IX is the glossopharyngeal nerve. The vagus nerve is cranial nerve X. Cranial nerve XII is the hypoglossal nerve. The spinal accessory is the cranial nerve XI.

The nurse practitioner advises a patient who is at high risk for a stroke to be vigilant in his medication regimen, to maintain a healthy weight, and to adopt a reasonable exercise program. This advice is based on research data that shows the most important risk factor for stroke is: Dyslipidemia Obesity Hypertension Smoking

Correct response: Hypertension Explanation: Hypertension is the most modifiable risk factor for either ischemic or hemorrhagic stroke. Unfortunately, it remains under-recognized and undertreated in most communities.

Following a transsphenoidal hypophysectomy, a nurse should assess a client for which condition? Hypocortisolism Hyperglycemia Hypoglycemia Hypercalcemia

Correct response: Hypocortisolism Explanation: Although steroids should be given during surgery to prevent hypocortisolism, the nurse should assess the client for it. Abrupt withdrawal of endogenous cortisol may lead to severe adrenal insufficiency. Signs of hypocortisolism include vomiting, increased weakness, dehydration, and hypotension. After the corticotropin-secreting tumor is removed, the client shouldn't be at risk for hyperglycemia. Calcium imbalance and hypoglycemia shouldn't occur in this situation. Chapter 61: Page 2010

The nurse is planning care of a client admitted to the neurologic rehabilitation unit following a cerebrovascular accident. Which nursing intervention would be of highest priority? Include client in planning of care and setting of goals. Praise client when using adaptive equipment. Assess client for ability to ambulate independently. Provide instruction on blood-thinning medication.

Correct response: Include client in planning of care and setting of goals. Explanation: The client in a rehabilitation setting has moved to the recovery phase. The highest priority is to include the client in the rehabilitation plan. Tailoring the rehabilitation plan to meet the needs of the client can promote optimal participation by the client in the rehabilitative process. The other options are appropriate in certain situations but not the highest priority.

Bone density testing will be completed for the client with post-polio syndrome. The nurse teaches the client bone density testing is used to identify what potential complication? Low bone mass and osteoporosis Pathologic fractures Osteoarthritis Calcification of long bones

Correct response: Low bone mass and osteoporosis Explanation: Bone density testing in clients with post-polio syndrome has demonstrated low bone mass and osteoporosis. Thus, the importance of identifying risks, preventing falls, and treating osteoporosis must be discussed with clients and their families.

A client with a cerebellar brain tumor is admitted to an acute care facility. The nurse formulates a nursing diagnosis of Risk for injury. Which "related-to" phrase should the nurse add to complete the nursing diagnosis statement? Related to impaired balance Related to visual field deficits Related to psychomotor seizures Related to difficulty swallowing

Correct response: Related to impaired balance Explanation: A client with a cerebellar brain tumor may suffer injury from impaired balance as well as disturbed gait and incoordination. Visual field deficits, difficulty swallowing, and psychomotor seizures may result from dysfunction of the pituitary gland, pons, occipital lobe, parietal lobe, or temporal lobe — not from a cerebellar brain tumor. Difficulty swallowing suggests medullary dysfunction. Psychomotor seizures suggest temporal lobe dysfunction.

The nurse in collaboration with the rehabilitation team is working with a patient on performing therapeutic exercises. Which of the following would the nurse expect to encourage to increase the patient's muscle power? Passive exercises Active exercises Isometric exercises Resistive exercises

Correct response: Resistive exercises Explanation: Resistive exercises provide resistance to increase muscle power. Passive exercises are used to retain as much joint range of motion as possible and to maintain circulation. Isometric exercises are used to maintain strength when a joint is immobilized. Active exercises are used to increase muscle strength.

The nurse is caring for a client immediately following a spinal cord injury (SCI). Which is an acute complication of SCI? Spinal shock Paraplegia Cardiogenic shock Tetraplegia

Correct response: Spinal shock Explanation: Acute complications of SCI include spinal and neurogenic shock and deep vein thrombosis (DVT). The spinal shock associated with SCI reflects a sudden depression of reflex activity in the spinal cord (areflexia) below the level of injury. Cardiogenic shock is not associated with SCI. Tetraplegia is paralysis of all extremities after a high cervical spine injury. Paraplegia occurs with injuries at the thoracic level. Autonomic dysreflexia is a long-term complication of SCI

Which condition occurs when blood collects between the dura mater and arachnoid membrane? Extradural hematoma Intracerebral hemorrhage Epidural hematoma Subdural hematoma

Correct response: Subdural hematoma Explanation: A subdural hematoma is a collection of blood between the dura mater and brain, space normally occupied by a thin cushion of fluid. Intracerebral hemorrhage is bleeding in the brain or the cerebral tissue with the displacement of surrounding structures. An epidural hematoma is bleeding between the inner skull and the dura, compressing the brain underneath. An extradural hematoma is another name for an epidural hematoma.

To assess a client's cranial nerve function, a nurse should assess: hand grip. arm drifting. orientation to person, time, and place. gag reflex.

Correct response: gag reflex. Explanation: The gag reflex is governed by the glossopharyngeal nerve, cranial nerve IX. Hand grip and arm drifting are part of motor function assessment. Orientation is an assessment parameter related to a mental status examination.

A nurse caring for a patient with head trauma will be monitoring the patient for Cushing's triad. What will the nurse recognize as the symptoms associated with Cushing's triad? Select all that apply. Pupillary constriction Bradypnea Hypertension Tachycardia Bradycardia

Bradycardia Bradypnea Hypertension Explanation: At a certain point as intracranial pressure increases due to an injury, the brain's ability to autoregulate becomes ineffective and decompensation (ischemia and infarction) begins. When this occurs, the patient exhibits significant changes in mental status and vital signs. The bradycardia, hypertension, and bradypnea associated with this deterioration are known as Cushing's triad, which is a grave sign.

The nurse is caring for a client who is being assessed for brain death. Which are cardinal signs of brain death? Select all that apply. Apnea Absence of brainstem reflexes Coma No brain waves

Correct response: Absence of brainstem reflexes Apnea Coma Explanation: The three cardinal signs of brain death on clinical examination are coma, the absence of brain stem reflexes, and apnea. Adjunctive tests, such as cerebral blood flow studies, electroencephalography, transcranial Doppler, and brain stem auditory evoked potential, are often used to confirm brain death.

A client with cerebral metastasis suddenly experiences a seizure for which phenytoin 10 mg/kg intravenously is ordered as an initial loading dose. The client weighs 165 pounds. How many milligrams of phenytoin should the client receive? Enter the number ONLY.

Correct response: 750 Explanation: First, change the client's weight in pounds to kilograms by dividing the weight by 2.2 (2.2 pounds = 1 kg). The client's weight is 75 kg. Next, set up a proportion: 10/1 = x/75; cross multiply and solve for x, which is 750.

A client has sustained a head injury to the parietal lobe and cannot identify a familiar object by touch. The nurse knows that this deficit is A positive Romberg Visual agnosia Astereognosis Ataxia

Correct response: Astereognosis Explanation: Astereognosis is the inability to identify an object by touch. Visual agnosia is the loss of ability to recognize objects through visualizing them. The Romberg test has to do with balance. Ataxia is defined as incoordination of voluntary muscle action.

A patient has a deficiency of the neurotransmitter serotonin. The nurse is aware that this deficiency can lead to: Myasthenia gravis. Parkinson's disease. Seizures. Depression.

Correct response: Depression. Explanation: Serotonin helps control mood and sleep. A deficiency leads to depression. Chapter 60: p. 1967.

The nurse is completing an assessment on a client with myasthenia gravis. Which of the following historical recounting provides the most significant evidence regarding when the disorder began? Drooping eyelids Sensitivity to bright light Shortness of breath Muscle spasms

Correct response: Drooping eyelids Explanation: Ptosis (eyelid drooping) is the most common manifestation of myasthenia gravis. Muscle weakness varies depending on the muscles affected. Shortness of breath and respiratory distress occurs later as the disease progresses. Muscle spasms are more likely in multiple sclerosis. Photophobia is not significant in myasthenia gravis.

Excessive levels of which neurotransmitter has been implicated in amyotrophic lateral sclerosis (ALS)? Dopamine Serotonin Epinephrine Glutamate

Correct response: Glutamate Explanation: Excessive levels of the neuro-excitatory neurotransmitter glutamate have been implicated in the neurodegenerative diseases such as ALS, Huntington's disease, and the sequelae of strokes.

A nurse is caring for a client with lower back pain who is scheduled for myelography using metrizamide (a water-soluble contrast dye). After the test, the nurse should place the client in which position? Supine with feet raised Head of the bed elevated 45 degrees Prone Supine with the head lower than the trunk

Correct response: Head of the bed elevated 45 degrees Explanation: After a myelogram, positioning depends on the dye injected. When a water-soluble dye such as metrizamide is injected, the head of the bed is elevated to a 45-degree angle to slow the upward dispersion of the dye. The prone and supine positions are contraindicated when a water-soluble contrast dye is used. The client should be positioned supine with the head lower than the trunk after an air-contrast study.

A client has experienced an ischemic stroke that has damaged the lower motor neurons of the brain. Which of the following deficits would the nurse expect during assessment? Limited attention span and forgetfulness Lack of deep tendon reflexes Visual agnosia Auditory agnosia

Correct response: Lack of deep tendon reflexes Explanation: Damage to the occipital lobe can result in visual agnosia, whereas damage to the temporal lobe can cause auditory agnosia. If damage has occurred to the frontal lobe, learning capacity, memory, or other higher cortical intellectual functions may be impaired. Such dysfunction may be reflected in a limited attention span, difficulties in comprehension, forgetfulness, and lack of motivation. Damage to the lower motor neurons may cause decreased muscle tone, flaccid muscle paralysis, and a decrease in or loss of reflexes.

Which of the following drugs may be used after a seizure to maintain a seizure-free state? Cerebyx Ativan Valium Phenobarbital

Correct response: Phenobarbital Explanation: IV diazepam (Valium), lorazepam (Ativan), or fosphenytoin (Cerebyx) are administered slowly in an attempt to halt seizures immediately. Other medications (phenytoin, phenobarbital) are administered later to maintain a seizure-free state. In general, a single drug is used to control the seizures.

The nurse who is employed in a neurologist's office is performing a history and assessment on a client experiencing hearing difficulty. The nurse is most correct to gather equipment to assess the function of which cranial nerve? VIII VI II XI

Correct response: VIII Explanation: There are 12 pairs of cranial nerves. Cranial nerve VIII is the vestibulocochlear or auditory nerve responsible for hearing and balance. Cranial nerve II is the optic nerve. Cranial nerve VI is the abducens nerve responsible for eye movement. Cranial nerve XI is the accessory nerve and is involved with head and shoulder movement.

A patient has been diagnosed with meningococcal meningitis at a community living home. When should prophylactic therapy begin for those who have had close contact with the patient? Within 48 hours after exposure Within 72 hours after exposure Within 24 hours after exposure Therapy is not necessary prophylactically and should only be used if the person develops symptoms.

Correct response: Within 24 hours after exposure Explanation: People in close contact with patients with meningococcal meningitis should be treated with antimicrobial chemoprophylaxis using rifampin (Rifadin), ciprofloxacin hydrochloride (Cipro), or ceftriaxone sodium (Rocephin). Therapy should be started within 24 hours after exposure because a delay in the initiation of therapy limits the effectiveness of the prophylaxis. Chapter 64 p. 2090.

The nurse is performing a neurologic assessment on a client diagnosed with a stroke and cannot elicit a gag reflex. This deficit is related to which of the following cranial nerves? VIII X VII III

Correct response: X Explanation: CN X is the vagus nerve and has to do with the gag reflex, laryngeal hoarseness, swallowing ability, and the symmetrical rise of the uvula and soft palate. CN VII is the facial nerve and has to do with symmetry of facial movements and the ability to discriminate between the tastes of sugar and salt. The inability to close one eyelid indicates impairment of this nerve. CN VIII is the acoustic nerve. It has to do with hearing, air and bone conduction, and balance. CN III is the oculomotor nerve and has to do with pupillary response, conjugate movements, and nystagmus.

To assess a client's cranial nerve function, a nurse should assess: arm drifting. orientation to person, time, and place. hand grip. gag reflex.

Correct response: gag reflex. Explanation: The gag reflex is governed by the glossopharyngeal nerve, cranial nerve IX. The other choices would not be involved in a cranial nerve assessment. Hand grip and arm drifting are part of motor function assessment. Orientation is an assessment parameter related to a mental status examination.

When communicating with a client who has sensory (receptive) aphasia, the nurse should: allow time for the client to respond. use short, simple sentences. give the client a writing pad. speak loudly and articulate clearly.

Correct response: use short, simple sentences. Explanation: Although sensory aphasia allows the client to hear words, it impairs the ability to comprehend their meaning. The nurse should use short, simple sentences to promote comprehension. Allowing time for the client to respond might be helpful but is less important than simplifying the communication. Because the client's hearing isn't affected, speaking loudly isn't necessary. A writing pad is helpful for clients with expressive, not receptive, aphasia.

A nurse is caring for a client who has traumatic brain injury. Which of the following findings should the nurse identify as an indication of increased intracranial pressure (ICP)? a. Restlessness b. Hypotension c. Amnesia d. Tachycardia

Restlessness

A nurse suspects that a client admitted for treatment of bacterial meningitis is experiencing increased intracranial pressure (ICP). Which of the following assessment findings by the nurse supports this suspicion? Photophobia Nuchal rigidity Positive Kernig's sign Restlessness

Restlessness

Students are reviewing information about the stages of pressure ulcer development. They demonstrate understanding when they identify which stage as characterized by a full-thickness wound? Select all that apply. Stage IV Stage III Stage I Deep tissue injury Stage II

Stage III Stage IV Explanation: Stages III and IV pressure ulcers are considered full-thickness wounds. Deep tissue injury is a localized area of discolored, purple, intact skin or blood-filled blister caused by underlying soft tissue damage from pressure or shear. Stage I ulcer is characterized by erythema (superficial reddening of the skin). Stage II pressure ulcer is a partial-thickness wound.

A nurse is performing a baseline assessment of a client's skin integrity. What are the priority assessments? Select all that apply. presence of pressure ulcers on the client indwelling catheter output potential areas of pressure ulcer development overall risk of developing pressure ulcers family history of pressure ulcers

Correct response presence of pressure ulcers on the client overall risk of developing pressure ulcers potential areas of pressure ulcer development Explanation: When assessing skin integrity, the overall risk potential of developing pressure ulcers takes priority. Overall risk assessment encompasses review of existing pressure ulcers as well as potential areas for development of pressure ulcers. Foley catheter output and family history of pressure ulcers are not important when assessing skin integrity.

A nurse is providing education about migraine headaches to a community group. The cause of migraines has not been clearly demonstrated, but is related to vascular disturbances. A member of the group asks about familial tendencies. The nurse's correct reply will be which of the following? "Only secondary migraine headaches show a familial tendency." "There is a very weak familial tendency." "No familial tendency has been demonstrated." "There is a strong familial tendency."

Correct response: "There is a strong familial tendency." Explanation: Migraine headaches have a strong familial tendency. Migraines are primary headaches, not secondary headaches.

Which disease includes loss of motor neurons in the anterior horns of the spinal cord and motor nuclei of the lower brain stem? Huntington disease Amyotrophic lateral sclerosis Parkinson disease Alzheimer disease

Correct response: Amyotrophic lateral sclerosis Explanation: Amyotrophic lateral sclerosis (ALS) is a disease of unknown cause in which there is a loss of motor neurons in the anterior horns of the spinal cord and the motor nuclei of the lower brain stem. Parkinson disease is a slowly progressing neurologic movement disorder that eventually leads to disability. Alzheimer disease is a chronic, progressive, and degenerative brain disorder that is accompanied by profound effects on memory, cognition and ability for self-care. Huntington disease is a chronic, progressive, hereditary disease of the nervous system that results in progressive involuntary choreiform movement and dementia.

A male client who has undergone a cervical discectomy is being discharged with a cervical collar. Which of the following would be most appropriate to include the client's discharge plan? Moving the neck from side to side when the collar is off Wearing the cervical collar when sleeping Removing the entire collar when shaving Keeping the head in a neutral position

Correct response: Keeping the head in a neutral position Explanation: After a cervical discectomy, the client typically wears a cervical collar. The client should be instructed to keep his head in a neutral position and wear the collar at all times unless the physician has instructed otherwise. The front part of the collar is removed for shaving and the neck should be kept still while the collar is open or off.

A nurse is preparing to administer an antiseizure medication to a client. Which of the following is an appropriate antiseizure medication? Lamictal Labetalol Lamisil Lomotil

Correct response: Lamictal Explanation: Lamictal is an antiseizure medication. Its packaging was recently changed in an attempt to reduce medication errors, because this medication has been confused with Lamisil (an antifungal), labetalol (an antihypertensive), and Lomotil (an antidiarrheal).

A client who was trapped inside a car for hours after a head-on collision is rushed to the emergency department with multiple injuries. During the neurologic examination, the client responds to painful stimuli with decerebrate posturing. This finding indicates damage to which part of the brain? Midbrain Diencephalon Cortex Medulla

Correct response: Midbrain Explanation: Damage to the midbrain causes decerebrate posturing that's characterized by abnormal extension in response to painful stimuli. With damage to the diencephalon or cortex, abnormal flexion (decorticate posturing) occurs when a painful stimulus is applied. Damage to the medulla results in flaccidity.

When educating a patient about the use of antiseizure medication, what should the nurse inform the patient is a result of long-term use of the medication in women? Obesity Anemia Osteoarthritis Osteoporosis

Correct response: Osteoporosis Explanation: Because of bone loss associated with the long-term use of antiseizure medications, patients receiving antiseizure agents should be assessed for low bone mass and osteoporosis. They should be instructed about strategies to reduce their risks of osteoporosis (AANN, 2009). Chapter 61: p. 2022.

A client presents to the emergency department stating numbness and tingling occurring down the left leg into the left foot. When documenting the experience, which medical terminology would the nurse be most correct to report? Paralysis Herniation Sciatic nerve pain Paresthesia

Correct response: Paresthesia Explanation: When a client reports numbness and tingling in an area, the client is reporting a paresthesia. The nurse would document the experience as such or place the client's words in parentheses. The nurse would not make a medical diagnosis of sciatic nerve pain or herniation. The symptoms are not consistent with paralysis.

During a neurological assessment examination, the nurse assesses a patient for tactile agnosia. The nurse places a familiar door key in the patient's hand and asks him to identify the object with his eyes closed. The nurse documents his inability to identify the object and notes the affected area of the brain. Which of the following is the most likely affected area of the brain? Occipital lobe Frontal lobe Parietal lobe Temporal lobe

Correct response: Parietal lobe Explanation: The parietal lobe analyzes sensor information and relays the interpretation to the cortical area. Failure to identify a familiar object by touch is indicative of parietal lobe dysfunction. Refer to Table 43-7

A patient has a nursing diagnosis of risk for impaired skin integrity related to immobility and secondary to diabetes. As part of the plan of care, the nurse plans to reposition the patient frequently. Based on an understanding of positioning and its effects, the nurse identifies which position as preferred to the semi-Fowler's position? Recumbent Fowler's Prone Lateral

Correct response: Recumbent Explanation: Although a patient should be repositioned laterally, prone, and dorsally in sequence, the recumbent position is preferred to the semi-Fowler's position because this position provides an increased body surface area of support.

When should the nurse plan the rehabilitation of a patient who is having an ischemic stroke? After the nurse has received the discharge orders After the patient has passed the acute phase of the stroke The day before the patient is discharged The day the patient has the stroke

Correct response: The day the patient has the stroke Explanation: Although rehabilitation begins on the day the patient has the stroke, the process is intensified during convalescence and requires a coordinated team effort.

If a client has a lower motor neuron lesion, the nurse would expect the client to exhibit muscle spasticity. hyperactive reflexes. no muscle atrophy. decreased muscle tone.

Correct response: decreased muscle tone. Explanation: A client with a lower motor neuron lesion would be expected to have decreased muscle tone. Those with upper motor neuron lesions would have hyperactive reflexes, no muscle atrophy, and muscle spasticity.

A client with a neurologic deficit is feeling frustrated because it is very difficult to pronounce words since having a stroke. The client is struggling with: ataxia. dysphasia. dysarthria. dysphagia.

Correct response: dysarthria. Explanation: Dysarthria is characterized by poor articulation of words due to muscle weakness or loss of muscle control. Dysphasia is characterized by the compromised ability to put words together meaningfully. Ataxia is a dysfunction of the parts of the nervous system that coordinate movement. Dysphagia is difficulty with swallowing.

A client is suspected of having amyotrophic lateral sclerosis (ALS). To help confirm this disorder, the nurse prepares the client for various diagnostic tests. The nurse expects the physician to order: electromyography (EMG). Doppler ultrasonography. Doppler scanning. quantitative spectral phonoangiography.

Correct response: electromyography (EMG). Explanation: To help confirm ALS, the physician typically orders EMG, which detects abnormal electrical activity of the involved muscles. To help establish the diagnosis of ALS, EMG must show widespread anterior horn cell dysfunction with fibrillations, positive waves, fasciculations, and chronic changes in the potentials of neurogenic motor units in multiple nerve root distribution in at least three limbs and the paraspinal muscles. Normal sensory responses must accompany these findings. Doppler scanning, Doppler ultrasonography, and quantitative spectral phonoangiography are used to detect vascular disorders, not muscular or neuromuscular abnormalities.

A patient is treated for a neurologic dysfunction affecting facial expressions. The affected cranial nerve originates in the: pons. cerebral hemisphere. medulla. midbrain.

Correct response: pons. Explanation: Cranial nerves V through VIII connect to the brain in the pons. Cranial nerve VII (facial nerve) affects facial expressions and muscle movements.

To evaluate a client's cerebellar function, a nurse should ask: "Do you have any trouble swallowing food or fluids?" "Have you noticed any changes in your muscle strength?" "Do you have any problems with balance?" "Do you have any difficulty speaking?"

Correct response: "Do you have any problems with balance?" Explanation: To evaluate cerebellar function, the nurse should ask the client about problems with balance and coordination. The nurse asks about difficulty speaking or swallowing to assess the functions of cranial nerves IX, X, and XII. Questions about muscle strength help her evaluate the client's motor system. Chapter 60:p. 1969.

Which of the following is a late symptom of spinal cord compression? Paralysis Fecal incontinence Urinary incontinence Urinary retention

Correct response: Paralysis Explanation: Later symptoms include evidence of motor weakness and sensory deficits progressing to paralysis. Early symptoms associated with spinal cord compression include bladder and bowel dysfunction (urinary incontinence or retention; fecal incontinence or constipation).

A patient was body surfing in the ocean and sustained a cervical spinal cord fracture. A halo traction device was applied. How does the patient benefit from the application of the halo device? It is the only device that can be applied for stabilization of a spinal fracture. It is less bulky and traumatizing for the patient to use. The patient can remove it as needed. It allows for stabilization of the cervical spine along with early ambulation.

Correct response: It allows for stabilization of the cervical spine along with early ambulation. Explanation: Halo devices provide immobilization of the cervical spine while allowing early ambulation. Chapter 63: Page 2075

Which lobe of the brain is responsible for spatial relationships? Temporal Occipital Parietal Frontal

Correct response: Parietal Explanation: The parietal lobe is essential to a person's awareness of body position in space, size and shape discrimination, and right-left orientation. The frontal lobe controls information storage or memory and motor function. The temporal lobe contains the auditory receptive area. The occipital lobe is responsible for visual interpretation.

A patient has an S5 spinal fracture from a fall. What type of assistive device will this patient require? Cane Voice or sip-n-puff controlled electric wheelchair Electric or modified manual wheelchair, needs transfer assistance The patient will be able to ambulate independently.

Correct response: The patient will be able to ambulate independently. Explanation: Patients with spinal cord injuries from S1 to S5 should be able to ambulate independently, without an assistive device.

What is one of the earliest signs of increased ICP? coma decreased level of consciousness (LOC) headache Cushing's triad

Correct response: decreased level of consciousness (LOC) Explanation: Headache is a symptom of increased ICP, but decreasing LOC is one of the earliest signs of increased ICP. Cushing's triad occurs late in increased ICP. If untreated, increasing ICP will lead to coma. Chapter 61:p. 2009.

The nurse is seeing a client who has just been diagnosed with a meningioma. The client states he is confused because the provider stated, "If you have to be diagnosed with a brain tumor, this is the least harmful." The client asks the nurse for clarification. How should the nurse respond? "I am assuming your provider was trying to explain to you that meningiomas have a high cure rate if treated with surgery, chemotherapy and radiation aggressively." "I am unable to interpret what your provider meant by making that statement; however, it is true that meningiomas are slow growing tumors that are not typically fatal." "It would have been important for you to clarify your provider's statement during your appointment. It is not within my scope to discuss the details of your diagnosis." "It is likely that your provider was trying to be as supportive as possible with those positive words. You need a lot of support during this challenging time."

Correct response: "I am unable to interpret what your provider meant by making that statement; however, it is true that meningiomas are slow growing tumors that are not typically fatal." Explanation: The nurse should inform the client that nurses cannot interpret what another provider meant by the statement. The nurse can provide client education regarding what is known about the type of brain tumor the client has been diagnosed with. The tumor is slow growing. and sometime treatment is a 'wait-and-see' approach. Thus, surgery, chemotherapy and radiation would not typically be used together or aggressively.

The initial sign of skin pressure is erythema, which normally resolves in less than 1 hour. 15 minutes. 45 minutes. 30 minutes.

Correct response: 1 hour. Explanation: The initial sign of pressure is erythema caused by reactive hyperemia, which normally resolves in less than 1 hour. All of the other time frames are incorrect.

A patient is admitted to the hospital with an ICP reading of 20 mm Hg and a mean arterial pressure of 90 mm Hg. What would the nurse calculate the CPP to be? 80 mm Hg 60 mm Hg 70 mm Hg 50 mm Hg

Correct response: 70 mm Hg Explanation: Changes in ICP are closely linked with cerebral perfusion pressure (CPP). The CPP is calculated by subtracting the ICP from the mean arterial pressure (MAP). For example, if the MAP is 100 mm Hg and the ICP is 15 mm Hg, then the CPP is 85 mm Hg. The normal CPP is 70 to 100 mm Hg Chapter 61: Page 2000

What diet can the nurse recommend to a patient with hypoproteinemia that spares protein? A diet high in minerals A diet high in vitamins A diet high in fats A diet high in carbohydrates

Correct response: A diet high in carbohydrates Explanation: Wounds from which body fluids and protein drain place the patient in a catabolic state and predispose to hypoproteinemia and serious secondary infections. Protein deficiency must be corrected to promote the healing of the pressure ulcer. Carbohydrates are necessary to "spare" the protein and to provide an energy source.

A client with quadriplegia is in spinal shock. What finding should the nurse expect? Hyperreflexia along with spastic extremities Absence of reflexes along with flaccid extremities Spasticity of all four extremities Positive Babinski's reflex along with spastic extremities

Correct response: Absence of reflexes along with flaccid extremities Explanation: During the period immediately following a spinal cord injury, spinal shock occurs. In spinal shock, all reflexes are absent and the extremities are flaccid. When spinal shock subsides, the client will demonstrate positive Babinski's reflex, hyperreflexia, and spasticity of all four extremities.

Which cranial nerve is tested by listening to a ticking watch? Facial Trigeminal Acoustic Vagus

Correct response: Acoustic Explanation: The acoustic nerve (VIII) assesses hearing by rubbing the fingers, placing a ticking watch, or whispering near each ear. The facial nerve (VII) is assessed for symmetry of facial movement. The trigeminal nerve (V) is assessed for facial sensation, corneal reflex, and chewing or mastication. The vagus nerve (X) is assessed by swallowing and gag reflex.

A community health nurse is giving an educational presentation about stroke and heart disease at the local senior citizens' center. What nonmodifiable risk factor for stroke should the nurse cite? Asian race Smoking Female gender Advanced age

Correct response: Advanced age Explanation: Advanced age, male gender, and race are well-known nonmodifiable risk factors for stroke. Smoking is a modifiable risk.

A nurse is working with a patient to establish a bowel training program. Based on the nurse's understanding of bowel function, the nurse would suggest planning for bowel evacuation at which time? Upon arising After breakfast Around lunchtime Before bed

Correct response: After breakfast Explanation: Natural gastrocolic and duodenocolic reflexes occur about 30 minutes after a meal; therefore, after breakfast is one of the best times to plan for bowel evacuation.

A patient sustained a head trauma in a diving accident and has a cerebral hemorrhage located within the brain. What type of hematoma is this classified as? An extradural hematoma An epidural hematoma A subdural hematoma An intracerebral hematoma

Correct response: An intracerebral hematoma Explanation: Intracerebral hemorrhage (hematoma) is bleeding within the brain, into the parenchyma of the brain. It is commonly seen in head injuries when force is exerted to the head over a small area (e.g., missile injuries, bullet wounds, stab injuries). A subdural hematoma (SDH) is a collection of blood between the dura and the brain, a space normally occupied by a thin cushion of cerebrospinal fluid. After a head injury, blood may collect in the epidural (extradural) space between the skull and the dura.

Which medication classification should be avoided in the treatment of brain tumors? Anticoagulants Anticonvulsants Corticosteroids Osmotic diuretics

Correct response: Anticoagulants Explanation: Anticoagulants usually are not prescribed because of the risk for central nervous system (CNS) hemorrhage; however, prophylactic therapy with low-molecular-weight heparin is under investigation. Osmotic diuretics, corticosteroids, and anticonvulsants are utilized in the treatment of brain tumors.

A client is ordered to undergo CT of the brain with IV contrast. Before the test, the nurse should complete which action first? Obtain a blood sample to evaluate BUN and creatinine concentrations. Assess the client for medication allergies. Maintain the client NPO for 6 hours before the test. Obtain two large-bore IV lines.

Correct response: Assess the client for medication allergies. Explanation: If a contrast agent is used, the client must be assessed before the CT scan for an iodine/shellfish allergy, because the contrast agent used may be iodine based. If the client has no allergies to iodine, then kidney function must also be evaluated, as the contrast material is cleared through the kidneys. A suitable IV line for contrast injection and a period of fasting (usually 4 hours) are required before the study. Clients who receive an IV contrast agent are monitored during and after the procedure for allergic reactions and changes in kidney function. Chapter 60: p. 1986.

Which term refers to the inability to coordinate muscle movements, resulting difficulty walking? Agnosia Rigidity Spasticity Ataxia

Correct response: Ataxia Explanation: Ataxia is the inability to coordinate voluntary muscle action; tremors (rhythmic, involuntary movements) noted at rest or during movement suggest a problem in the anatomic areas responsible for balance and coordination. Agnosia is the loss of ability to recognize objects through a particular sensory system. Spasticity is the sustained increase in tension of a muscle when it is passively lengthened or stretched.

The nurse is assessing the pupils of a patient who has had a head injury. What does the nurse recognize as a parasympathetic effect? One pupil is dilated and the opposite pupil is normal Dilated pupils Roth's spots Constricted pupils

Correct response: Constricted pupils Explanation: Constricted pupils are a parasympathetic effect; dilated pupils are a sympathetic effect.

A nurse is working in the neurologic intensive care unit and admits from the emergency department a patient with an inoperable brain tumor. Upon entering the room, the nurse observes that the patient is positioned like the person in part B of the accompanying image. Which posturing is the patient exhibiting? Flaccidity Tonic clonic Decorticate Decerebrate

Correct response: Decerebrate Explanation: An inappropriate or nonpurposeful response is random and aimless. Posturing may be decorticate or decerebrate. Decorticate posture is the flexion and internal rotation of forearms and hands. Decerebrate posture is extension and external rotation. Flaccidity is the absence of motor response; tonic clonic movements are seen with seizures. Chapter 61: p. 1995.

Which posture exhibited by abnormal flexion of the upper extremities and extension of the lower extremities? Flaccid Normal Decorticate Decerebrate

Correct response: Decorticate Explanation: Decorticate posturing is an abnormal posture associated with severe brain injury, characterized by abnormal flexion of the upper extremities and extension of the lower extremities. Decerebration is an abnormal body posture associated with a severe brain injury, characterized by extreme extension of the upper and lower extremities. Flaccidity occurs when the client has no motor function, is limp, and lacks motor tone.

The nurse is caring for a client who is hospitalized with an exacerbation of MS. To ensure the client's safety, what nursing action should be performed? Maintain bed rest whenever possible. Pad the client's bed rails. Provide several small meals each day. Ensure that suction apparatus is set up at the bedside.

Correct response: Ensure that suction apparatus is set up at the bedside. Explanation: Because of the client's risk of aspiration, it is important to have a suction apparatus at hand. Bed rest should be generally be minimized, not maximized, and there is no need to pad the client's bed rails or to provide multiple small meals.

A client with a traumatic brain injury has developed increased intracranial pressure resulting in diabetes insipidus. While assessing the client, the nurse expects which of the following findings? Oliguria and serum hyperosmolarity Excessive urine output and serum hypo-osmolarity Oliguria and decreased urine osmolality Excessive urine output and decreased urine osmolality

Correct response: Excessive urine output and decreased urine osmolality Explanation: Diabetes insipidus is the result of decreased secretion of antidiuretic hormone (ADH). The client has excessive urine output, decreased urine osmolality, and serum hyperosmolarity.

A nurse is caring for a client with lower back pain who is scheduled for myelography using metrizamide (a water-soluble contrast dye). After the test, the nurse should place the client in which position? Prone Supine with the head lower than the trunk Supine with feet raised Head of the bed elevated 45 degrees

Correct response: Head of the bed elevated 45 degrees Explanation: After a myelogram, positioning depends on the dye injected. When a water-soluble dye such as metrizamide is injected, the head of the bed is elevated to a 45-degree angle to slow the upward dispersion of the dye. The prone and supine positions are contraindicated when a water-soluble contrast dye is used. The client should be positioned supine with the head lower than the trunk after an air-contrast study.

The sympathetic and parasympathetic nervous systems have a direct effect on the circulatory system. Stimulation of the parasympathetic nervous system (PNS) causes which of the following? Blood vessels in the skeletal muscles to dilate Blood vessels in the heart muscle to dilate Heartbeat to decrease Blood pressure to increase

Correct response: Heartbeat to decrease Explanation: The parasympathetic nervous system has a constricting effect on the blood vessels in the heart and skeletal muscles; the heartbeat and blood pressure will decrease.

A client in the emergency department has a suspected neurologic disorder. To assess gait, the nurse asks the client to take a few steps; with each step, the client's feet make a half circle. To document the client's gait, the nurse should use which term Dystrophic Steppage Helicopod Ataxic

Correct response: Helicopod Explanation: A helicopod gait is an abnormal gait in which the client's feet make a half circle with each step. An ataxic gait is staggering and unsteady. In a dystrophic gait, the client waddles with the legs far apart. In a steppage gait, the feet and toes rise high off the floor and the heel comes down heavily with each step.

A client is diagnosed with a brain angioma. When providing care to this client, the nurse would be especially vigilant in monitoring for signs and symptoms of which of the following? Hemorrhagic stroke Hydrocephalus Infection Seizures

Correct response: Hemorrhagic stroke Explanation: Although any brain tumor may lead to seizures and hydrocephalus, a client with an angioma is at high risk for hemorrhagic stroke because the walls of the blood vessels in angiomas are thin. Infection is a possibility but would be considered less likely.

A healthcare provider orders several drugs for a client with hemorrhagic stroke. Which drug order should the nurse question? Heparin sodium Phenytoin Dexamethasone Methyldopa

Correct response: Heparin sodium Explanation: Administering heparin, an anticoagulant, could increase the bleeding associated with hemorrhagic stroke. Therefore, the nurse should question this order to prevent additional hemorrhage in the brain. In a client with hemorrhagic stroke, the healthcare provider may use dexamethasone (Decadron) to decrease cerebral edema and pressure, methyldopa (Aldomet) to reduce blood pressure, and phenytoin (Dilantin) to prevent seizures.

Which disturbance results in loss of half of the visual field? Homonymous hemianopsia Nystagmus Anisocoria Diplopia

Correct response: Homonymous hemianopsia Explanation: Homonymous hemianopsia (loss of half of the visual field) may occur from stroke and may be temporary or permanent. Double vision is documented as diplopia. Nystagmus is ocular bobbing and may be seen in multiple sclerosis. Anisocoria is unequal pupils.

The nurse is caring for a client with a ventriculostomy. Which assessment finding demonstrates effectiveness of the ventriculostomy? Cerebral perfusion pressure (CPP) is 21 mm Hg. Increased ICP is 12 mm Hg. The mean arterial pressure (MAP) is equal to the intracranial pressure (ICP). The pupils are dilated and fixed.

Correct response: Increased ICP is 12 mm Hg. Explanation: A ventriculostomy is used to continuously measure ICP and allows cerebral spinal fluid to drain, especially during a period of increased ICP. Normal ICP is 0 to 15 mm Hg, so ICP measured at 12 mm Hg would demonstrate the effectiveness of the ventriculostomy. Dilated and fixed pupils are not a normal assessment finding Cerebral circulation ceases if the ICP is equal to the MAP. Normal CPP is 70 to 100. A CPP reading less than 50 is consistent with irreversible neurologic damage.

A client is receiving an IV infusion of mannitol (Osmitrol) after undergoing intracranial surgery to remove a brain tumor. To confirm that this drug is producing its therapeutic effect, the nurse should consider which finding most significant? Decreased heart rate Decreased level of consciousness (LOC) Elevated blood pressure Increased urine output

Correct response: Increased urine output Explanation: The therapeutic effect of mannitol is diuresis, which is confirmed by an increased urine output. A decreased LOC and elevated blood pressure may indicate lack of therapeutic effectiveness. A decreased heart rate doesn't indicate that mannitol is effective. Chapter 62: p. 2037.

Which of the following is a brain tumor arising from the supporting structures? Glioblastoma multiforme Astrocytomas Medulloblastoma Meningiomas

Correct response: Meningiomas Explanation: Brain tumors arising from the supporting structures include meningiomas, neuromas, and pituitary adenomas. Intracerebral tumors include astrocytomas, medulloblastoma, and glioblastoma multiforme.

A nurse helps a patient recently diagnosed with a pituitary adenoma understand that: Most tumors are malignant (>90%). Transcranial surgery is usually necessary to remove the tumor. The cause is directly related to prior exposure to radiation. Most tumors produce too much of one or more hormones.

Correct response: Most tumors produce too much of one or more hormones. Explanation: The majority of these tumors are benign. In rare cases, they may be malignant. Functioning tumors produce hormones, frequently in excessive amounts, resulting in conditions such as hyperthyroidism, Cushing's syndrome, and gigantism or acromegaly.

During a neurological assessment examination, the nurse assesses a patient for tactile agnosia. The nurse places a familiar door key in the patient's hand and asks him to identify the object with his eyes closed. The nurse documents his inability to identify the object and notes the affected area of the brain. Which of the following is the most likely affected area of the brain? Occipital lobe Temporal lobe Parietal lobe Frontal lobe

Correct response: Parietal lobe Explanation: The parietal lobe analyzes sensor information and relays the interpretation to the cortical area. Failure to identify a familiar object by touch is indicative of parietal lobe dysfunction

A client has an exacerbation of multiple sclerosis. The physician orders dantrolene (Dantrium), 25 mg P.O. daily. Which assessment finding indicates the medication is effective? Relief from constipation Relief from pain Reduced muscle spasticity Increased ability to sleep

Correct response: Reduced muscle spasticity Explanation: Dantrolene reduces muscle spasticity. It doesn't increase the ability to sleep or relieve constipation or pain.

A patient with amyotrophic lateral sclerosis (ALS) asks if the nurse has heard of a drug that will prolong the patient's life. The nurse knows that there is a medication that may prolong the life by 3 to 6 months. To which medication is the patient referring? Riluzole Baclofen Diazepam Dantrolene sodium

Correct response: Riluzole Explanation: Riluzole, a glutamate antagonist, has been shown to prolong survival for persons with ALS for 3 to 6 months.

Autonomic dysreflexia can occur with spinal cord injuries above which of the following levels? L4 S2 T10 T6

Correct response: T6 Explanation: Any patient with a lesion above T6 segment is informed that autonomic dysreflexia can occur and that it may occur even years after the initial injury. Chapter 63: p. 2078.

Which of the following is not a manifestation of Cushing's triad (Cushing reflex)? Hypertension Widening pulse pressure Irregular respiration Tachycardia

Correct response: Tachycardia Explanation: Cushing's triad, or Cushing reflex, is a nervous system response to increased intracranial pressure. The client has a slower heart rate (bradycardia), higher systolic blood pressure (hypertension) with lower diastolic pressure (widening pulse pressure), and irregular respiration. More rapid heart rate (tachycardia) is not a component of the triad.

Which of the following tests confirms the diagnosis of myasthenia gravis (MG)? Serum studies Tensilon test Computed tomography (CT) scan Electromyogram (EMG)

Correct response: Tensilon test Explanation: Edrophonium chloride (Tensilon) is an acetylcholinesterase inhibitor that stops the breakdown of acetylcholine. The drug is used because it has a rapid onset of 30 seconds and a short duration of 5 minutes. Immediate improvement in muscle strength after administration of this agent represents a positive test and usually confirms the diagnosis. The presence of acetylcholine receptor antibodies is identified in serum. Repetitive nerve stimulation demonstrates a decrease in successive action potentials. The thymus gland may be enlarged in MG, and a T scan of the mediastinum is performed to detect thymoma or hyperplasia of the thymus.

A patient comes to the emergency department with severe pain in the face that was stimulated by brushing the teeth. What cranial nerve does the nurse understand can cause this type of pain? IV V III VI

Correct response: V Explanation: The trigeminal nerve (cranial nerve V) innervates the forehead, cheeks, and jaw, so pain in the face elicited when brushing the teeth would most likely involve this nerve

Which finding indicates increasing intracranial pressure (ICP) in the client who has sustained a head injury? Widened pulse pressure Increased pulse Decreased body temperature Increased respirations

Correct response: Widened pulse pressure Explanation: Signs of increasing ICP include slowing of the heart rate (bradycardia), increasing systolic blood pressure, and widening pulse pressure (Cushing reflex). As brain compression increases, respirations decrease or become erratic, blood pressure may decrease, and the pulse slows further. This is an ominous development, as is a rapid fluctuation of vital signs. Temperature is maintained at less than 38°C (100.4°F). Tachycardia and arterial hypotension may indicate that bleeding is occurring elsewhere in the body.

The initial sign of increasing intracranial pressure (ICP) includes sore throat. herniation. vomiting. decreased level of consciousness.

Correct response: decreased level of consciousness. Explanation: The initial signs of increasing ICP include decreased level of consciousness and focal motor deficits. If ICP is not controlled, the uncus of the temporal lobe may be herniated through the tentorium, causing pressure on the brain stem. Vomiting and sore throat are not initial signs of increasing ICP.

A nurse is completing a neurological assessment and determines that the client has significant visual deficits. Considering the functions of the lobes of the brain, which area will most likely contain the neurologic deficit? frontal temporal parietal occipital

Correct response: occipital Explanation: The vision center is located in the occipital lobe. There is little that may interfere with the visual process in the other lobes of the brain.

Age-related changes in the neurologic system must be carefully assessed. Which of the following changes does the nurse expect to find in some degree depending on the patient's age and medical condition? Select all that apply. Stage IV sleep is prolonged Decreased muscle mass Hyper-reactive deep tendon reflexes Reduced papillary responses Increased sensitivity to heat and cold Increased sensitivity of taste buds

Decreased muscle mass Increased sensitivity to heat and cold Reduced papillary responses Explanation: Refer to Table 43-5 in the text.

A client has sustained a head injury to the parietal lobe and cannot identify a familiar object by touch. The nurse knows that this deficit is Astereognosis Visual agnosia A positive Romberg Ataxia

Correct response: Astereognosis Explanation: Astereognosis is the inability to identify an object by touch. Visual agnosia is the loss of ability to recognize objects through visualizing them. Romberg test has to do with balance. Ataxia is defined as incoordination of voluntary muscle action.

The nurse is providing education to a group of young people about the dangers of tattoos and body piercings. Which of the following would the nurse describe as a possible result of a tongue piercing? Damage to cranial nerve V Strep throat infection Brain abscess Otitis media

Correct response: Brain abscess Explanation: A brain abscess can result from intracranial surgery, penetrating head injury, or tongue piercing. The other choices are not associated with tongue piercing.

A client with a spinal cord injury has full head and neck control when the injury is at which level? C1 C2 to C3 C4 C5

Correct response: C5 Explanation: At level C5, the client retains full head and neck control. At C1 the client has little or no sensation or control of the head and neck. At C2 to C3 the client feels head and neck sensation and has some neck control. At C4 the client has good head and neck sensation and motor control.

A nurse is working in a neurologist's office. The physician orders a Romberg test. What should the nurse instruct the client to do? Touch nose with one finger. Close eyes and discriminate between dull and sharp. Close eyes and stand erect. Close eyes and jump on one foot.

Correct response: Close eyes and stand erect. Explanation: In the Romberg test, the client stands erect with the feet close together and eyes closed. If the client sways as if to fall, it is considered a positive Romberg test. All of the other options include components of neurologic tests, indicating neurologic deficits and balance.

There are 12 pairs of cranial nerves. Only three are sensory. Select the cranial nerve that is affected with decreased visual fields. Cranial nerve IV Cranial nerve II Cranial nerve III Cranial nerve I

Correct response: Cranial nerve II Explanation: The three sensory cranial nerves are I, II and VIII. Cranial nerve II (optic) is affected with decreased visual fields and acuity.

A diagnostic test has determined that the appropriate diet for the client with a left cerebrovascular accident (CVA) should include thickened liquids. Which of the following is the priority nursing diagnosis for this client? Malnutrition Risk Impaired Swallowing Aspiration Risk Decreased Fluid Volume Risk

Correct response: Impaired Swallowing Explanation: Impaired Swallowing was evident on the video fluoroscopy. Aspiration, Malnutrition, and Decreased Fluid Volume Risk can occur but are not the primary diagnosis at this point in time.

Which is a contraindication for the administration of tissue plasminogen activator (t-PA)? Ischemic stroke Systolic blood pressure less than or equal to 185 mm Hg Intracranial hemorrhage Age 18 years or older

Correct response: Intracranial hemorrhage Explanation: Intracranial hemorrhage, neoplasm, and aneurysm are contraindications for t-PA. Clinical diagnosis of ischemic stroke, age 18 years or older, and a systolic blood pressure less than or equal to 185 mm Hg are eligibility criteria. Chapter 62: Page 2048

A frontal lobe brain abscess produces which manifestation? Nystagmus Seizures Localized headache Ataxia

Correct response: Seizures Explanation: A frontal lobe brain abscess produces seizures, hemiparesis, and frontal headache. A temporal lobe brain abscess is manifested by localized headache. A cerebellar abscess is manifested by ataxia and nystagmus.

A client is hospitalized with Guillain-Barré syndrome. Which nursing assessment finding is most significant? Warm, dry skin Uneven, labored respirations Soft, nondistended abdomen Urine output of 40 ml/hour

Correct response: Uneven, labored respirations Explanation: A characteristic feature of Guillain-Barré syndrome is ascending weakness, which usually begins in the legs and progresses upward to the trunk, arms, and face. Respiratory muscle weakness, evidenced by uneven, labored respirations, is a particularly dangerous effect of this disease progression because it may lead to respiratory failure and death. Therefore, although warm, dry skin; urine output of 40 ml/hour; and a soft, nondistended abdomen are pertinent assessment data, those related to respiratory function and status are most significant.

A patient had a small pituitary adenoma removed by the transsphenoidal approach and has developed diabetes insipidus. What pharmacologic therapy will the nurse be administering to this patient to control symptoms? Phenobarbital Vasopressin Mannitol Furosemide (Lasix)

Correct response: Vasopressin Explanation: Manipulation of the posterior pituitary gland during surgery may produce transient diabetes insipidus of several days' duration It is treated with vasopressin but occasionally persists.

When is the optimal time for the nurse to begin the rehabilitation process for a patient with a cervical spine injury? When an exercise program has been initiated With initial patient contact After the patient feels comfortable in the clinical setting After the physician has prescribed rehabilitative goals

Correct response: With initial patient contact Explanation: The principles of rehabilitation are basic to the care of all patients, and rehabilitation efforts should begin during the initial contact with a patient.

When is the optimal time for the nurse to begin the rehabilitation process for a patient with a cervical spine injury? With initial patient contact After the physician has prescribed rehabilitative goals After the patient feels comfortable in the clinical setting When an exercise program has been initiated

Correct response: With initial patient contact Explanation: The principles of rehabilitation are basic to the care of all patients, and rehabilitation efforts should begin during the initial contact with a patient.

Which activities would the client with a T4 spinal cord injury be able to perform independently? Select all that apply. Breathing Transferring to a wheelchair Writing Eating Ambulating

Eating Breathing Transferring to a wheelchair Writing Explanation: Eating, breathing, transferring to a wheelchair, and writing are functional abilities for those with a T4 injury. Ambulation can be performed independently by a client with an injury at T11-S5 injury.

At which of the following spinal cord injury levels does the patient have full head and neck control? C3 C5 C2 C4

orrect response: C5 Explanation: At the level of C5, the patient should have full head and neck control, shoulder strength, and elbow flexion. At C4 injury, the patient will have good head and neck sensation and motor control, some shoulder elevation, and diaphragm movement. At C2 to C3, the patient will have head and neck sensation, some neck control, and can be independent of mechanical ventilation for short periods of time.

The nurse is assessing a 78-year-old female client admitted with a stroke of recent onset, within 2 hours of admission. Vital signs: blood pressure, 150/90 mm Hg; pulse, 112 beats/min; respirations, 20 breaths/min; temperature, 100.4°F (38°C); pulse oximetry, 96% on room air. An audible murmur is heard upon auscultation. The client is awake but somewhat lethargic and cannot respond to questions. The client is exhibiting neurologic deficits and impaired mobility of the left side of the body. The client is being evaluated for tissue plasminogen activator (t-PA) therapy. Drag the assessment paramenters that indicate whether the t-PA can be used to the box on the right. Click an option, hold and drag it to the desired position, or click an option to highlight it and move it right or left using the arrows in the middle. INR 2.0 Heart murmur Blood pressure reading Stroke scale of 17 Computed tomography (CT) confirmation of ischemic stroke

Blood pressure reading Stroke scale of 17 Computed tomography (CT) confirmation of ischemic stroke Explanation: Tissue plasminogen activator (t-PA) can be administered to clients with a blood pressure range of equal or below 185 mm Hg systolic and equal or below 110 mm Hg diastolic, a confirmed imaging study that identifies an ischemic stroke, and validation of the National Institutes of Health Stroke Scale (NIHSS) scale. An international normalized ratio (INR) reading of 2.0 indicates hypercoagulation; per protocol, the INR should be equal to or less than 1.7. Administration of t-PA to a client with an increased INR can lead to further bleeding. Client with a heart murmur in conjunction with an increased temperature, this could suggest the presence of infective endocarditis, which is a contraindication for t-PA therapy.

The provider diagnoses the patient as having had an ischemic stroke. The etiology of an ischemic stroke would include which of the following? Arteriovenous malformation Intracerebral hemorrhage Cardiogenic emboli Cerebral aneurysm

Correct response: Cardiogenic emboli Explanation: Aneurysms, hemorrhages, and malformations are all examples of a hemorrhagic stroke. An embolism can block blood flow, leading to ischemia.

A patient has been diagnosed with a lipoma. The nurse explains to the patient that this tumor is located in the part of the brain known as the: Brainstem. Cerebrum. Optic chiasm. Corpus callosum.

Correct response: Corpus callosum. Explanation: The corpus callosum is a thick collection of nerve fibers that connect both hemispheres of the brain and is responsible for transmitting information from one side of the brain to another. A lipoma only occurs in this area. Chapter 65: p. 2115.

The nurse is completing an assessment on a client with myasthenia gravis. Which of the following historical recounting provides the most significant evidence regarding when the disorder began? Muscle spasms Sensitivity to bright light Shortness of breath Drooping eyelids

Correct response: Drooping eyelids Explanation: Ptosis (eyelid drooping) is the most common manifestation of myasthenia gravis. Muscle weakness varies depending on the muscles affected. SOB and respiratory distress occurs later as the disease progresses. Muscle spasms are more likely in multiple sclerosis. Photophobia is not significant in myasthenia gravis

Which activity should be avoided in clients with increased intracranial pressure (ICP)? Suctioning Position changes Enemas Minimal environmental stimuli

Correct response: Enemas Explanation: Enemas should be avoided in clients with increased ICP. The Valsalva maneuver causes increased ICP. Suctioning should not last longer than 15 seconds. Environmental stimuli should be minimal. If monitoring reveals that turning the client increases the ICP, rotating beds, turning sheets, and holding the client's head during turning may minimize the stimuli that cause increased ICP.

A nurse is caring for a client who requires intracranial pressure (ICP) monitoring. The nurse should be alert for what complication of ICP monitoring? Apnea Coma High blood pressure Infection

Correct response: Infection Explanation: The catheter for measuring ICP is inserted through a burr hole into a lateral ventricle of the cerebrum, thereby creating a risk of infection. Coma, high blood pressure, and apnea are late signs of increased ICP, not complications.

A nurse is preparing a teaching plan for a client diagnosed with amyotrophic lateral sclerosis (ALS) and his family about the disorder and changes that may occur. Which of the following would the nurse least likely include in the discussion? Spasticity Loss of bladder control Speech difficulties Difficulty swallowing

Correct response: Loss of bladder control Explanation: With amyotrophic lateral sclerosis, the client typically develops spasticity, difficulty swallowing, and speech difficulties. Usually the function of the anal and bladder sphincters remain intact because the spinal nerves that control these muscles are not affected.

The nurse explains to the client with projectile vomiting and severe headache that a medication is being prescribed to reduced edema surrounding the brain and lessen these symptoms. What medication is the nurse preparing to administer? Bevacizumab Mannitol Temozolomide Everolimus

Correct response: Mannitol Explanation: Mannitol is an osmotic diuretic that is administered to decrease the fluid content of the brain, which leads to a decrease in intracranial pressure. Temozolomide is a chemotherapeutic agent which is commonly used to stop or slow cell growth in certain types of brain tumors. Bevacizumab and everolimus are immunotherapy agents that reduce the vascularization of tumors, thereby inhibiting tumor growth.

A female client who reports recurring headaches, accompanied by increased irritability, photophobia, and fatigue is asked to track the headache symptoms and occurrence on a calendar log. Which is the best nursing rationale for this action? Headaches are the most common type of reported pain. Cluster headaches can cause severe debilitating pain. Tension headaches are easier to treat. Migraines often coincide with menstrual cycle.

Correct response: Migraines often coincide with menstrual cycle. Explanation: Changes in reproductive hormones as found during menstrual cycle can be a trigger for migraine headaches and may assist in the management of the symptoms. Cluster headaches can cause severe pain but are not the reason for tracking. Tension headaches can be managed but is not associated with a monthly calendar. Headaches are common but not the reason for tracking.

The nurse is performing an initial assessment on a client who is admitted to rule out myasthenia gravis. Which of the following findings would the nurse expect to observe? Ptosis and diplopia Difficulty with urination Facial distortion and pain Muscle weakness and hyporeflexia of the lower extremities

Correct response: Ptosis and diplopia Explanation: The initial manifestation of myasthenia gravis in two-thirds of clients involves the ocular muscles; diplopia and ptosis are common. Muscle weakness and hyporeflexia of the lower extremities are associated with Guillain-Barre syndrome. Facial distortion and pain are associated with Bell's palsy and tic douloureux.

A nurse completes the Glasgow Coma Scale on a patient with traumatic brain injury (TBI). Her assessment results in a score of 6, which is interpreted as: Mild TBI. Severe TBI. Brain death. Moderate TBI.

Correct response: Severe TBI. Explanation: A score of 13 to 15 is classified as mild TBI, 9 to 12 is moderate TBI, and 3 to 8 is severe TBI. A score of 3 indicates severe impairment of neurologic function, deep coma, brain death, or pharmacologic inhibition of the neurologic response; a score of 8 or less typically indicates an unconscious patient; a score of 15 indicates a fully alert and oriented patient.

While providing information to a community group, the nurse tells them the primary initial symptoms of a hemorrhagic stroke are: Confusion or change in mental status Weakness on one side of the body and difficulty with speech Severe headache and early change in level of consciousness Foot drop and external hip rotation

Correct response: Severe headache and early change in level of consciousness Explanation: The main presenting symptoms for ischemic stroke are numbness or weakness of the face, arm, or leg, especially on one side of the body, confusion or change in mental status, and trouble speaking or understanding speech. Severe headache, vomiting, early change in level of consciousness, and seizures are early signs of a hemorrhagic stroke. Foot drop and external hip rotation can occur if a stroke victim is not turned or positioned correctly.

Which term refers to muscular hypertonicity in a weak muscle, with increased resistance to stretch? Myoclonus Akathisia Ataxia Spasticity

Correct response: Spasticity Explanation: Spasticity is often associated with weakness, increased deep tendon reflexes, and diminished superficial reflexes. Akathisia refers to restlessness, an urgent need to move around, and agitation. Ataxia refers to impaired ability to coordinate movement. Myoclonus refers to spasm of a single muscle or group of muscles.

The nurse assists the health care provider (HCP) in completing a lumbar puncture (LP). Which should the nurse note as a concern? The cerebrospinal fluid (CSF) is cloudy in nature. The pressure is noted to be 90 mm H20. The HCP maintains aseptic procedure. The HCP administers a drug by intrathecal injection.

Correct response: The cerebrospinal fluid (CSF) is cloudy in nature. Explanation: The CSF is normally clear and colorless; therefore, CSF that is cloudy would be noted by the nurse as a concern. The HCP is correct to maintain aseptic procedure. At 90 mm H20, the client's CSF fluid pressure falls within normal limits (between 80 and 100 mm H20). Sometimes the HCP will administer medication via intrathecal injection during an LP, which should not be a cause for concern. Chapter 60: p. 1990.

The nurse is caring for a client with Bell's palsy. Which of the following teaching points is a priority in the management of symptoms for this client? Encourage semiannual dental exams. Avoid stimuli that trigger pain. Use ophthalmic lubricant and protect the eye. Complete the course of antibiotics as prescribed.

Correct response: Use ophthalmic lubricant and protect the eye. Explanation: The VII cranial nerve supplies muscles to the face. In Bell's palsy, the eye can be affected which results in incomplete closure and risk for injury. The eye can become dry and irritated unless eye moisturizing drops and ophthalmic ointment is applied. Avoiding stimuli that can trigger pain is specific to tic douloureux (cranial nerve V disorder). Encouraging dental exams is a part of care but not the priority. Antibiotics are not used in the treatment of Bell's palsy because it is thought to be caused by a virus.

A patient had a small pituitary adenoma removed by the transsphenoidal approach and has developed diabetes insipidus. What pharmacologic therapy will the nurse be administering to this patient to control symptoms? Mannitol Vasopressin Furosemide (Lasix) Phenobarbital

Correct response: Vasopressin Explanation: Manipulation of the posterior pituitary gland during surgery may produce transient diabetes insipidus of several days' duration. It is treated with vasopressin but occasionally persists.

The nurse educator is providing orientation to a new group of staff nurses on an oncology unit. Part of the orientation is to help nurses understand the differences between various types of brain tumors. The nurse educator correctly identifies that glioma tumors are classified based on the fact that they originate where in the brain? In the cranial nerves Within the brain tissue From metastasis of a primary tumor From the coverings of the brain

Correct response: Within the brain tissue Explanation: Gliomas tumors are a type of intracerebral brain neoplasm. They originate within brain tissue. Tumors arising from the coverings of the brain include meningiomas. These tumors grow on the membrane covering of the brain, called the meninges. An acoustic neuroma is an example of tumors that grow out of or on cranial nerves and cause compression leading to sensory deficits. Metastasis refers to spreading of any kind of malignant primary tumor. This term is not specific to any one classification of tumor.

A nurse is monitoring a client for increasing intracranial pressure (ICP). Early signs of increased ICP include: elevated temperature. diminished responsiveness. decreasing blood pressure. pupillary changes.

Correct response: diminished responsiveness. Explanation: Usually, diminished responsiveness is the first sign of increasing ICP. Pupillary changes occur later. Increased ICP causes systolic blood pressure to rise. Temperature changes vary and may not occur even with a severe decrease in responsiveness. Chapter 61:p. 2009.

The nurse is caring for a client newly diagnosed with multiple sclerosis who is overwhelmed by learning about the disease. The client indicates understanding that there is a disruption in the covering of axons but does not remember what the covering is called. The nurse should tell the client: not to worry about the finer details of the disease. that the covering is called myelin and that it can be discussed further at the next meeting. that the disease process requires more research. that because there is so much to learn, there will be another meeting to discuss it again.

Correct response: that the covering is called myelin and that it can be discussed further at the next meeting. Explanation: Myelin is a fatty substance that covers some axons in the CNS and PNS. The nurse would be most correct in answering the question and then, if the client is tired, following up at the next meeting. It would also be appropriate to provide literature for the client to review at leisure. Discounting the client's need to know information about the disease process is belittling. Telling the client that more research needs to be done discounts the valuable information which is known.

The nurse is performing range-of-motion exercises. Which of the following best depicts dorsiflexion of the foot?

Explanation: With dorsiflexion (Option B), the nurse moves the foot up and down toward the leg. Flexion of the hip and knee (Option A) involves bending the hip by moving the leg forward as afar as possible and then returning it to the neutral position. Inversion and eversion (Option C) involves moving the foot so that the sole is facing outward (eversion) and then so that the sole is facing inward (inversion). Flexion of toes (Option D) involves bending the toes toward the ball of the foot.

The nurse is educating a group of people newly diagnosed with migraine headaches. What information should the nurse include in the educational session? Select all that apply. Maintain a headache diary. Sleep no more than 5 hours at a time. Use St. John's Wort. Exercise in a dark room. Keep a food diary.

Keep a food diary. Maintain a headache diary. Explanation: The clients should be encouraged to keep food and headache diaries to identify triggers and to track frequency and characteristics of the migraines. The clients should maintain a routine sleep pattern and avoid fatigue. Symptoms of a migraine are nausea, vomiting, and photophobia. Being in a dark room may ease the photophobia, but exercise may worsen the headache and associated symptoms. Clients who are taking medications specific for migraines should avoid St. John's Wort due to potential drug interactions.

The nurse is evaluating whether a client's walker is the right height for the client. While the client's hands are on the hand grip, the nurse assesses the client's elbows. The nurse determines that the walker is at the right height when the client's elbows are in which position? 45-degree flexion 25-degree flexion 15-degree flexion 0-degree flexion

Correct response: 25-degree flexion Explanation: When a walker is at the right height for a client, the client's elbows will be between 20 and 30 degrees flexion when the hands are resting on the hand grip.

The nurse is preparing to administer tissue plasminogen activator (t-PA) to a patient who weighs 132 lb. The order reads 0.9 mg/kg t-PA. The nurse understands that 10% of the calculated dose is administered as an IV bolus over 1 minute, and the remaining dose (90%) is administered IV over 1 hour via an infusion pump. How many milligrams IV bolus over 1 minute will the nurse initially administer?

Correct response: 5.4 Explanation: The patient is weighed to determine the dose of t-PA. Typically two or more IV sites are established prior to administration of t-PA. The dosage for t-PA is 0.9 mg/kg, with a maximum dose of 90 mg. Of the calculated dose, 10% is administered as an IV bolus over 1 minute. The remaining dose (90%) is administered IV over 1 hour via an infusion pump. First, the nurse must convert the patient's weight to kilograms (132/2.2 = 60 kg) then multiply 0.9 mg × 60 kg = 54 mg. Next, the nurse figure out that 10% of 54 mg is 5.4 (54 ×.10). The nurse will initially administer 5.4 mgs IV bolus over 1 minute.

A client with a head injury is being monitored for increased intracranial pressure (ICP). His blood pressure is 90/60 mm Hg and the ICP is 18 mm Hg; therefore his cerebral perfusion pressure (CPP) is: 52 mm Hg. 48 mm Hg. 88 mm Hg. 68 mm Hg.

Correct response: 52 mm Hg. Explanation: To determine CPP, subtract the ICP from the mean arterial pressure (MAP). The MAP is derived using the following formula using the diastolic pressure (DP) and systolic pressure (SP): MAP = DP + 1/3(SP - DP) In this case MAP = 60 mm Hg + 1/3(90 mm Hg - 60 mm Hg) = 70 mm Hg CPP = MAP - ICP CPP = 70 mm Hg - 18 mm Hg = 52 mm Hg

A 154-pound woman has been prescribed tPA (0.9 mg/kg) for an ischemic stroke. The nurse knows to give how many mg initially? 6.3 mg 7.5 mg 8.3 mg 10 mg

Correct response: 6.3 mg Explanation: A person who weighs 154 lbs weighs 70 kg. To calculate total dosage, multiply 70 × 0.9 mg/kg = 63 mg. Ten percent of the calculated dose is given as an IV bolus over 1 minute. The remaining dose (90%) is given IV over 1 hour via an infusion pump. So initially the nurse gives 10% (6.3 mg) over 1 minute. Chapter 62: Page 2037

A client with cerebral metastasis suddenly experiences a seizure for which phenytoin 10 mg/kg intravenously is ordered as an initial loading dose. The client weighs 132 pounds. How many milligrams of phenytoin should the client receive? Enter the number ONLY.

Correct response: 600 Explanation: First, change the client's weight in pounds to kilograms by dividing the weight by 2.2 (2.2 pounds = 1 kg). The client's weight is 60 kg. Next, set up a proportion: 10/1 = x/60; cross multiply and solve for x, which is 600.

A nurse knows that a patient exhibiting seizure-like movements localized to one side of the body most likely has what type of tumor? A motor cortex tumor An occipital lobe tumor A frontal lobe tumor A cerebellar tumor

Correct response: A motor cortex tumor Explanation: A tumor in the motor cortex of the frontal lobe produces hemiparesis and partial seizures on the opposite side of the body or generalized seizures. Frontal lobe tumor may also produce changes in emotional state and behavior, as well as an apathetic mental attitude. Cerebellar tumor causes dizziness; an ataxic or staggering gait with a tendency to fall toward the side of the lesion; marked muscle incoordination; and nystagmus (involuntary rhythmic eye movements), usually in the horizontal direction. Occipital lobe tumor produces visual manifestations: contralateral homonymous hemianopsia (visual loss in half of the visual field on the opposite side of the tumor) and visual hallucinations.

A client with quadriplegia is in spinal shock. What finding should the nurse expect? Spasticity of all four extremities Positive Babinski's reflex along with spastic extremities Absence of reflexes along with flaccid extremities Hyperreflexia along with spastic extremities

Correct response: Absence of reflexes along with flaccid extremities Explanation: During the period immediately following a spinal cord injury, spinal shock occurs. In spinal shock, all reflexes are absent and the extremities are flaccid. When spinal shock subsides, the client will demonstrate positive Babinski's reflex, hyperreflexia, and spasticity of all four extremities.

The nurse is caring for a client hospitalized with a severe exacerbation of myasthenia gravis. When administering medications to this client, what is a priority nursing action? Give client plenty of fluids with medications. Assess client's reaction to new medication schedule. Document medication given and dose. Administer medications at exact intervals ordered.

Correct response: Administer medications at exact intervals ordered. Explanation: The nurse must administer medications at the exact intervals ordered to maintain therapeutic blood levels and prevent symptoms from returning. Assessing the client's reaction, documenting medication and dose, and giving the client plenty of fluids are not the priority nursing action for this client.

Which nursing intervention is appropriate for a client with double vision in the right eye due to MS? Administer eye drops as needed. Apply an eye patch to the right eye. Place needed items on the right side. Exercise the right eye twice a day.

Correct response: Apply an eye patch to the right eye. Explanation: An eye patch to the affected eye would help the client with double vision see more clearly, thus promoting safety. Exercises for the eye would not benefit the client. Eye drops may be needed for dryness to prevent corneal abrasion but would not have any benefit for a client with double vision. Needed items should be placed on the unaffected (left) side.

Which term refers to the inability to coordinate muscle movements, resulting in difficulty walking? Agnosia Ataxia Spasticity Rigidity

Correct response: Ataxia Explanation: Ataxia is the inability to coordinate voluntary muscle action; tremors (rhythmic, involuntary movements) noted at rest or during movement suggest a problem in the anatomic areas responsible for balance and coordination. Agnosia is the loss of ability to recognize objects through a particular sensory system. Spasticity is the sustained increase in tension of a muscle when it is passively lengthened or stretched.

Which of the following, if left untreated, can lead to an ischemic stroke? Atrial fibrillation Ruptured cerebral arteries Cerebral aneurysm Arteriovenous malformation (AVM)

Correct response: Atrial fibrillation Explanation: Atrial fibrillation is the most frequently diagnosed arrhythmia in the United States. If left untreated, it can lead to an ischemic stroke. Cerebral hemorrhage, ruptured cerebral arteries, Cerebral aneurysm, and arteriovenous malformation can lead to a hemorrhagic stroke.

Lesions in the temporal lobe may result in which type of agnosia? Tactile Visual Auditory Relationship

Correct response: Auditory Explanation: Lesions in the temporal lobe (lateral and superior portions) may result in auditory agnosia. Lesions in the occipital lobe may result in visual agnosia. Lesions in the parietal lobe may result in tactile agnosia. Lesions in the parietal lobe (posteroinferior regions) may result in relationship and body part agnosia.

A client has experienced an ischemic stroke that has damaged the temporal (lateral and superior portions) lobe. Which of the following deficits would the nurse expect during assessment of this client? Lack of deep tendon reflexes Limited attention span and forgetfulness Hemiplegia or hemiparesis Auditory agnosia

Correct response: Auditory agnosia Explanation: Damage to the occipital lobe can result in visual agnosia damage to the temporal lobe can cause auditory agnosia. damage to the frontal lobe, learning capacity, memory, or other higher cortical intellectual functions may be impaired. Such dysfunction may be reflected in a limited attention span, difficulties in comprehension, forgetfulness, and lack of motivation. Damage to motor neurons may cause hemiparesis, hemiplegia, and a change in reflexes.

A patient is admitted to the emergency room with a fractured skull sustained in a motorcycle accident. The nurse notes fluid leaking from the patient's ears. The nurse knows this is a probable sign of which type of skull fracture? Depressed Simple Basilar Comminuted

Correct response: Basilar Explanation: Basilar skull fractures are suspected when cerebrospinal fluid (CSF) escapes from the ears (CSF otorrhea) and/or the nose (CSF rhinorrhea).

A nurse is caring for a client diagnosed with a cerebral aneurysm who reports a severe headache. Which action should the nurse perform? Inform the nurse manager. Sit with the client for a few minutes. Administer an analgesic. Call the physician immediately.

Correct response: Call the physician immediately. Explanation: The nurse should notify the physician immediately because the headache may be an indication that the aneurysm is leaking. Sitting with the client is appropriate but only after the physician has been notified of the change in the client's condition. The physician will decide whether or not administration of an analgesic is indicated. Informing the nurse manager isn't necessary.

The nurse responds to the call light of a client who has had a cervical discectomy earlier in the day. The client states that she is having severe pain that had a sudden onset. What is the nurse's most appropriate action? Palpate the surgical site. Remove the dressing to assess the surgical site. Administer a dose of an NSAID. Call the surgeon to report the client's pain.

Correct response: Call the surgeon to report the client's pain. Explanation: If the client experiences a sudden increase in pain, extrusion of the graft may have occurred, requiring reoperation. A sudden increase in pain should be promptly reported to the surgeon. Administration of an NSAID would be an insufficient response. The dressing should not be removed without an order. Palpation could cause further damage.

A client is waiting in a triage area to learn the medical status of family members following a motor vehicle accident. The client is pacing, taking deep breaths, and handwringing. Considering the effects in the body systems, what does the nurse anticipate the liver will do? Maintain a basal rate of functioning. Cease function and shunt blood to the heart and lungs. Convert glycogen to glucose for immediate use. Produce a toxic byproduct in relation to stress.

Correct response: Convert glycogen to glucose for immediate use. Explanation: When the body is under stress, the sympathetic nervous system is activated readying the body for action. The effect of the body is to mobilize stored glycogen to glucose to provide additional energy for body action.

Low levels of the neurotransmitter serotonin lead to which of the following disease processes? Seizures Myasthenia gravis Depression Parkinson's disease

Correct response: Depression Explanation: A decrease of serotonin leads to depression. A decrease in the amount of acetylcholine causes myasthenia gravis Parkinson's disease is caused by a depletion of dopamine. Decreased levels of GABA may cause seizures.

The nurse is evaluating the transmission of a report from a paramedic unit to the emergency department. The medic reports that a client is unconscious with edema of the head and face and Battle sign. What clinical picture would the nurse anticipate? Edema to the head with fixed pupils Edema to the head with bruising of the mastoid process Edema to the head and a blackened eye Edema to the head with a large scalp laceration

Correct response: Edema to the head with bruising of the mastoid process Explanation: Battle sign is the presence of bruising of the mastoid process behind the ear. It is not related to periorbital bleeding, lacerations, or fixed pupils. Chapter 63: p. 2057

Which type of hematoma results from a skull fracture that causes a rupture or laceration of the middle meningeal artery? Diffuse axonal Subdural Epidural Intracerebral

Correct response: Epidural Explanation: An epidural hematoma can result from a skull fracture that causes a rupture or laceration of the middle meningeal artery. Subdural hematoma is a collection of blood between the dura and the brain. Intracerebral hemorrhage is bleeding into the substance of the brain. Diffuse axonal injury involves widespread damage to axons in the cerebral hemispheres, corpus callosum, and brainstem.

The nurse is assessing a client with meningitis. Which of the following signs would the nurse expect to observe? Hyporeflexia in the lower extremities Ptosis and diplopia Numbness and vomiting Headache and nuchal rigidity

Correct response: Headache and nuchal rigidity Explanation: Headache and fever are the initial symptoms of meningitis. Nuchal rigidity can be an early sign. Photophobia is also a well-recognized sign in meningitis. Ptosis and diplopia are usually seen with myasthenia gravis. Hyporeflexia in the legs is seen with Guillain-Barre syndrome.

A client undergoes cerebral angiography for evaluation of a subarachnoid hemorrhage. Which findings indicate spasm or occlusion of a cerebral vessel by a clot? Hemiplegia, seizures, and decreased level of consciousness Nausea, vomiting, and profuse sweating Difficulty breathing or swallowing Tachycardia, tachypnea, and hypotension

Correct response: Hemiplegia, seizures, and decreased level of consciousness Explanation: Spasm or occlusion of a cerebral vessel by a clot causes signs and symptoms similar to those of a stroke: hemiplegia, seizures, decreased level of consciousness, aphasia, hemiparesis, and increased focal symptoms. Nausea, vomiting, and profuse sweating suggest a delayed reaction to the contrast medium used in cerebral angiography. Difficulty breathing or swallowing may signal a hematoma in the neck. Tachycardia, tachypnea, and hypotension suggest internal hemorrhage. Chapter 62: p. 2052.

The nurse is caring for a patient postoperatively after intracranial surgery for the treatment of a subdural hematoma. The nurse observes an increase in the patient's blood pressure from the baseline and a decrease in the heart rate from 86 to 54. The patient has crackles in the bases of the lungs. What does the nurse suspect is occurring? Infection Increased ICP Increase in cerebral perfusion pressure Exacerbation of uncontrolled hypertension

Correct response: Increased ICP Explanation: Increased ICP and bleeding are life threatening to the patient who has undergone intracranial surgery. An increase in blood pressure and decrease in pulse with respiratory failure may indicate increased ICP.

A patient comes to the emergency department with a large scalp laceration after being struck in the head with a glass bottle. After assessment of the patient, what does the nurse do before the physician sutures the wound? Administers an oral analgesic for pain Administers acetaminophen (Tylenol) for headache Shaves the hair around the wound Irrigates the wound to remove debris

Correct response: Irrigates the wound to remove debris Explanation: Scalp wounds are potential portals of entry for organisms that cause intracranial infections. Therefore, the area is irrigated before the laceration is sutured to remove foreign material and to reduce the risk for infection.

The nurse working on a neurological unit is mentoring a nursing student who asks about a client who has sustained primary and secondary brain injuries. The nurse correctly tells the student which of the following, related to the secondary injury? It refers to the difficulties suffered by the client and family related to the changes in the client. It results from initial damage to the brain from the traumatic event. It refers to the permanent deficits seen after the rehabilitation process. It results from inadequate delivery of nutrients and oxygen to the cells.

Correct response: It results from inadequate delivery of nutrients and oxygen to the cells. Explanation: Secondary injury results from inadequate delivery of nutrients and oxygen to the cells, usually as a result of cerebral edema and increased intracranial pressure. Primary injury results from initial damage related to the traumatic event.

A 53-year-old man presents to the emergency department with a chief complaint of inability to form words, and numbness and weakness of the right arm and leg. Where would you locate the site of injury? Left temporal region Right frontoparietal region Left frontoparietal region Left basal ganglia

Correct response: Left frontoparietal region Explanation: The patient is exhibiting signs of expressive aphasia with numbness/tingling and weakness of the right arm and leg. This indicates injury to the expressive speech center (Broca's area), which is located in the inferior portion of the frontal lobe. The motor strip is located in the posterior portion of the frontal lobe. The sensory strip is located in the anterior parietal lobe.

Which of the following is accurate regarding a hemorrhagic stroke? One of the main presenting symptoms is numbness or weakness of the face. Main presenting symptom is an "exploding headache." It is caused by a large-artery thrombosis. Functional recovery usually plateaus at 6 months.

Correct response: Main presenting symptom is an "exploding headache." Explanation: One of hemorrhagic stroke's main presenting symptom is an "exploding headache." In ischemic stroke, functional recovery usually plateaus at 6 months; it may be caused by a large artery thrombosis and may have a presenting symptoms of numbness or weakness of the face. Chapter 62: Page 2049

The nurse is caring for a client with a traumatic brain injury who has developed increased intracranial pressure resulting in syndrome of inappropriate antidiuretic hormone (SIADH). While assessing this client, the nurse expects which of the following findings? Oliguria and serum hyperosmolarity Excessive urine output and decreased urine osmolality Excessive urine output and serum hyponatremia Oliguria and serum hyponatremia

Correct response: Oliguria and serum hyponatremia Explanation: SIADH is the result of increased secretion of antidiuretic hormone (ADH). The client becomes volume overloaded, urine output diminishes, and serum sodium concentration becomes dilute.

Which of the following is a disorder due to a lesion in the basal ganglia? Multiple sclerosis Myasthenia gravis Guillain-Barré Parkinson's disease

Correct response: Parkinson's disease Explanation: Disorders due to lesions of the basal ganglia include Parkinson's disease, Huntington's disease, and spasmodic torticollis.

A client with neurological infection develops cerebral edema from syndrome of inappropriate antidiuretic hormone (SIADH). Which is an important nursing action for this client? Maintaining adequate hydration Hyperoxygenation before and after tracheal suctioning Restricting fluid intake and hydration Administering prescribed antipyretics

Correct response: Restricting fluid intake and hydration Explanation: Fluid restriction may be necessary if the client develops cerebral edema and hypervolemia from SIADH. Antipyretics are administered to clients who develop hyperthermia. In addition, it is important to maintain adequate hydration in such clients. A client with neurological infection should be given tracheal suctioning and hyperoxygenation only when respiratory distress develops. Chapter 61: Page 2001/2014

Which neurotransmitter demonstrates inhibitory action, helps control mood and sleep, and inhibits pain pathways? Enkephalin Acetylcholine Norepinephrine Serotonin

Correct response: Serotonin Explanation: The brain stem, hypothalamus, and dorsal horn of the spinal cord are sources of serotonin. Enkephalin is excitatory and associated with pleasurable sensations. Norepinephrine is usually excitatory and affects mood and overall activity. Acetylcholine is usually excitatory, but the parasympathetic effects are sometimes inhibitory. p. 1967.

The nurse is caring for a client with chronic migraines who is prescribed medication. What drug-related instructions should the nurse give the client? Take medication as soon as symptoms of the migraine begin. Take medication only when migraine is intense. Take medication just before going to bed at night. Take medication only during the morning when it's calm and quiet.

Correct response: Take medication as soon as symptoms of the migraine begin. Explanation: The nurse reinforces the drug therapy regimen and instructs the client on self-administration of medications. To stop the migraine headache, the nurse stresses the importance of taking medication as soon as symptoms of the migraine begin and not when the migraine intensifies.

During the recovery phase of a neurologic deficit, assessment tools may be used to help identify a client's level of functioning. Which tool is used to measure performance in activities of daily living (ADL)? The Barthel Index The National Institute for Health Stroke Scale Hamilton Assessment Scale The American Heart Association's Stroke Outcome Classification

Correct response: The Barthel Index Explanation: On The Barthel Index, each performance ADL item is rated with a given number of points assigned to each level or ranking. A higher number is associated with a greater likelihood of being able to live at home with a degree of independence following discharge from hospital.

A client has just returned from surgery after undergoing a lumbar laminectomy. Which of the following would be most important to do when positioning the client in bed? Keeping the knees flat with the head on a pillow Allowing the client to sit up at the edge of the bed Using a logrolling motion to change positions Maintaining full knee flexion when lying on the side

Correct response: Using a logrolling motion to change positions Explanation: After a laminectomy, logrolling is used to change the client's position. When in bed, a pillow is placed under the client's head and the knee rest is elevated slightly to relax the back muscles. When lying on his or her side, extreme knee flexion is avoided. Sitting is discouraged except for defecation. Chapter 65: p. 2122.

A client diagnosed with a stroke is ordered to receive warfarin. Later, the nurse learns that the warfarin is contraindicated and the order is canceled. The nurse knows that the best alternative medication to give is dipyridamole. ticlopidine. clopidogrel. aspirin.

Correct response: aspirin. Explanation: If warfarin is contraindicated, aspirin is the best option, although other medications may be used if both are contraindicated.

A gymnast sustained a head injury after falling off the balance beam at practice. The client was taken to surgery to repair an epidural hematoma. In postoperative assessments, the nurse measures the client's temperature every 15 minutes. This measurement is important to: assess for infection. follow hospital protocol. prevent embolism. decrease the potential for brain damage.

Correct response: decrease the potential for brain damage. Explanation: It is important to monitor the client's body temperature closely; hyperthermia increases brain metabolism, increasing the potential for brain damage. Therefore, elevated temperature must be relieved with an antipyretic and other measures.

A client who was found unconscious at home is brought to the hospital by a rescue squad. In the intensive care unit, the nurse checks the client's oculocephalic (doll's eye) response by: shining a bright light into the pupil. introducing ice water into the external auditory canal. turning the client's head suddenly while holding the eyelids open. touching the cornea with a wisp of cotton.

Correct response: turning the client's head suddenly while holding the eyelids open. Explanation: To elicit the oculocephalic response, which detects cranial nerve compression, the nurse turns the client's head suddenly while holding the eyelids open. Normally, the eyes move from side to side when the head is turned; in an abnormal response, the eyes remain fixed. The nurse introduces ice water into the external auditory canal when testing the oculovestibular response; normally, the client's eyes deviate to the side of ice water introduction. The nurse touches the client's cornea with a wisp of cotton to elicit the corneal reflex response, which reveals brain stem function; blinking is the normal response. Shining a bright light into the client's pupil helps evaluate brain stem and cranial nerve III function; normally, the client's pupil responds by constricting.

The nurse is completing an assessment on a client with a history of migraines. The nurse would identify which of the following factors as a possible trigger for a migraine headache? Select all that apply. Exposure to flashing light Red wine Change in environmental temperature Menstruation Nausea Prolonged positioning

Red wine Menstruation Exposure to flashing light Explanation: Research on the cause of migraines is ongoing; however, changes in reproductive hormones (menstruation), exposure to flashing light, and particular food/beverages and alcohol can be a trigger for some clients. Nausea is a symptom of a migraine. Exposure to changes in environmental temperature does not trigger a migraine headache. Prolonged positioning can cause muscle fatigue and strain that trigger tension headaches.

A nurse is caring for a client with a cerebral aneurysm. Which nursing interventions would be most useful to the nurse to avoid bleeding in the brain? Select all that apply. Maintain the head of the bed at 30 degrees. Use a well-lighted room for assessments every 2 hours. Avoid any activities that cause a Valsalva maneuver. Follow the healthcare provider's orders to increase fluid volume. Report changes in neurologic status as soon as a worsening trend is identified.

Report changes in neurologic status as soon as a worsening trend is identified. Maintain the head of the bed at 30 degrees. Avoid any activities that cause a Valsalva maneuver. Explanation: Cerebral aneurysm precautions are implemented for the patient with a diagnosis of aneurysm to provide a nonstimulating environment, prevent increases in intracranial pressure, and prevent further bleeding. The patient is placed on bed rest in a quiet, nonstressful environment, because activity, pain, and anxiety are thought to elevate the blood pressure, which may increase the risk for bleeding. The head of the bed is elevated 30 degrees to promote venous drainage and decrease intracranial pressure. Any activity that suddenly increases the blood pressure or obstructs venous return is avoided. This includes the Valsalva maneuver, straining, forceful sneezing, pushing oneself up in bed and acute flexion or rotation of the head and neck (which compromises the jugular veins). Stool softeners and mild laxatives are prescribed to prevent constipation, which can cause an increase in intracranial pressure. Dim lighting is helpful for photophobia. Increasing fluid volume does not affect brain bleeding.

A nurse is teaching a client who was recently diagnosed with myasthenia gravis. Which statement should the nurse include in her teaching? "You'll need to take edrophonium (Tensilon) to treat the disease." "The disease is a disorder of motor and sensory dysfunction." "You'll continue to experience progressive muscle weakness and sensory deficits." "This disease doesn't cause sensory impairment."

Correct response: "This disease doesn't cause sensory impairment." Explanation: Myasthenia gravis affects motor function; therefore, the nurse should inform the client that sensory impairments won't occur. This disease is chronic; there's no cure. It can be managed with edrophonium in the diagnostic phase; however, this drug isn't used to treat the condition.

A patient sustained a head trauma in a diving accident and has a cerebral hemorrhage located within the brain. What type of hematoma is this classified as? A subdural hematoma An extradural hematoma An intracerebral hematoma An epidural hematoma

Correct response: An intracerebral hematoma Explanation: Intracerebral hemorrhage (hematoma) is bleeding within the brain, into the parenchyma of the brain. It is commonly seen in head injuries when force is exerted to the head over a small area (e.g., missile injuries, bullet wounds, stab injuries). A subdural hematoma (SDH) is a collection of blood between the dura and the brain, a space normally occupied by a thin cushion of cerebrospinal fluid. After a head injury, blood may collect in the epidural (extradural) space between the skull and the dura.

Which of the following methods may be used by the nurse to maintain the peripheral circulation in a patient with increased intracerebral pressure (ICP)? Apply elastic stockings to lower extremities. Assist the patient with frequent ambulation. Take care not to jar the bed or cause unnecessary activity. Elevate patient's head or follow the physician's directive for body position.

Correct response: Apply elastic stockings to lower extremities. Explanation: To maintain the peripheral circulation in a patient with increased ICP, the nurse must apply elastic stockings to lower extremities. Elastic stockings support the valves of veins in the lower extremities to prevent venous stasis, and relieving pressure promotes the circulation of oxygenated blood through the capillary to peripheral cells and tissues and facilitates venous blood return. The patient's bed should not be jarred or shaken because unexpected physical movement tends to aggravate the pain and does not help in maintaining the peripheral circulation. On the other hand, head elevation helps venous blood and cerebrospinal fluid drain from cerebral areas.

The nurse is taking care of a client with a history of headaches. The nurse takes measures to reduce headaches and administer medications. Which appropriate nursing interventions may be provided by the nurse to such a client? Perform the Heimlich maneuver Use pressure-relieving pads or a similar type of mattress Apply warm or cool cloths to the forehead or back of the neck Maintain hydration by drinking eight glasses of fluid a day

Correct response: Apply warm or cool cloths to the forehead or back of the neck Explanation: Applying warm or cool cloths to the forehead or back of the neck and massaging the back relaxes muscles and provides warmth to promote vasodilation. These measures are aimed at reducing the occurrence of headaches in the client. A client with transient ischemic attacks is advised to maintain hydration and drink eight glasses of fluid a day. A Heimlich maneuver is performed to clear the airway if the client cannot speak or breathe after swallowing food. The nurse uses pressure-relieving pads or a similar type of mattress to maintain peripheral circulation in the client's body.

At a certain point, the brain's ability to autoregulate becomes ineffective and decompensation (ischemia and infarction) begins. Which of the following are associated with Cushing's triad? Select all that apply. Hypertension Bradypnea Tachycardia Hypotension Bradycardia

Correct response: Bradycardia Hypertension Bradypnea Explanation: The bradycardia, hypertension, and bradypnea associated with this deterioration are known as Cushing's triad, a grave sign. At this point, herniation of the brainstem and occlusion of the cerebral blood flow occur if therapeutic intervention is not initiated immediately.

Which occurs when reflexes are hyperactive when the foot is abruptly dorsiflexed? Rigidity Ataxia Clonus Flaccidity

Correct response: Clonus Explanation: Clonus occurs when the foot is abruptly dorsiflexed. It continues to "beat" two or three times before it settles into a position of rest. Sustained clonus always indicates the present of central nervous system disease and requires further evaluation. Ataxia is incoordination of voluntary muscle action. Rigidity is an increase in muscle tone at rest characterized by increased resistance to passive movement. Flaccid posturing is usually the result of lower brain stem dysfunction; the client has no motor function, is limp, and lacks motor tone.

The nurse reviews the patient's drug regimen for treatment of a brain tumor. She explains to the patient why one of the following drugs would not be prescribed, even though it might have therapeutic benefits. Which drug would not be prescribed for this patient? Decadron Dilantin Paclitaxel Coumadin

Correct response: Coumadin Explanation: Although deep vein thrombosis and pulmonary embolism occur in about 15% of patients and cause significant morbidity, anticoagulants are not prescribed due to the risk for CNS hemorrhage.

A patient suffering a stroke is having a difficult time swallowing. What would the nurse document this finding as? Dysphagia Dysarthria Ataxia Arthralgia

Correct response: Dysphagia Explanation: Stroke can result in dysphagia (difficulty swallowing) due to impaired function of the mouth, tongue, palate, larynx, pharynx, or upper esophagus. Patients must be observed for paroxysms of coughing, food dribbling out of or pooling in one side of the mouth, food retained for long periods in the mouth, or nasal regurgitation when swallowing liquids. Swallowing difficulties place the patient at risk for aspiration, pneumonia, dehydration, and malnutrition.

From which direction should a nurse approach a client who is blind in the right eye? From the left side of the client From directly behind the client From the right side of the client From directly in front of the client

Correct response: From the left side of the client Explanation: The nurse should approach the client from the left side so that the client can be aware of the nurse's approach. Likewise, personal items should be placed on the client's left side so that he can see them easily. Chapter 62: Page 2034

The nurse is assessing a client who was brought to the emergency department due to a severe headache with sudden onset, lowered level of consciousness and slurred, non-sensical speech. The client completed chemotherapy and radiation treatment for a glioma-type brain tumor 6 months ago. The client has been taking low molecular weight heparin since completing treatment. The nurse should be prepared to provide care for which possible problem? Intracerebral hemorrhage Spinal metastasis Pulmonary embolism Deep vein thrombosis

Correct response: Intracerebral hemorrhage Explanation: Clients receiving anticoagulant agents, such as low molecular weight heparin, must be closely monitored because of the risk of central nervous system hemorrhage, also known as an intercerebral hemorrhage. Both deep vein thrombosis and pulmonary embolism would be prevented or mitigated by the use of anticoagulant medications such as low molecular weight heparin. The nurse should always consider the risk of these latter problems, however, because the client is clearly at risk for impaired coagulation. Spinal metastasis can result in spinal cord compression, which is considered a medical emergency requiring immediate treatment. In this case, the nurse would observe reports of back pain, extremity weakness, ataxia and/or paralysis. Chapter 65: p. 2113.

The nurse is caring for a client admitted with a stroke. Imaging studies indicate an embolus partially obstructing the right carotid artery. What type of stroke does the nurse know this client has? Ischemic Hemorrhagic Right-sided Left-sided

Correct response: Ischemic Explanation: Ischemic strokes occur when a thrombus or embolus obstructs an artery carrying blood to the brain; about 80% of strokes are the ischemic variety. The other options are incorrect.

Which diagnostic is most commonly used for spinal cord compression? X-ray Positron emission tomography (PET) Magnetic resonance imaging (MRI) Computed tomography (CT)

Correct response: Magnetic resonance imaging (MRI) Explanation: MRI is the most commonly used diagnostic tool, detecting epidural spinal cord compression and metastases.

A client with a traumatic brain injury is showing early signs of increasing intracranial pressure (ICP). While planning care for this client, what would be the priority expected outcome? Demonstrates optimal cerebral tissue perfusion Attains desired fluid balance Maintains a patent airway Displays no signs or symptoms of infection

Correct response: Maintains a patent airway Explanation: Maintenance of a patent airway is always a first priority. Loss of airway is a possible complication of increasing ICP, as well as aspiration from vomiting.

Which is the primary vector of arthropod-borne viral encephalitis in North America? Birds Mosquitoes Spiders Ticks

Correct response: Mosquitoes Explanation: The primary vector in North America related to anthropoid-borne virus encephalitis is a mosquito. Birds are associated with the West Nile virus. Spiders and ticks are not vectors for arthropod-borne virus encephalitis. Chapter 64: p. 2093.

A patient 3 days postoperative from a craniotomy informs the nurse, "I feel something trickling down the back of my throat and I taste something salty." What priority intervention does the nurse initiate? Request an antihistamine for the postnasal drip. Ask the patient to cough to observe the sputum color and consistency. Notify the physician of a possible cerebrospinal fluid leak. Give the patient some mouthwash to gargle with.

Correct response: Notify the physician of a possible cerebrospinal fluid leak. Explanation: Any sudden discharge of fluid from a cranial incision is reported at once, because a large leak requires surgical repair. Attention should be paid to the patient who complains of a salty taste or "postnasal drip," because this can be caused by cerebrospinal fluid trickling down the throat.

Which is a modifiable risk factor for transient ischemic attacks and ischemic strokes? Advanced age Smoking Thyroid disease Social drinking

Correct response: Smoking Explanation: Modifiable risk factors for TIAs and ischemic stroke include hypertension, diabetes, cardiac disease, smoking, and excessive alcohol consumption. Advanced age, gender, and race are nonmodifiable risk factors for stroke.

A patient is diagnosed with a spinal cord tumor and has had a course of radiation and chemotherapy. Two months after the completion of the radiation, the patient complains of severe pain in the back. What is pain an indicator of in a patient with a spinal cord tumor? Lumbar sacral strain Hematoma formation Spinal metastasis The development of a skin ulcer from the radiation

Correct response: Spinal metastasis Explanation: Pain is the hallmark of spinal metastasis. Patients with sensory root involvement may suffer excruciating pain, which requires effective pain management.

A client is being taught to go down stairs using a cane. What action would the nurse instruct the patient to do first? Step down with the unaffected leg. Place the cane on the lower step. Step down with the affected leg. Place cane and affected leg on step simultaneously.

Correct response: Step down with the affected leg. Explanation: When using a cane to go down stairs, first the patient would step down with the affected leg, then place the cane, and then place the unaffected extremity on the down step. The affected leg and cane should not be used simultaneously.

A client diagnosed with Huntington's disease has developed severe depression. What would be most important for the nurse to assess for? Choreiform movements Emotional apathy Suicidal ideations Loss of bowel and bladder control

Correct response: Suicidal ideations Explanation: Severe depression is common and can lead to suicide, so it is most important for the nurse to assess for suicidal ideations. Symptoms of Huntington's disease develop slowly and include mental apathy and emotional disturbances, choreiform movements (uncontrollable writhing and twisting of the body), grimacing, difficulty chewing and swallowing, speech difficulty, intellectual decline, and loss of bowel and bladder control. Assessing for these symptoms is appropriate, but not as important as assessing for suicidal ideations. Chapter 65: p. 2130.

A client has been transferred to a rehabilitative setting from an acute care unit. What is the most important reason for the nurse to begin a program for activities of daily living (ADLs) as soon as the client is admitted to a rehabilitation facility? The ability to perform ADLs is essential to living in a group home. The ability to perform ADLs is necessary to function in an assisted-living situation. The ability to perform ADLs may be the key to re-entering the community. The ability to perform ADLs may be the key to dependence.

Correct response: The ability to perform ADLs may be the key to re-entering the community. Explanation: An ADL program is started as soon as the rehabilitation process begins because the ability to perform ADLs is frequently the key to independence, return to the home, and re-entry into the community. ADLs are frequently the key to independence, not dependence. The ability to perform ADLs is not always a criterion for admission to a group home or assisted-living facility.

The nurse is seeing the mother of a client who states, "I'm so relieved because my son's doctor told me his brain tumor is benign." The nurse knows what is true about benign brain tumors? They do not require surgical removal. They can affect vital functioning. The prognosis is very poor. They are all metastatic.

Correct response: They can affect vital functioning. Explanation: Benign tumors are usually slow growing but can occur in a vital area, where they can grow large enough to cause serious effects. Surgical removal of a benign tumor is dependent on many factors; even if the tumor is slow growing or not growing at all, the location of the tumor in the brain factors into the decision for surgical removal. The prognosis for all brain tumors is not necessarily poor. Treatment is individualized and can have varying prognostic outcomes. Benign tumors are not metastatic, meaning they do not grow rapidly or spread into surrounding tissue, but they can still be considered life-threatening.

A nurse is caring for a client who underwent a lumbar laminectomy 2 days ago. Which finding requires immediate intervention? Paresthesia in the dermatomes near the wounds Urine retention or incontinence Temperature of 99.2° F (37.3° C) More back pain than the first postoperative day

Correct response: Urine retention or incontinence Explanation: Urine retention or incontinence may indicate cauda equina syndrome, which requires immediate surgery. An increase in back pain is more common because on the second postoperative day the long-acting local anesthetic, which may have been injected during surgery, will wear off. Although paresthesia is common after surgery, progressive weakness or paralysis may indicate spinal nerve compression. A mild fever is also common after surgery but is considered significant only if the temperature reaches 101° F (38.3° C).

A client is being treated for a lumbar spinal injury that occurred 5 days ago and is currently experiencing the symptoms of spinal shock. Characteristic for this condition, the client is unable to move the lower extremities, is being closely monitored for hypotension and bradycardia, and has impaired temperature control. Which would not be an expected outcome of care? client's skin remains clean, dry, and intact client regains bowel elimination capacity client maintains mechanical ventilation with minimal mucus accumulation client reports no discomfort

Correct response: client maintains mechanical ventilation with minimal mucus accumulation Explanation: A client with a lumbar spinal injury would not require mechanical ventilation.

Bone density testing in clients with post-polio syndrome has demonstrated no significant findings. osteoarthritis. calcification of long bones. low bone mass and osteoporosis.

Correct response: low bone mass and osteoporosis. Explanation: Bone density testing in clients with post-polio syndrome has demonstrated low bone mass and osteoporosis. Thus, the importance of identifying risks, preventing falls, and treating osteoporosis must be discussed with clients and their families. p. 2139.

Which are characteristics of autonomic dysreflexia? severe hypertension, tachycardia, blurred vision, dry skin severe hypotension, slow heart rate, anxiety, dry skin severe hypertension, slow heart rate, pounding headache, sweating severe hypotension, tachycardia, nausea, flushed skin

Correct response: severe hypertension, slow heart rate, pounding headache, sweating Explanation: Autonomic dysreflexia is an exaggerated sympathetic nervous system response. Hypertension, tachycardia, bradycardia, and flushed skin would occur.

A patient is admitted via ambulance to the emergency room of a stroke center at 1:30 p.m. with symptoms that the patient said began at 1:00 p.m. Within 1 hour, an ischemic stroke had been confirmed and the doctor ordered tPA. The nurse knows to give this drug no later than what time? 2:00 p.m. 3:00 p.m. 4:00 p.m. 7:00 p.m.

Correct response: 4:00 p.m. Explanation: Tissue plasminogen activator (tPA) must be given within 3 hours after symptom onset. Therefore, since symptom onset was 1:00 pm, the window of opportunity ends at 4:00 pm.

Which of the following is the most common side effect of tissue plasminogen activator (tPA)? Bleeding Headache Hypertension Increased intracranial pressure (ICP)

Correct response: Bleeding Explanation: Bleeding is the most common side effect of tPA. The patient is closely monitored for bleeding (at IV insertion sites, gums, urine/stools, and intracranially by assessing changes in level of consciousness). Headache, increased ICP, and hypertension are not side effects of tPA

What part of the brain controls and coordinates muscle movement? Cerebellum Cerebrum Midbrain Brain stem

Correct response: Cerebellum Explanation: The cerebellum, which is located behind and below the cerebrum, controls and coordinates muscle movement.

Which type of brain injury has occurred if the client can be aroused with effort but soon slips back into unconsciousness? Contusion Concussion Diffuse axonal injury Intracranial hemorrhage

Correct response: Contusion Explanation: Contusions can be characterized by loss of consciousness associated with stupor and confusion. A concussion is a temporary loss of neurologic function with no apparent structural damage. A diffuse axonal injury involves widespread damage to the axons in the cerebral hemispheres, corpus callosum, and brainstem. An intracranial hemorrhage is a collection of blood that develops within the cranial vault.

When assessing a client's risk for pressure ulcer development, which finding would alert the nurse to an increased risk? Select all that apply. Edema Constipation Sensory overload Diaphoresis Anemia

Correct response: Edema Anemia Diaphoresis Explanation: Risk factors for pressure ulcer development include prolonged pressure on the tissue, sensory deficit or loss, edema, urinary or fecal incontinence, malnutrition, anemia, hypoproteinemia, and excessively moist skin

After a stroke, a client is admitted to the facility. The client has left-sided weakness and an absent gag reflex. He's incontinent and has a tarry stool. His blood pressure is 90/50 mm Hg, and his hemoglobin is 10 g. Which nursing intervention is a priority for this client? Keeping skin clean and dry Performing range-of-motion (ROM) exercises on the left side Elevating the head of the bed to 30 degrees Checking stools for occult blood

Correct response: Elevating the head of the bed to 30 degrees Explanation: Because the client's gag reflex is absent, elevating the head of the bed to 30 degrees helps minimize the client's risk of aspiration. Checking the stools, performing ROM exercises, and keeping the skin clean and dry are important, but preventing aspiration through positioning is the priority.

Which type of hematoma is evidenced by a momentary loss of consciousness at the time of injury, followed by an interval of apparent recovery (lucid interval)? Epidural Subdural Contusion Intracerebral

Correct response: Epidural Explanation: Symptoms of the epidural hematoma are caused by the expanding hematoma. Usually a momentary loss of consciousness occurs at the time of injury, followed by an interval of apparent recovery (lucid interval). Subdural hematoma is a collection of blood between the dura and the brain, a space normally occupied by a thin cushion of cerebrospinal fluid.

A nurse is caring for a client with a pituitary adenoma. Which laboratory test result suggests that a client has a corticotropin-secreting pituitary adenoma? High corticotropin and low cortisol levels Low corticotropin and low cortisol levels Low corticotropin and high cortisol levels High corticotropin and high cortisol levels

Correct response: High corticotropin and high cortisol levels Explanation: Pituitary adenomas secrete excess amounts of hormones, including adrenocortical-tropic hormone, resulting in Cushing syndrome, in which a corticotropin-secreting pituitary tumor causes high corticotropin and high cortisol levels. A high corticotropin level with a low cortisol level and a low corticotropin level with a low cortisol level are associated with hypocortisolism. A primary defect in the adrenal glands causes low corticotropin and high cortisol levels.

While assessing the client at the beginning of the shift, the nurse inspects a surgical dressing covering the operative site after the clients' cervical discectomy. The nurse notes that the drainage is 75% saturated with serosanguineous discharge. What is the nurse's most appropriate action? Page the health care provider and report this sign of infection. Reposition the client to prevent further hemorrhage. Inform the surgeon of the possibility of a dural leak. Reinforce the dressing and reassess in 1 to 2 hours.

Correct response: Inform the surgeon of the possibility of a dural leak. Explanation: After a cervical discectomy, the nurse will monitor the operative site and dressing covering this site. Serosanguineous drainage may indicate a dural leak. This constitutes a risk for meningitis, but is not a direct sign of infection. This should be reported to the surgeon, not just reinforced and observed.

A client with a brain tumor experiences projectile vomiting. The nurse integrates understanding of this occurrence as resulting from which of the following? Irritation of the medullary vagal centers Distortion of pain-sensitive structures Edema associated with the tumor Compression of surrounding structures

Correct response: Irritation of the medullary vagal centers Explanation: Vomiting associated with a brain tumor is usually the result of irritation of the vagal centers in the medulla. Edema secondary to the tumor or distortion of the pain-sensitive structures is thought to be the cause of the headache associated with brain tumors. Compression of the surrounding structures results in the signs and symptoms of increased intracranial pressure.

A client with a brain tumor experiences projectile vomiting. The nurse integrates understanding of this occurrence as resulting from which of the following? Distortion of pain-sensitive structures Irritation of the medullary vagal centers Compression of surrounding structures Edema associated with the tumor

Correct response: Irritation of the medullary vagal centers Explanation: Vomiting associated with a brain tumor is usually the result of irritation of the vagal centers in the medulla. Edema secondary to the tumor or distortion of the pain-sensitive structures is thought to be the cause of the headache associated with brain tumors. Compression of the surrounding structures results in the signs and symptoms of increased intracranial pressure

A patient who has suffered a stroke begins having complications regarding spasticity in the lower extremity. What ordered medication does the nurse administer to help alleviate this problem? Diphenhydramine (Benadryl) Heparin Lioresal (Baclofen) Pregabalin (Lyrica)

Correct response: Lioresal (Baclofen) Explanation: Spasticity, particularly in the hand, can be a disabling complication after stroke. Botulinum toxin type A injected intramuscularly into wrist and finger muscles has been shown to be effective in reducing this spasticity (although the effect is temporary, typically lasting 2 to 4 months) Other treatments for spasticity may include stretching, splinting, and oral medications such as baclofen (Lioresal).

A nurse working on a medical-surgical floor walks into a patient's room to find the patient with an altered level of consciousness (LOC). Which of the following actions would be the first priority? Determination of the cause Positioning to prevent complications Assessment of pupillary light reflexes Maintenance of a patent airway

Correct response: Maintenance of a patent airway Explanation: The most important consideration in managing the patient with altered LOC is to establish an adequate airway and ensure ventilation.

Which nursing intervention can prevent a client from experiencing autonomic dysreflexia? Placing the client in Trendelenburg's position Administering zolpidem tartrate (Ambien) Monitoring the patency of an indwelling urinary catheter Assessing laboratory test results as ordered

Correct response: Monitoring the patency of an indwelling urinary catheter Explanation: A full bladder can precipitate autonomic dysreflexia, the nurse should monitor the patency of an indwelling urinary catheter to prevent its occlusion, which could result in a full bladder. Administering zolpidem tartrate, assessing laboratory values, and placing the client in Trendelenburg's position can't prevent autonomic dysreflexia.

A nurse is continually monitoring a client with a traumatic brain injury for signs of increasing intracranial pressure. The cranial vault contains brain tissue, blood, and cerebrospinal fluid; an increase in any of the components causes a change in the volume of the others. This hypothesis is called which of the following? Cushing's Dawn phenomenon Monro-Kellie Hashimoto's disease

Correct response: Monro-Kellie Explanation: The Monro-Kellie hypothesis states that, because of the limited space for expansion in the skull, an increase in any one of its components causes a change in the volume of the others. Cushing's response is seen when cerebral blood flow decreases significantly. Systolic blood pressure increases, pulse pressure widens, and heart rate slows. Dawn phenomenon is related to high blood glucose levels in the morning in clients with diabetes. Hashimoto's disease is related to the thyroid gland. p. 2000.

The nurse is caring for a client with a spinal cord injury. What test reveals the level of spinal cord injury? Computed tomography (CT) scan Radiography Neurologic examination Myelography

Correct response: Neurologic examination Explanation: A neurologic examination reveals the level of spinal cord injury. Radiography, myelography, and a CT scan show the evidence of fracture or compression of one or more vertebrae, edema, or a hematoma.

A client with a T4-level spinal cord injury (SCI) reports severe headache. The nurse notes profuse diaphoresis of the client's forehead and scalp and suspects autonomic dysreflexia. What is the first thing the nurse will do? Lay the client flat. Notify the physician. Apply antiembolic stockings. Place the client in a sitting position.

Correct response: Place the client in a sitting position. Explanation: The nurse immediately places the client in a sitting position to lower blood pressure. Next, the nurse will do a rapid assessment to identify and alleviate the cause, and then check the bladder and bowel. The nurse will examine skin for any places of irritation. If no cause can be found, the nurse will give an antihypertensive as ordered and continue to look for cause. He or she watches for rebound hypotension once cause is alleviated. Antiembolic stockings will not decrease the blood pressure.

The nurse is caring for a client with dysphagia. Which intervention would be contraindicated while caring for this client? Placing food on the affected side of the mouth Allowing ample time to eat Testing the gag reflex before offering food or fluids Assisting the client with meals

Correct response: Placing food on the affected side of the mouth Explanation: Interventions for dysphagia include placing food on the unaffected side of the mouth, allowing ample time to eat, assisting the client with meals, and testing the client's gag reflex before offering food or fluids. Chapter 62: pp. 2032-2034.

The home health nurse is caring for a client with Parkinson's disease. The nurse understands that the purpose of adding selegiline with carbidopa-levodopa to the medication regime should result in which purpose? Relieves symptoms of dyskinesia Prevents side effects from carbidopa-levodopa Replaces dopamine Slows the progression of the disease

Correct response: Slows the progression of the disease Explanation: Selegiline increases dopaminergic activity and slows the progression of the disease. Carbidopa-levodopa is a dopamine replacement drug. Anticholinergic drugs are used to reduce the symptoms of dyskinesia and other side effects.

When caring for a client diagnosed with a brain tumor of the parietal lobe, the nurse expects to find: loss of motor function. tactile agnosia. contralateral homonymous hemianopia. short-term memory impairment.

Correct response: tactile agnosia. Explanation: The nurse should expect to find tactile agnosia (inability to identify objects by touch), a sign of a parietal lobe tumor. Short-term memory impairment occurs with a frontal lobe tumor. Contralateral homonymous hemianopia suggests an occipital lobe tumor.

When caring for a client who is post-intracranial surgery what is the most important parameter to monitor? Extreme thirst Intake and output Nutritional status Body temperature

Correct response: Body temperature Explanation: It is important to monitor the client's body temperature closely because hyperthermia increases brain metabolism, increasing the potential for brain damage. Therefore, elevated temperature must be relieved with an antipyretic and other measures. Extreme thirst, intake and output, and nutritional status are not the most important parameters to monitor

A nurse is working in a neurologist's office. The physician orders a Romberg test. What should the nurse instruct the client to do? Close eyes and discriminate between dull and sharp. Touch nose with one finger. Close eyes and jump on one foot. Close eyes and stand erect.

Correct response: Close eyes and stand erect. Explanation: In the Romberg test, the client stands erect with the feet close together and eyes closed. If the client sways as if to fall, it is considered a positive Romberg test. All of the other options include components of neurologic tests, indicating neurologic deficits and balance. Chapter 60: p. 1981.

A client has sustained a traumatic brain injury. Which of the following is the priority nursing diagnosis for this client? Deficient fluid balance related to decreased level of consciousness and hormonal dysfunction Ineffective cerebral tissue perfusion related to increased intracranial pressure Disturbed thought processes related to brain injury Ineffective airway clearance related to brain injury

Correct response: Ineffective airway clearance related to brain injury Explanation: Maintaining an airway is always the priority. All the other choices are appropriate nursing diagnoses for this client, but the priority is maintenance of the airway.

A client with spinal cord compression from a tumor must undergo diagnostic testing. Which of the following is the most likely procedure for this client? Computed tomography Magnetic resonance imaging Core needle biopsy Ultrasonography

Correct response: Magnetic resonance imaging Explanation: Magnetic resonance imaging is the most commonly used diagnostic procedure. It is the most sensitive diagnostic tool that is particularly helpful in detecting epidural spinal cord compression and vertebral bone metastases.

A patient diagnosed with multiple sclerosis (MS) has ataxia. Which of the following medications could be used to treat this clinical manifestation? Dantrium Valium Neurontin Baclofen

Correct response: Neurontin Explanation: Ataxia is a chronic problem most resistant to treatment. Medications used to treat ataxia include beta-adrenergic blockers (Inderal), antiseizure agents (Neurontin), and benzodiazepines (Klonopin). Baclofen, Dantrium, and Valium are used in the treatment of spasticity.

A physician orders aspirin, 325 mg P.O. daily for a client who has experienced a transient ischemic attack (TIA). The nurse should teach the client that the physician has ordered this medication to: reduce the chance of blood clot formation. prevent intracranial bleeding. control headache pain. enhance the immune response.

Correct response: reduce the chance of blood clot formation. Explanation: TIAs are considered forerunners of stroke. Because strokes may result from clots in cerebral vessels, physicians order aspirin to prevent clot formation by reducing platelet agglutination. A 325-mg dose of aspirin is inadequate to relieve headache pain in an adult. Aspirin doesn't affect the body's immune response. Intracranial bleeding isn't associated with TIAs, and aspirin probably would worsen any existing bleeding.

A nurse is teaching a client with multiple sclerosis (MS). When teaching the client how to reduce fatigue, the nurse should tell the client to: increase the dose of muscle relaxants. rest in an air-conditioned room. take a hot bath. avoid naps during the day.

Correct response: rest in an air-conditioned room. Explanation: Fatigue is a common symptom in clients with MS. Lowering the body temperature by resting in an air-conditioned room may relieve fatigue; however, extreme cold should be avoided. A hot bath or shower can increase body temperature, producing fatigue. Muscle relaxants, ordered to reduce spasticity, can cause drowsiness and fatigue. Frequent rest periods and naps can relieve fatigue. Other measures to reduce fatigue in the client with MS include treating depression, using occupational therapy to learn energy-conservation techniques, and reducing spasticity. Chapter 64: p. 2095.

A female client reports to a nurse that she experiences a loss of urine when she jogs. The nurse's assessment reveals no nocturia, burning, discomfort when voiding, or urine leakage before reaching the bathroom. The nurse explains to the client that this type of problem is called: functional incontinence. stress incontinence. reflex incontinence. total incontinence.

Correct response: stress incontinence. Explanation: Stress incontinence is a small loss of urine with activities that increase intra-abdominal pressure, such as running, laughing, sneezing, jumping, coughing, and bending. These symptoms occur only in the daytime. Functional incontinence is the inability of a usually continent client to reach the toilet in time to avoid unintentional loss of urine. Reflex incontinence is an involuntary loss of urine at predictable intervals when a specific bladder volume is reached. Total incontinence occurs when a client experiences a continuous and unpredictable loss of urine.

A client is actively hallucinating during an assessment. The nurse would be correct in documenting the hallucination as a disturbance in thought content. motor ability. intellectual function. emotional status.

Correct response: thought content. Explanation: Hallucinations are disturbances of thought content. They are not disturbances in motor ability, intellectual function, or emotional status. Chapter 60: p. 1979.

A 30-year-old was diagnosed with amyotrophic lateral sclerosis (ALS). Which statement by the client would indicate a need for more teaching from the nurse? "I will lose strength in my arms." "My children are at greater risk to develop this disease." "I will have progressive muscle weakness." "I need to remain active for as long as possible."

Correct response: "My children are at greater risk to develop this disease." Explanation: There is no known cause for ALS, and no reason to suspect genetic inheritance. ALS usually begins with muscle weakness of the arms and progresses. The client is encouraged to remain active for as long as possible to prevent respiratory complications.

The nurse caring for a patient with bacterial meningitis is administering dexamethasone (Decadron) that has been ordered as an adjunct to antibiotic therapy. When does the nurse know is the appropriate time to administer this medication? 1 hour after the antibiotic has infused and daily for 7 days It can be administered every 6 hours for 10 days. 15 to 20 minutes before the first dose of antibiotic and every 6 hours for the next 4 days 2 hours prior to the administration of antibiotics for 7 days

Correct response: 15 to 20 minutes before the first dose of antibiotic and every 6 hours for the next 4 days Explanation: Dexamethasone (Decadron) has been shown to be beneficial as adjunct therapy in the treatment of acute bacterial meningitis and in pneumococcal meningitis if it is administered 15 to 20 minutes before the first dose of antibiotic and every 6 hours for the next 4 days. Research suggests that dexamethasone improves the outcome in adults and does not increase the risk of gastrointestinal bleeding

The client has been brought to the emergency department by their caregiver. The caregiver says that she found the client diaphoretic, nauseated, flushed and complaining of a pounding headache when she came on shift. What are these symptoms indicative of? Concussion Contusion Spinal shock Autonomic dysreflexia

Correct response: Autonomic dysreflexia Explanation: Characteristics of this acute emergency are as follows: Severe hypertension; Slow heart rate; Pounding headache; Nausea; Blurred vision; Flushed skin; Sweating; Goosebumps (erection of pilomotor muscles in the skin); Nasal stuffiness; and Anxiety. The symptoms in the scenario are not symptoms or concussion, spinal shock, or contusion.

An older client complains of a constant headache. A physical examination shows papilledema. What may the symptoms indicate in this client? Epilepsy Trigeminal neuralgia Brain tumor Hypostatic pneumonia

Correct response: Brain tumor Explanation: The incidence of brain tumor increases with age. Headache and papilledema are less common symptoms of a brain tumor in the older adult. Symptoms of epilepsy include fits and spasms Symptoms of trigeminal neuralgia would be pain in the jaws or facial muscles. Hypostatic pneumonia develops due to immobility or prolonged bed rest in older clients.

The provider diagnoses the patient as having had an ischemic stroke. The etiology of an ischemic stroke would include which of the following? Arteriovenous malformation Cerebral aneurysm Cardiogenic emboli Intracerebral hemorrhage

Correct response: Cardiogenic emboli Explanation: Aneurysms, hemorrhages, and malformations are all examples of a hemorrhagic stroke. An embolism can block blood flow, leading to ischemia. Chapter 62: p. 2032.

The critical care nurse is caring for a client with bacterial meningitis. The client has developed cerebral vasculitis and increased ICP. What neurologic sequelae might this client develop? Damage to the olfactory nerve Damage to the facial nerve Damage to the vagal nerve Damage to the optic nerve

Correct response: Damage to the optic nerve Explanation: Neurologic sequelae in survivors include damage to the cranial nerves that facilitate vision and hearing. Sequelae to meningitis do not include damage to the vagal nerve (X), the olfactory nerve (I) or the facial nerve (VII).

A client in the emergency department has a suspected neurologic disorder. To assess gait, the nurse asks the client to take a few steps; with each step, the client's feet make a half circle. To document the client's gait, the nurse should use which term? Steppage Ataxic Dystrophic Helicopod

Correct response: Helicopod Explanation: A helicopod gait is an abnormal gait in which the client's feet make a half circle with each step. Ataxic gait is staggering and unsteady. Dystrophic gait, the client waddles with the legs far apart. Steppage gait, the feet and toes rise high off the floor and the heel comes down heavily with each step.

A nurse is caring for a client who has a history of a cerebral aneurysm. Which diagnostic test does the nurse anticipate to monitor the status of the aneurysm? echoencephalography electroencephalogram cerebral angiography milligram

Correct response: cerebral angiography Explanation: The nurse would anticipate a cerebral angiography, which detects distortion of the cerebral arteries and veins . A milligram detects abnormalities of the spinal canal. An electroencephalogram records electrical impulses of the brain. An echoencephalography is an ultrasound of the structures of the brain.

Which condition is a rare, transmissible, progressive fatal disease of the central nervous system characterized by spongiform degeneration of the gray matter of the brain? Creutzfeldt-Jakob disease Parkinson disease Multiple sclerosis Huntington disease

Correct response: Creutzfeldt-Jakob disease Explanation: Creutzfeldt-Jakob disease causes severe dementia and myoclonus. Multiple sclerosis is a chronic, degenerative, progressive disease of the central nervous system characterized by the occurrence of small patches of demyelination in the brain and spinal cord. Parkinson disease is associated with decreased levels of dopamine due to destruction of pigmented neuronal cells in the substantia nigra in the basal ganglia of the brain. Huntington disease is a chronic, progressive, hereditary disease of the nervous system that results in progressive involuntary dancelike movements and dementia.

A nurse is working in the neurologic intensive care unit and admits from the emergency department a patient with a severe head injury. Upon entering the room, the nurse observes that the patient is positioned like part A of the accompanying image. Which posturing is the patient exhibiting? Tonic clonic Decerebrate Decorticate Flaccidity

Correct response: Decorticate Explanation: An inappropriate or nonpurposeful response is random and aimless. Posturing may be decorticate or decerebrate. Decorticate posture is the flexion and internal rotation of forearms and hands. Decerebrate posture is extension and external rotation. Flaccidity is the absence of motor response; tonic clonic movements are seen with seizures.

Which is the most common cause of acute encephalitis in the United States? Herpes simplex virus St. Louis virus Western equine virus West Nile virus

Correct response: Herpes simplex virus Explanation: Viral infection is the most common cause of encephalitis. Herpes simplex virus is the most common cause of acute encephalitis in the United States. The Western equine encephalitis virus, West Nile virus, and St. Louis virus are types of arboviral encephalitis that occur in North America, but they are not the most common causes of acute encephalitis.

A patient with a brain tumor is complaining of headaches that are worse in the morning. What does the nurse know could be the reason for the morning headaches? Dehydration Increased intracranial pressure The tumor is shrinking. Migraines

Correct response: Increased intracranial pressure Explanation: Headache, although not always present, is most common in the early morning and is made worse by coughing, straining, or sudden movement. It is thought to be caused by the tumor invading, compressing, or distorting the pain-sensitive structures or by edema that accompanies the tumor, leading to increased intracranial pressure. Chapter 65: p. 2115.

A client has experienced an ischemic stroke that has damaged the frontal lobe of his brain. Which of the following deficits does the nurse expect to observe during assessment? Limited attention span and forgetfulness Hemiplegia or hemiparesis Visual and auditory agnosia Lack of deep tendon reflexes

Correct response: Limited attention span and forgetfulness Explanation: Damage to the frontal lobe may impair learning capacity, memory, or other higher cortical intellectual functions. Such dysfunction may be reflected in a limited attention span, difficulties in comprehension, forgetfulness, and a lack of motivation. Damage to the motor neurons may cause hemiparesis, hemiplegia, and a change in reflexes. Damage to the occipital lobe can result in visual agnosia. Damage to the temporal lobe can cause auditory agnosia.

A client was undergoing conservative treatment for a herniated nucleus pulposus, at L5 - S1, which was diagnosed by magnetic resonance imaging. Because of increasing neurologic symptoms, the client undergoes lumbar laminectomy. The nurse should take which step during the immediate postoperative period? Logroll the client from side to side. Discourage the client from doing any range-of-motion (ROM) exercises. Elevate the head of the bed to 90 degrees. Have the client sit up in a chair as much as possible.

Correct response: Logroll the client from side to side. Explanation: Logrolling the client maintains alignment of his hips and shoulders and eliminates twisting in his operative area. The nurse should encourage ROM exercises to maintain muscle strength. Because of pressure on the operative area, having the client sit up in a chair or with the head of the bed elevated should be allowed only for short durations. Chapter 65: p. 2138.

A client with a concussion is discharged after the assessment. Which instruction should the nurse give the client's family? Look for signs of increased intracranial pressure Emphasize complete bed rest Have the client avoid physical exertion Look for a halo sign

Correct response: Look for signs of increased intracranial pressure Explanation: The nurse informs the family to monitor the client closely for signs of increased intracranial pressure if findings are normal and the client does not require hospitalization. Signs of increased intracranial pressure include headache, blurred vision, vomiting, and lack of energy or sleepiness. The nurse looks for a halo sign to detect any cerebrospinal fluid drainage.

A client with weakness and tingling in both legs is admitted to the medical-surgical unit with a tentative diagnosis of Guillain-Barré syndrome. On admission, which assessment is most important for this client? Lung auscultation and measurement of vital capacity and tidal volume Evaluation of nutritional status and metabolic state Evaluation of pain and discomfort Evaluation for signs and symptoms of increased intracranial pressure (ICP)

Correct response: Lung auscultation and measurement of vital capacity and tidal volume Explanation: In Guillain-Barré syndrome, polyneuritis commonly causes weakness and paralysis, which may ascend to the trunk and involve the respiratory muscles. Lung auscultation and measurement of vital capacity, tidal volume, and negative inspiratory force are crucial in detecting and preventing respiratory failure — the most serious complication of polyneuritis. A peripheral nerve disorder, polyneuritis doesn't cause increased ICP. Although the nurse must evaluate the client for pain and discomfort and must assess the nutritional status and metabolic state, these aren't priorities.

A female client who reports recurring headaches, accompanied by increased irritability, photophobia, and fatigue is asked to track the headache symptoms and occurrence on a calendar log. Which is the best nursing rationale for this action? Headaches are the most common type of reported pain. Cluster headaches can cause severe debilitating pain. Migraines often coincide with menstrual cycle. Tension headaches are easier to treat.

Correct response: Migraines often coincide with menstrual cycle. Explanation: Changes in reproductive hormones as found during menstrual cycle can be a trigger for migraine headaches and may assist in the management of the symptoms. Cluster headaches can cause severe pain but are not the reason for tracking. Tension headaches can be managed but is not associated with a monthly calendar. Headaches are common but not the reason for tracking.

A patient diagnosed with multiple sclerosis (MS) has ataxia. Which of the following medications could be used to treat this clinical manifestation? Baclofen Neurontin Valium Dantrium

Correct response: Neurontin Explanation: Ataxia is a chronic problem most resistant to treatment. Medications used to treat ataxia include beta-adrenergic blockers (Inderal), antiseizure agents (Neurontin), and benzodiazepines (Klonopin). Baclofen, Dantrium, and Valium are used in the treatment of spasticity.

A nurse is caring for a client with an injury to the central nervous system. When caring for a client with a spinal cord insult that is slowing transmission of the motor neurons, in what would the nurse anticipate a delayed reaction? Cognitive ability to understand relayed information. Response due to interrupted impulses from the central nervous system Processing information transferred from the environment. Identification of information due to slowed passages of information to brain.

Correct response: Response due to interrupted impulses from the central nervous system Explanation: The central nervous system is composed of the brain and the spinal cord. Motor neurons transmit impulses from the central nervous system. Slowing transmission in this area would slow the response of transmission leading to a delay in reaction. Sensory neurons transmit impulses from the environment to the central nervous system, allowing identification of a stimulus. Cognitive centers of the brain interpret the information

Following a generalized seizure in a client, which nursing assessment is a priority for detailing the event? Seizure began at 1300 hours. Seizure was 1 minute in duration including tonic-clonic activity. Sleeping quietly after the seizure The client cried out before the seizure began.

Correct response: Seizure was 1 minute in duration including tonic-clonic activity. Explanation: Describing the length and the progression of the seizure is a priority nursing responsibility. During this time, the client will experience respiratory spasms, and their skin will appear cyanotic, indicating a period of lack of tissue oxygenation. Noting when the seizure began and presence of an aura are also valuable pieces of information. Postictal behavior should be documented along with vital signs, oxygen saturation, and assessment of tongue and oral cavity. Chapter 61: Page 2019

The nursing instructor is talking with her clinical group about the central nervous system. What should the instructor tell the students about the function of the spinal cord? Extends through the whole vertebral column Provides centers for planned action Supports the skeletal system Serves as a conduit for impulses to and from the brain

Correct response: Serves as a conduit for impulses to and from the brain Explanation: The spinal cord functions as a passageway for ascending sensory and descending motor neurons. Its two main functions are to provide centers for reflex action and to serve as a pathway for impulses to and from the brain. The spinal cord ends between the first and second lumbar vertebrae. The spinal column supports the skeletal system.

Which condition occurs when blood collects between the dura mater and arachnoid membrane? Intracerebral hemorrhage Epidural hematoma Subdural hematoma Extradural hematoma

Correct response: Subdural hematoma Explanation: A subdural hematoma is a collection of blood between the dura mater and brain, space normally occupied by a thin cushion of fluid. Intracerebral hemorrhage is bleeding in the brain or the cerebral tissue with the displacement of surrounding structures. An epidural hematoma is bleeding between the inner skull and the dura, compressing the brain underneath. An extradural hematoma is another name for an epidural hematoma. Chapter 63: p. 2056.

A client has a spinal cord injury. The home health nurse is making an initial visit to the client at home and plans on reinforcing teaching on autonomic dysreflexia. What symptom would the nurse stress to the client and his family? Rapid heart rate Runny nose Sweating Slight headache

Correct response: Sweating Explanation: Characteristics of this acute emergency are as follows: severe hypertension; slow heart rate; pounding headache; nausea; blurred vision; flushed skin; sweating; goosebumps (erection of pilomotor muscles in the skin); nasal stuffiness; and anxiety.

A nurse assesses the patient's level of consciousness using the Glasgow Coma Scale. What score indicates severe impairment of neurologic function? 6 15 9 3

Correct response: 3 Explanation: Each criterion in the Glasgow Coma Scale (eye opening, verbal response, and motor response) is rated on a scale from 3 to 15. A total score of 3 indicates severe impairment of neurologic function, brain death, or pharmacologic inhibition of the neurologic response. A score of 15 indicates that the patient is fully responsive. Chapter 61: p. 1994.

The nurse in the emergency department is caring for a patient brought in by the rescue squad after falling from a second-story window. The nurse assesses ecchymosis over the mastoid and clear fluid from the ears. What type of skull fracture is this indicative of? Frontal skull fracture Occipital skull fracture Basilar skull fracture Temporal skull fracture

Correct response: Basilar skull fracture Explanation: A fracture of the base of the skull is referred to as a basilar skull fracture. Fractures of the base of the skull tend to traverse the paranasal sinus of the frontal bone or the middle ear located in the temporal bone. Therefore, they frequently produce hemorrhage from the nose, pharynx, or ears, and blood may appear under the conjunctiva. An area of ecchymosis (bruising) may be seen over the mastoid (Battle's sign). Basilar skull fractures are suspected when CSF escapes from the ears (CSF otorrhea) and the nose (CSF rhinorrhea).

Which is the most common cause of spinal cord injury (SCI)? Acts of violence Sports-related injuries Falls Motor vehicle crashes

Correct response: Motor vehicle crashes Explanation: The most common cause of SCI is motor vehicles crashes, which account for 35% of the injuries. Falls, sports-related injuries, and acts of violence are also potential causes of SCI, but are not most common. Chapter 63: p. 2070.

The nurse is participating in a health fair for stroke prevention. Which will the nurse say is a modifiable risk factor for ischemic stroke? Social drinking Smoking Thyroid disease Advanced age

Correct response: Smoking Explanation: Modifiable risk factors for transient ischemic attack (TIA) and ischemic stroke include hypertension, type 1 diabetes, cardiac disease, smoking, and chronic alcoholism. Advanced age, gender, and race are nonmodifiable risk factors for stroke. Chapter 62: p. 4032.

A nurse is monitoring a client for increasing intracranial pressure (ICP). Early signs of increased ICP include: pupillary changes. diminished responsiveness. elevated temperature. decreasing blood pressure.

Correct response: diminished responsiveness. Explanation: Usually, diminished responsiveness is the first sign of increasing ICP. Pupillary changes occur later. Increased ICP causes systolic blood pressure to rise. Temperature changes vary and may not occur even with a severe decrease in responsiveness. p. 2009.

When caring for a client who is post-intracranial surgery, what is the most important parameter to monitor? Signs of infection Intake and output Nutritional status Body temperature

Correct response: Body temperature Explanation: It is important to monitor the client's body temperature closely; hyperthermia increases brain metabolism, increasing the potential for brain damage. Therefore, elevated temperature must be relieved with an antipyretic and other measures. Options A, B, and C are not the most important parameters to monitor.

The diagnosis of multiple sclerosis is based on which test? CSF electrophoresis Evoked potential studies Neuropsychological testing Magnetic resonance imaging

Correct response: Magnetic resonance imaging Explanation: The diagnosis of MS is based on the presence of multiple plaques in the central nervous system observed with magnetic resonance imaging. Electrophoresis of CSF identifies the presence of oligoclonal banding. Evoked potential studies can help define the extent of the disease process and monitor changes. Neuropsychological testing may be indicated to assess cognitive impairment.

A client is suspected of having had a stroke. Which is the initial diagnostic test for a stroke? Electrocardiography Noncontrast computed tomography Transcranial Doppler studies Carotid Doppler

Correct response: Noncontrast computed tomography Explanation: The initial diagnostic test for a stroke is usually a noncontrast computed tomography (CT) scan. This should be performed within 25 minutes or less from the time the client presents to the ED to determine whether the event is ischemic or hemorrhagic (the category of stroke determines treatment). Further diagnostics include a carotid Doppler, electrocardiogram, and transcranial Doppler.

The nurse has just received report on a client in the ED being transferred to the acute stroke unit with a diagnosis of a right hemispheric stroke. Which findings does the nurse understand is indicative of a right hemispheric stroke? Aphasia Altered intellectual ability Spatial-perceptual deficits Slow, cautious behavior

Correct response: Spatial-perceptual deficits Explanation: Clients with right hemispheric stroke exhibit partial perceptual deficits, left visual field deficit, and paralysis with weakness on the left side of the body. Left hemispheric damage causes aphasia, slow, cautious behavior, and altered intellectual ability.

A nurse observes that decerebrate posturing is a comatose client's response to painful stimuli. Decerebrate posturing as a response to pain indicates: risk for increased intracranial pressure. dysfunction in the brain stem. dysfunction in the cerebrum. dysfunction in the spinal column.

Correct response: dysfunction in the brain stem. Explanation: Decerebrate posturing indicates damage of the upper brain stem. Decorticate posturing indicates cerebral dysfunction. Increased intracranial pressure is a cause of decortication and decerebration. Alterations in sensation or paralysis indicate dysfunction in the spinal column. p. 1976.

The nurse is caring for a client with a traumatic brain injury. Which assessment findings indicate to the nurse that the client is developing Cushing's reflex? Select all that apply. Urine output over 100 mL/hr Systolic blood pressure is 180 mm/Hg Blood pressure is 140/38 mmHg Weakness on one side of the body Apical pulse is 42 beats per minute

Apical pulse is 42 beats per minute Blood pressure is 140/38 mmHg Systolic blood pressure is 180 mm/Hg Explanation: Signs of increasing intracranial pressure and Cushing's reflex include bradycardia, widening pulse pressure, elevated systolic blood pressure, and irregular respirations. Badycardia is a heart or apical rate below 50. Widening pulse pressure is typically defined as a large or wide difference between the two blood pressure readings (systolic and diastolic pressure). Widening pulse pressure readings are present when the difference is greater than 60. In this instance the difference between 140 and 38 is 102. Elevated systolic blood pressure and/or hypertension generally have readings above 140 mm/Hg. Urine output is not an indicator for Cushing's reflex. Weakness on one side of the body or hemiparesis is a finding associated with a stroke.

During a class on stroke, a junior nursing student asks what the clinical manifestations of stroke are. What would be the instructor's best answer? "Clinical manifestations of a stroke depend on the area of the cortex, the affected hemisphere, the degree of blockage, and the availability of collateral circulation." "Clinical manifestations of a stroke are highly variable, depending on the cardiovascular health of the client." "Clinical manifestations of a stroke generally include aphasia, one-sided flaccidity, and trouble swallowing." "Clinical manifestations of a stroke depend on how quickly the clot can be dissolved."

Correct response: "Clinical manifestations of a stroke depend on the area of the cortex, the affected hemisphere, the degree of blockage, and the availability of collateral circulation." Explanation: Clinical manifestations following a stroke are highly variable and depend on the area of the cerebral cortex and the affected hemisphere, the degree of blockage (total, partial), and the presence or absence of adequate collateral circulation. (Collateral circulation is circulation formed by smaller blood vessels branching off from or near larger occluded vessels.) Clinical manifestations of a stroke do not depend on the cardiovascular health of the client or how quickly the clot can be dissolved. Clinical manifestations of a stroke are not "general" but individual.

The nurse is providing discharge teaching for a client who was admitted to hospital after having complex partial seizures secondary to a glioma. The client has been prescribed levetiracetam to manage the seizures. What should the nurse include in the discharge teaching for this medication? "Suicidal ideation is a common side effect of this medication and should be reported immediately." "If a corticosteroid has been prescribed, do not take it at the same time as this medication." "Driving a car should be avoided until the you know how this medication effects you." "If the previous day's dose was forgotten, take two at the regular time the next day."

Correct response: "Driving a car should be avoided until the you know how this medication effects you." Explanation: The nurse should caution the client against driving until the client has a good understanding of how the medication affects his or her central nervous system. For some individuals, the degree of somnolence is much greater than for others and, in some cases, the somnolence is higher when the medication is first initiated and then begins to lesson with physiological adaptation. If a dose is forgotten, the client should be told to take the same dose as soon as he or she remembers. If the time is too close to the following day's dose, the client should be instructed to omit the previous day's dose and just take the current day's dose only. The client should never double up on the dose. There are no cautionary concerns about taking the medication at the same time as a glucocorticoid. There are no established drug-drug interactions between these two type of medications. Suicidal ideation is a rare side effect of levetiracetam. Although the nurse can provide education to the client about this rare side effect, the nurse must indicate this is not a common finding with this medication.

The nurse is seeing a client in the oncology outpatient clinic. The client has recently been diagnosed with grade I meningioma. The client asks, "Is there a cure for my condition?" How should the nurse respond? "You will need to speak to your doctor regarding questions about your prognosis." "For most clients, surgery is an effective treatment for this type of tumor." "This type of tumor is fast growing and difficult to treat." "Radiation and chemotherapy are good treatment options for this type of tumor."

Correct response: "For most clients, surgery is an effective treatment for this type of tumor." Explanation: During this client interaction, it is important that the nurse provide the newly diagnosed client with facts about the condition and use a method of communicating that helps to reduce the client's anxiety. The nurse can accomplish this by telling the client that the type of tumor that client has been diagnosed with responds well to surgery, because this is true. Grade I meningiomas are the most common type and can be cured by surgery. Radiation and chemotherapy are not used in the treatment of grade I meningiomas. The main treatment is surgery. It is incorrect to tell the client that the tumor is fast growing and difficult to treat. Grade 1 meningiomas are slow growing and respond well to surgery.

The community health nurse is conducting a home visit with a client who was discharged from hospital 3 days ago after surgical resection of a brain tumor and radiation therapy. The client is accompanied by his partner during the nurse's visit. During the visit, the client's partner becomes tearful. How should the nurse respond? "Many caregivers experience burnout. Are you experiencing symptoms depression and anxiety?" "Going through this experience with your partner has been very difficult for you, I'm sure. Can you tell me about your experience so far?" "Sometimes people are unhappy about the way they were treated in the hospital. Were you happy with the care your partner received in the hospital?" "It is okay to say you give up. Are you worried you will not be able to continue to provide care for your partner?"

Correct response: "Going through this experience with your partner has been very difficult for you, I'm sure. Can you tell me about your experience so far?" Explanation: Nurses caring for clients with brain tumors need to be able to understand the difficulties the caregivers experience. To fully understand what the caregiver may need to continue to support the client through treatment toward end of life, the nurse should aim to capture the individual perspective of the care provider. In this case, the nurse only asks closed-ended questions in the alternate answer options. In these answer options, the nurse also makes assumptions about why the client's partner is tearful. In the correct answer, the nurse asks an open-ended question, demonstrating genuine curiosity about the caregiver's experience. This type of communication alone can help the caregiver feel less isolated and more supported.

The nurse is seeing a client who has just been diagnosed with a meningioma. The client states he is confused because the provider stated, "If you have to be diagnosed with a brain tumor, this is the least harmful." The client asks the nurse for clarification. How should the nurse respond? "It would have been important for you to clarify your provider's statement during your appointment. It is not within my scope to discuss the details of your diagnosis." "I am unable to interpret what your provider meant by making that statement; however, it is true that meningiomas are slow growing tumors that are not typically fatal." "I am assuming your provider was trying to explain to you that meningiomas have a high cure rate if treated with surgery, chemotherapy and radiation aggressively." "It is likely that your provider was trying to be as supportive as possible with those positive words. You need a lot of support during this challenging time."

Correct response: "I am unable to interpret what your provider meant by making that statement; however, it is true that meningiomas are slow growing tumors that are not typically fatal." Explanation: The nurse should inform the client that nurses cannot interpret what another provider meant by the statement. The nurse can provide client education regarding what is known about the type of brain tumor the client has been diagnosed with. It would be incorrect for the nurse to state that the treatment for this type of brain tumor is aggressive. The tumor is slow growing. and sometime treatment is a 'wait-and-see' approach. Thus, surgery, chemotherapy and radiation would not typically be used together or aggressively. By telling the client the provider was trying to be supportive with the statement that was made communicates to the client that the provider was not telling the truth about the nature of the diagnosis. In this case, the nurse is making an assumption and should not try to interpret for the client what the provider said. It would be countertherapeutic and serve to increase the client's anxiety if the nurse stated discussing the details of the client's diagnosis is not within the nurse's scope. The nurse can provide information about the type of brain tumor within the scope of practice.

The nurse is caring for a client following a spinal cord injury who has a halo device in place. The client is preparing for discharge. Which statement by the client indicates the need for further instruction? "I'll check under the liner for blisters and redness." "I will change the vest liner periodically." "I can apply powder under the liner to help with sweating." "If a pin becomes detached, I'll notify the surgeon."

Correct response: "I can apply powder under the liner to help with sweating." Explanation: Powder is not used inside the vest because it may contribute to the development of pressure ulcers. The areas around the four pin sites of a halo device are cleaned daily and observed for redness, drainage, and pain. The pins are observed for loosening, which may contribute to infection. If one of the pins becomes detached, the head is stabilized in a neutral position by one person while another notifies the neurosurgeon. The skin under the halo vest is inspected for excessive perspiration, redness, and skin blistering, especially on the bony prominences. The vest is opened at the sides to allow the torso to be washed. The liner of the vest should not become wet because dampness can cause skin excoriation. The liner should be changed periodically to promote hygiene and good skin care.

The nurse is providing end-of-life care to a client who was diagnosed with glioblastoma multiforme (GBM) 8 months ago. Despite a calm interaction with the client 1 hour ago, the client is now angry and yells, "Get out of my room and don't touch me anymore. I don't need your help!" How should the nurse respond? "You are not permitted to speak to me this way. I am a professional and I deserve for you to treat me with respect." "I can see you no longer want me as your nurse today. I will ask one of my colleagues to come in to complete the rest of my assessment." "I can tell now is not the right time for me to come in and check on you. Please let me know when it is a better time for me to come back." "I am your nurse and caring for you is my obligation. If you no longer want my care, you have to make a request to my supervisor."

Correct response: "I can tell now is not the right time for me to come in and check on you. Please let me know when it is a better time for me to come back." Explanation: Personality changes, mood swings and irritability can be common manifestations of both growth of the brain tumor and also the process of grief and loss, such as in the case of the client who is receiving end-of-life care. The client's anger and yelling at the nurse is indicative of ineffective coping and warrants the nurse to take a therapeutic approach when responding to the anger. Acknowledging that the client is not ready to receive care at the moment and asking the client to contact the nurse when he or she is ready enables to client to maintain control and promotes self-esteem. Telling the client to speak to the nurse's supervisor does not promote a strong nurse-patient relationship and is not a supportive way to manage end-of-life care. Telling the client that he or she is not permitted to speak to the nurse "that way" may increase the client's anger and puts limits on the client's sense of control. This response does not promote an effective nurse-patient relationship. The nurse must use extra caution when responding to a client who is experiencing emotional swings when faced with death and dying. By stating, "I can see you no longer want me as your nurse," the nurse is making an assumption that the client does not want him or her as the nurse any longer. By making this statement, the nurse is limiting opportunities for the client to verbalize feelings and emotions related to stress, grief and loss. Chapter 65: p. 2119.

The nurse is caring for a patient in the emergency department with an onset of pain related to trigeminal neuralgia. What subjective data stated by the patient does the nurse determine triggered the paroxysms of pain? "I was sitting at home watching television." "I was brushing my teeth." "I was taking a bath." "I was putting my shoes on."

Correct response: "I was brushing my teeth." Explanation: Trigeminal neuralgia is a condition of the fifth cranial nerve that is characterized by paroxysms of sudden pain in the area innervated by any of the three branches of the nerve. Paroxysms can occur with any stimulation of the terminals of the affected nerve branches, such as washing the face, shaving, brushing the teeth, eating, and drinking. Chapter 64: p. 2108.

A client with post-polio syndrome displays fatigue and decreased muscle strength. How should the nurse best respond to the client? "Intravenous immunoglobulin infusion may help you." "These symptoms are not related to your past diagnosis." "Once you sleep, you should be fine." "This will pass, you need to relax."

Correct response: "Intravenous immunoglobulin infusion may help you." Explanation: There is no specific treatment for post-polio syndrome; however, the infusion of IV immunoglobulin has been shown to help with the physical pain and weakness. Sleeping and relaxation may not assist the client with post-polio syndrome. The syndrome is very common and is most likely related to the past diagnosis of polio.

A 30-year-old was diagnosed with amyotrophic lateral sclerosis (ALS). Which statement by the client would indicate a need for more teaching from the nurse? "I need to remain active for as long as possible." "I will lose strength in my arms." "My children are at greater risk to develop this disease." "I will have progressive muscle weakness."

Correct response: "My children are at greater risk to develop this disease." Explanation: There is no known cause for ALS, and no reason to suspect genetic inheritance. ALS usually begins with muscle weakness of the arms and progresses. The client is encouraged to remain active for as long as possible to prevent respiratory complications.

A family will be providing care at home to an immobilized patient at risk for impaired skin integrity. What statement made by the family indicates that more teaching is needed? "We need to use a mild soap on the skin when bathing." "We need to avoid massaging any reddened areas that may appear." "We elevate the head of the bed to comfort level throughout the day." "We need to make sure that the patient drinks enough fluids."

Correct response: "We elevate the head of the bed to comfort level throughout the day." Explanation: The elevation of the head of the bed increases the shearing force, so the semi-reclining position should be avoided. Keeping the patient well hydrated helps prevent skin cracking and infection because intact healthy skin is the body's first line of defense. Caregivers should use a mild soap for bathing. Massaging any reddened areas is to be avoided, because this may increase the damage to already traumatized skin and tissue.

A client who has just been diagnosed with mixed muscular dystrophy asks the nurse about the usual course of this disease. How should the nurse respond? "You may experience progressive deterioration in all voluntary muscles." "This form of muscular dystrophy is a relatively benign disease that progresses slowly." "The strength of your arms and pelvic muscles will decrease gradually, but this should cause only slight disability." "You should ask your physician about that."

Correct response: "You may experience progressive deterioration in all voluntary muscles." Explanation: The nurse should tell the client that muscular dystrophy causes progressive, symmetrical wasting of skeletal muscles, without neural or sensory defects. The mixed form of the disease typically strikes between ages 30 and 50 and progresses rapidly, causing deterioration of all voluntary muscles. Because the client asked the nurse this question directly, the nurse should answer and not simply refer the client to the physician. Limb-girdle muscular dystrophy causes a gradual decrease in arm and pelvic muscle strength, resulting in slight disability. Facioscapulohumeral muscular dystrophy is a slowly progressive, relatively benign form of muscular dystrophy; it usually arises before age 10.

A client who has just been diagnosed with mixed muscular dystrophy asks the nurse about the usual course of this disease. How should the nurse respond? "You may experience progressive deterioration in all voluntary muscles." "This form of muscular dystrophy is a relatively benign disease that progresses slowly." "You should ask your physician about that." "The strength of your arms and pelvic muscles will decrease gradually, but this should cause only slight disability."

Correct response: "You may experience progressive deterioration in all voluntary muscles." Explanation: The nurse should tell the client that muscular dystrophy causes progressive, symmetrical wasting of skeletal muscles, without neural or sensory defects. The mixed form of the disease typically strikes between ages 30 and 50 and progresses rapidly, causing deterioration of all voluntary muscles. Because the client asked the nurse this question directly, the nurse should answer and not simply refer the client to the physician. Limb-girdle muscular dystrophy causes a gradual decrease in arm and pelvic muscle strength, resulting in slight disability. Facioscapulohumeral muscular dystrophy is a slowly progressive, relatively benign form of muscular dystrophy; it usually arises before age 10. Chapter 65: p. 2132.

When planning care for a client with a head injury, which position should the nurse include in the care plan to enhance client outcomes? Trendelenburg's 30-degree head elevation Flat Side-lying

Correct response: 30-degree head elevation Explanation: For clients with increased intracranial pressure (ICP), the head of the bed should be elevated to 30 degrees to promote venous outflow. Trendelenburg's position is contraindicated because it can raise ICP. Flat or neutral positioning is indicated when elevating the head of the bed would increase the risk of neck injury or airway obstruction. A side-lying position isn't specifically a therapeutic treatment for increased ICP. Chapter 63: p. 2064.

An emergency department nurse understands that a 110-lb (50-kg) recent stroke victim will receive at least the minimum dose of recombinant tissue plasminogen activator (t-PA). What minimum dose will the client receive? 45 mg 50 mg 90 mg 85 mg

Correct response: 45 mg Explanation: The client is weighed to determine the dose of t-PA. Two or more IV sites are established prior to administration of t-PA (one for the t-PA and the other for administration of IV fluids). The dosage for t-PA is 0.9 mg/kg, with a maximum dose of 90 mg. 50 kg X 0.9 mg= 45 mg dose

A client has been hospitalized for diagnostic testing. The client has just been diagnosed with multiple sclerosis, which the physician explains is an autoimmune disorder. How would the nurse explain an autoimmune disease to the client? A disorder in which the body does not have enough immunoglobulins A disorder in which killer T cells and autoantibodies attack or destroy natural cells—those cells that are "self" A disorder in which the body has too many immunoglobulins A disorder in which histocompatible cells attack the immunoglobulins

Correct response: A disorder in which killer T cells and autoantibodies attack or destroy natural cells—those cells that are "self" Explanation: Autoimmune disorders are those in which killer T cells and autoantibodies attack or destroy natural cells—those cells that are "self." Autoantibodies, antibodies against self-antigens, are immunoglobulins. They target histocompatible cells, cells whose antigens match the person's own genetic code. Autoimmune disorders are not caused by too many or too few immunoglobulins, and histocompatible cells do not attack immunoglobulins in an autoimmune disorder. Chapter 64: p. 2096.

The nurse is assisting with a lumbar puncture and observes that when the physician obtains CSF, it is clear and colorless. What does this finding indicate? Local trauma from the insertion of the needle An overwhelming infection A subarachnoid hemorrhage A normal finding; the fluid will be sent for testing to determine other factors

Correct response: A normal finding; the fluid will be sent for testing to determine other factors Explanation: The CSF should be clear and colorless. Pink, blood-tinged, or grossly bloody CSF may indicate a subarachnoid hemorrhage. The CSF may be bloody initially because of local trauma but becomes clearer as more fluid is drained. Specimens are obtained for cell count, culture, glucose, protein, and other tests as indicated. The specimens should be sent to the laboratory immediately because changes will take place and alter the result if the specimens are allowed to stand.

The nurse is administering the IV antiviral medication ganciclovir (Cytovene) to the patient with HSV-1 encephalitis. What is the best way for the nurse to administer the medication to avoid crystallization of the medication in the urine? Administer the medication rapidly over 15 minutes with 100 mL of normal saline. Administer in a drip over 4 hours. Dilute the medicine in 500 mL of lactated Ringer's solution. Administer via slow IV over 1 hour.

Correct response: Administer via slow IV over 1 hour. Explanation: Antiviral agents, Acyclovir (Zovirax) or Ganciclovir (Cytovene), are the medications of choice in the treatment for HSV. Early administration of antiviral agents (usually well tolerated) improves the prognosis associated with HSV-1 encephalitis. Slow IV administration over 1 hour prevents crystallization of the medication in the urine.

A client is transferred to the intensive care unit after evacuation of a subdural hematoma. Which nursing intervention reduces the client's risk of increased intracranial pressure (ICP)? Suctioning the client once each shift Encouraging oral fluid intake Elevating the head of the bed 90 degrees Administering a stool softener as ordered

Correct response: Administering a stool softener as ordered Explanation: To prevent the client from straining at stool, which may cause a Valsalva maneuver that increases ICP, the nurse should institute a regular bowel program that includes use of a stool softener. For a client at risk for increased ICP, the nurse should prevent, not encourage, oral fluid intake and should elevate the head of the bed only 30 degrees. Suctioning, indicated for a client with lung congestion, isn't necessary for this client.

The school nurse notes a 6-year-old running across the playground with his friends. The child stops in midstride, freezing for a few seconds. Then the child resumes his progress across the playground. The school nurse suspects what in this child? A tonic-clonic seizure An absence seizure A myoclonic seizure A partial seizure

Correct response: An absence seizure Explanation: Absence seizures, formerly referred to as petit mal seizures, are more common in children. They are characterized by a brief loss of consciousness during which physical activity ceases. The person stares blankly; the eyelids flutter; the lips move; and slight movement of the head, arms, and legs occurs. These seizures typically last for a few seconds, and the person seldom falls to the ground. Because of their brief duration and relative lack of prominent movements, these seizures often go unnoticed. People with absence seizures can have them many times a day. Partial, or focal, seizures begin in a specific area of the cerebral cortex. Both myoclonic and tonic-clonic seizures involve jerking movements

The nurse is admitting a client into the rehabilitation unit after an industrial accident. The client's nursing diagnoses include disturbed sensory perception and the nurse identifies that he has decreased strength and dexterity. The nurse should know that this client may need what to accomplish self-care? A personal health care aide An assisted-living environment Advice from his family Appropriate assistive devices

Correct response: Appropriate assistive devices Explanation: Clients with impaired mobility, sensation, strength, or dexterity may need to use assistive devices to accomplish self-care. An assisted living environment is less common than the use of assistive devices. Family involvement is imperative, but this may or may not take the form of advice. A health care aide is not needed by most clients. Chapter 64: p. 2099.

A client is ordered to undergo CT of the brain with IV contrast. Before the test, the nurse should complete which action first? Obtain two large-bore IV lines. Assess the client for medication allergies. Obtain a blood sample to evaluate BUN and creatinine concentrations. Maintain the client NPO for 6 hours before the test.

Correct response: Assess the client for medication allergies. Explanation: If a contrast agent is used, the client must be assessed before the CT scan for an iodine/shellfish allergy, because the contrast agent used may be iodine based. If the client has no allergies to iodine, then kidney function must also be evaluated, as the contrast material is cleared through the kidneys. A suitable IV line for contrast injection and a period of fasting (usually 4 hours) are required before the study. Clients who receive an IV contrast agent are monitored during and after the procedure for allergic reactions and changes in kidney function.

A client is admitted to the hospital after sustaining a closed head injury in a skiing accident. The physician ordered neurologic assessments to be performed every 2 hours. The client's neurologic assessments have been unchanged since admission, and the client is complaining of a headache. Which intervention by the nurse is best? Administer codeine 30 mg by mouth as ordered and continue neurologic assessments as ordered. Reassure the client that a headache is expected and will go away without treatment. Notify the physician; a headache is an early sign of worsening neurologic status. Assess the client's neurologic status for subtle changes, administer acetaminophen, and then reassess the client in 30 minutes.

Correct response: Assess the client's neurologic status for subtle changes, administer acetaminophen, and then reassess the client in 30 minutes. Explanation: Headache is common after a head injury. Therefore, the nurse should administer acetaminophen to try to manage the client's pain without causing sedation. The nurse should then reassess the client in 30 minutes to note the effectiveness of the pain medication. Administering codeine, an opioid, could cause sedation that may mask changes in the client's neurologic status. Although a headache is expected, the client should receive treatment to alleviate pain. The nurse should notify the physician if the client's neurologic status changes or if treatment doesn't relieve the headache.

A client has experienced an ischemic stroke that has damaged the temporal (lateral and superior portions) lobe. Which of the following deficits would the nurse expect during assessment of this client? Hemiplegia or hemiparesis Auditory agnosia Limited attention span and forgetfulness Lack of deep tendon reflexes

Correct response: Auditory agnosia Explanation: Damage to the occipital lobe can result in visual agnosia. Damage to the temporal lobe can cause auditory agnosia. Damage has occurred to the frontal lobe, learning capacity, memory, or other higher cortical intellectual functions may be impaired. Such dysfunction may be reflected in a limited attention span, difficulties in comprehension, forgetfulness, and lack of motivation. Damage to motor neurons may cause hemiparesis, hemiplegia, and a change in reflexes.

The nurse is caring for a client who was diagnosed with a glioma 5 months ago. Today, the client was brought to the emergency department by his caregiver because he collapsed at home. The nurse suspects late signs of rising intracranial pressure (ICP) when which blood pressure and pulse readings are noted? BP = 175/45 mm Hg; HR = 42 bpm BP =130/80 mm Hg; HR = 55 bpm BP = 150/90 mm Hg; HR = 90 bpm BP = 90/50 mm Hg; HR = 75 bpm

Correct response: BP = 175/45 mm Hg; HR = 42 bpm Explanation: With a blood pressure of 175/45 mm Hg, it is evident that this client is experiencing progressively rising ICP, resulting from an advanced stage of the brain tumor. This blood pressure demonstrates a wide pulse pressure, meaning the difference between systolic and diastolic pressure is large. A heart rate of 42 bpm indicates the client is bradycardic. This finding paired with hypertensive blood pressure with a widening pulse pressure are part of the Cushing triad related to increased ICP. Chapter 65: p. 2114.

The nurse is using a measurement tool to determine a patient's level of independence in activities of daily living, such as continence, toileting, transfers, and ambulation. What would be the appropriate tool for the nurse to use? Barthel Index Patient evaluation conference system The Pulses Profile The Braden Scale

Correct response: Barthel Index Explanation: The Barthel Index is used to measure the patient's level of independence in ADLs, continence, toileting, transfers, and ambulation (or wheelchair mobility). This scale does not address communicative or cognitive abilities. The Patient Evaluation Conference System (PECS), which contains 15 categories, is a comprehensive assessment scale that includes such areas as medications, pain, nutrition, use of assistive devices, psychological status, vocation, and recreation. The PULSES profile is used to assess physical condition (e.g., health/ illness status), upper extremity functions (e.g., eating, bathing), lower extremity functions (e.g., transfer, ambulation), sensory function (e.g., vision, hearing, speech), bowel and bladder function (i.e., control of bowel or bladder), and situational factors (e.g., social and financial support). Each of these areas is rated on a scale from one (independent) to four (greatest dependency). Scales such as the Braden scale (Table 10-3) or Norton scale may be used to facilitate systematic assessment and quantification of a patient's risk for pressure ulcer

For a patient with an SCI, why is it beneficial to administer oxygen to maintain a high partial pressure of oxygen (PaO2)? Because hypoxemia can create or worsen a neurologic deficit of the spinal cord To prevent secondary brain injury So that the patient will not have a respiratory arrest To increase cerebral perfusion pressure

Correct response: Because hypoxemia can create or worsen a neurologic deficit of the spinal cord Explanation: Oxygen is administered to maintain a high partial pressure of arterial oxygen (PaO2) because hypoxemia can create or worsen a neurologic deficit of the spinal cord.

A nurse caring for a patient with head trauma will be monitoring the patient for Cushing's triad. What will the nurse recognize as the symptoms associated with Cushing's triad? Select all that apply. Pupillary constriction Bradycardia Hypertension Tachycardia Bradypnea

Correct response: Bradycardia Bradypnea Hypertension Explanation: At a certain point as intracranial pressure increases due to an injury, the brain's ability to autoregulate becomes ineffective and decompensation (ischemia and infarction) begins. When this occurs, the patient exhibits significant changes in mental status and vital signs. Bradycardia, hypertension, and bradypnea associated with this deterioration are known as Cushing's triad, which is a grave sign

A 76-year-old client is brought to the clinic by his daughter. The daughter states that her father has had two transient ischemic attacks (TIAs) in the past week. The physician orders carotid angiography, and the report reveals that the carotid artery has been narrowed by atherosclerotic plaques. What treatment option does the nurse expect the physician to offer this client to increase blood flow to the brain? Stent placement Removal of the carotid artery Percutaneous transluminal coronary artery angioplasty Carotid endarterectomy

Correct response: Carotid endarterectomy Explanation: If narrowing of the carotid artery by atherosclerotic plaques is the cause of the TIAs, a carotid endarterectomy (surgical removal of atherosclerotic plaque) is a treatment option. A balloon angioplasty, a procedure similar to a percutaneous transluminal coronary artery angioplasty, is performed to dilate the carotid artery and increase blood flow to the brain. Options A, B, and C are not surgical options to increase blood flow through the carotid artery to the brain.

The nurse is caring for a client who has been hospitalized for investigation of a sudden change in gait due to loss of balance and coordination. A magnetic resonance imaging scan reveals the client has a brain tumor. On or close to which brain structure is the tumor most likely situated? Pituitary gland Cerebellum Brain stem Temporal lobe

Correct response: Cerebellum Explanation: The cerebellum is the brain structure responsible for balance, coordination and fine muscle control. The tumor is most likely located on or near this brain structure. A tumor located on or near the brain stem would more likely cause changes in autonomic functioning such as blood pressure. The temporal lobe is responsible for language comprehension, behavior, memory, hearing and emotions. A tumor effecting the pituitary gland would result in hormonal changes as this structure is responsible for hormones, growth and reproductive processes in the body. Chapter 65: p. 2116.

The physician's office nurse is caring for a client who has a history of a cerebral aneurysm. Which diagnostic test does the nurse anticipate to monitor the status of the aneurysm? Electroencephalogram Myelogram Echoencephalography Cerebral angiography

Correct response: Cerebral angiography Explanation: The nurse would anticipate a cerebral angiography, which detects distortion of the cerebral arteries and veins . A myelogram detects abnormalities of the spinal canal. An electroencephalogram records electrical impulses of the brain. An echoencephalography is an ultrasound of the structures of the brain.

When performing a postoperative assessment on a client who has undergone surgery to manage increased intracranial pressure (ICP), a nurse notes an ICP reading of 0 mm Hg. Which action should the nurse perform first? Continue the assessment because no actions are indicated at this time. Check the equipment. Contact the physician to review the care plan. Document the reading because it reflects that the treatment has been effective.

Correct response: Check the equipment. Explanation: A reading of 0 mm Hg indicates equipment malfunction. The nurse should check the equipment and report problems. Normal and stable ICP values are less than 15 mm Hg. Some pressure is always present in the cranial vault. The nurse shouldn't contact the physician to review the care plan at this time. The nurse needs to complete the assessment of the client and equipment before making a report to the physician.

The office nurse is reviewing an 80-year-old female client's reports related to the onset of a severe headache, rated at 9 out of 10 on the pain scale, with recent onset. The client denies any visual changes. During a prior visit to the office a few months ago, the client had reported a ground-level fall as a result of falling off a chair and hitting the back of their head. The client had been taken to the emergency department, where imaging was performed with negative results. Complete the following sentence by choosing from the lists of options. The nurse anticipates that the client has developed Select... stroke acute subdural hematoma chronic subdural hematoma and that Select... coagulation profile electrocardiogram (ECG) computed tomography (CT) imaging of the brain will be ordered.

Correct response: Chronic subdural hematoma computed tomography (CT) imaging of the brain Explanation: This client has had a prior head trauma with a negative imaging scan. Prior head trauma can lead to the development of a chronic subdural hematoma, which presents with symptoms such as severe headache, mental deterioration, focal neurologic changes, personality changes, and/or symptoms that the client is having a stroke. There is no indication that the client had follow-up imaging based on the prior head trauma, which should be included in protocol management of head injuries. Prior head trauma can lead to the development of a chronic subdural hematoma. Based on the clinical presentation of a severe headache, this is the most likely clinical diagnosis. Based on the clinical presentation, follow-up imaging is indicated to confirm the presence of a chronic subdural hematoma, which can occur following a recent head trauma. Because the head trauma occurred a few months ago, an acute finding would have presented earlier, at the time of injury. The differential diagnosis of chronic subdural hematoma includes a stroke but there is insufficient clinical evidence to support this finding. An electrocardiogram (ECG) is not indicated at this time because there is no provided clinical evidence of any cardiac abnormalities. Coagulation studies are not indicated at this time because the priority is to obtain an imaging study. Chapter 63: Page 2059

While making initial rounds after coming on shift, the nurse finds a client thrashing about in bed with a severe headache. The client tells the nurse the pain is behind the right eye, which is red and tearing. What type of headache would the nurse suspect this client of having? Sinus Tension Cluster Migraine

Correct response: Cluster Explanation: A cluster headache has pain on one side of the head, usually behind the eye, accompanied by nasal congestion, rhinorrhea (watery discharge from the nose), and tearing and redness of the eye. The pain is so severe that the person is not likely to lie still; instead, the person may pace or thrash about.

The critical care nurse is giving end-of-shift report on a client. The nurse uses the Glasgow Coma Scale (GCS) to assess the level of consciousness (LOC) of a female client and reports to the oncoming nurse that the client has an LOC of 6. What does an LOC score of 6 in a client indicate? Comatose Normal Somnolence Stupor

Correct response: Comatose Explanation: The GSC is used to measure LOC and consists of three parts: eye opening response, best verbal response, and best motor response. A normal response is 15. A score of 7 or less is considered comatose.

Which is the primary medical management of arthropod-borne virus (arboviral) encephalitis? Preventing muscular atrophy Maintaining hemodynamic stability and adequate cardiac output Controlling seizures and increased intracranial pressure Preventing renal insufficiency

Correct response: Controlling seizures and increased intracranial pressure Explanation: There is no specific medication for arboviral encephalitis; therefore symptom management is key. Medical management is aimed at controlling seizures and increased intracranial pressure.

Which type of brain injury is characterized by a loss of consciousness associated with stupor and confusion? Concussion Intracranial hemorrhage Diffuse axonal injury Contusion

Correct response: Contusion Explanation: Other characteristics can include tissue alteration and neurologic deficit without hematoma formation, alteration in consciousness without localizing signs, and hemorrhage into the tissue that varies in size and is surrounded by edema. The effects of injury (hemorrhage and edema) peak after about 18 to 36 hours. A concussion is a temporary loss of neurologic function with no apparent structural damage. A diffuse axonal injury involves widespread damage to the axons in the cerebral hemispheres, corpus callosum, and brainstem. An intracranial hemorrhage is a collection of blood that develops within the cranial vault.

A client with a traumatic brain injury has already displayed early signs of increasing intracranial pressure (ICP). Which of the following would be considered late signs of increasing ICP? Mental confusion and pupillary changes Loss of gag reflex and mental confusion Decerebrate posturing and loss of corneal reflex Complaints of headache and lack of pupillary response

Correct response: Decerebrate posturing and loss of corneal reflex Explanation: Early indications of increasing ICP include disorientation, restlessness, increased respiratory effort, mental confusion, pupillary changes, weakness on onside of the body or in one extremity, and constant, worsening headache. Later indications of increasing ICP include decreasing LOC until client is comatose, decreased or erratic pulse and respiratory rate, increased blood pressure and temperature, widened pulse pressure, Cheyne-Stokes breathing, projectile vomiting, hemiplegia or decorticate or decerebrate posturing, and loss of brain stem reflexes (pupillary, corneal, gag, and swallowing).

A client with a traumatic brain injury has already displayed early signs of increasing intracranial pressure (ICP). Which of the following would be considered late signs of increasing ICP? Mental confusion and pupillary changes Complaints of headache and lack of pupillary response Loss of gag reflex and mental confusion Decerebrate posturing and loss of corneal reflex

Correct response: Decerebrate posturing and loss of corneal reflex Explanation: Early indications of increasing ICP include disorientation, restlessness, increased respiratory effort, mental confusion, pupillary changes, weakness on onside of the body or in one extremity, and constant, worsening headache. Later indications of increasing ICP include decreasing LOC until client is comatose, decreased or erratic pulse and respiratory rate, increased blood pressure and temperature, widened pulse pressure, Cheyne-Stokes breathing, projectile vomiting, hemiplegia or decorticate or decerebrate posturing, and loss of brain stem reflexes (pupillary, corneal, gag, and swallowing).

The nurse is caring for a client with a history of transient ischemic attacks (TIAs) and moderate carotid stenosis who has undergone a carotid endarterectomy. Which postoperative finding would cause the nurse the most concern? Blood pressure 128/86 mm Hg Neck pain rated 3 of 10 (on a 0 to 10 pain scale) Mild neck edema Difficulty swallowing

Correct response: Difficulty swallowing Explanation: The client's inability to swallow without difficulty would cause the nurse the most concern. Difficulty swallowing, hoarseness, or other signs of cranial nerve dysfunction must be assessed. The nurse focuses on assessment of the following cranial nerves: facial (VII), vagus (X), spinal accessory (XI), and hypoglossal (XII). Some edema in the neck after surgery is expected; however, extensive edema and hematoma formation can obstruct the airway. Emergency airway supplies, including those needed for a tracheostomy, must be available. The client's neck pain and mildly elevated blood pressure need to be addressed but would not cause the nurse the most concern. Hypotension is avoided to prevent cerebral ischemia and thrombosis. Uncontrolled hypertension may precipitate cerebral hemorrhage, edema, hemorrhage at the surgical incision, or disruption of the arterial reconstruction. Chapter 62: pp. 2032-2036.

The nurse is caring for a client hospitalized after a motor vehicle accident. The client has a comorbidity of Parkinson's disease. Why should the nurse closely monitor the condition and the drug regimen of a client with Parkinson's disease? Drugs administered may not cause the requisite therapeutic effect. Drugs administered may cause a wide variety of adverse effects. Clients generally do not adhere to the drug regimen. Clients take an assortment of different drugs.

Correct response: Drugs administered may cause a wide variety of adverse effects. Explanation: Drugs administered for Parkinsonism may cause a wide variety of adverse effects, which requires careful observation of the client. Over time, clients may respond less and less to their standard drug therapy and have more frequent "off episodes" of hypomobility. As a result, the nurse should administer the drugs closely to the schedule. Generally, a single drug called levodopa is administered to clients with Parkinson's disease. It is also not true that drugs may not cause the requisite therapeutic effect or such clients do not adhere to the drug regimen. Chapter 65: p. 2124.

The nurse working on the neurological unit is caring for a client with a basilar skull fracture. During the assessment, the nurse expects to observe Battle's sign, which is a sign of basilar skull fracture. Which of the following correctly describes Battle's sign? Bruising under the eyes Drainage of cerebrospinal fluid from the ears Ecchymosis over the mastoid Drainage of cerebrospinal fluid from the nose

Correct response: Ecchymosis over the mastoid Explanation: With fractures of the base of the skull, an area of ecchymosis (bruising) may be seen over the mastoid and is called Battle's sign. Basilar skull fractures are suspected when cerebrospinal fluid escapes from the ears or the nose.

Which term refers to a method of recording, in graphic form, the electrical activity of a muscle? Electroencephalography Electrogastrography Electrocardiography Electromyography

Correct response: Electromyography Explanation: An electromyogram is obtained by inserting needle electrodes into the skeletal muscles to measure changes in the electrical potential of the muscles. Electroencephalography is a method of recording, in graphic form, the electrical activity of the brain. Electrocardiography is performed to assess the electrical activity of the heart. Electrogastrography is an electrophysiologic study performed to assess gastric motility disturbances. p. 1989.

After a stroke, a client is admitted to the facility. The client has left-sided weakness and an absent gag reflex. He's incontinent and has a tarry stool. His blood pressure is 90/50 mm Hg, and his hemoglobin is 10 g. Which nursing intervention is a priority for this client? Checking stools for occult blood Performing range-of-motion (ROM) exercises on the left side Elevating the head of the bed to 30 degrees Keeping skin clean and dry

Correct response: Elevating the head of the bed to 30 degrees Explanation: Because the client's gag reflex is absent, elevating the head of the bed to 30 degrees helps minimize the client's risk of aspiration. Checking the stools, performing ROM exercises, and keeping the skin clean and dry are important, but preventing aspiration through positioning is the priority.

A patient brought to the hospital after a skiing accident was unconscious for a brief period of time at the scene, then woke up disoriented and refused to go to the hospital for treatment. The patient became very agitated and restless, then quickly lost consciousness again. What type of TBI is suspected in this situation? Epidural hematoma Chronic subdural hematoma Acute subdural hematoma Grade 1 concussion

Correct response: Epidural hematoma Explanation: Epidural hematomas are often characterized by a brief loss of consciousness followed by a lucid interval in which the patient is awake and conversant. The patient then becomes increasingly restless, agitated, and confused as the condition progresses to coma.

A nurse is caring for a client with L1-L2 paraplegia who is undergoing rehabilitation. Which goal is appropriate? Preventing autonomic dysreflexia by preventing bowel impaction Establishing an ambulation program using short leg braces Establishing an intermittent catheterization routine every 4 hours Managing spasticity with range-of-motion exercises and medications

Correct response: Establishing an intermittent catheterization routine every 4 hours Explanation: The paraplegic client with an L1-L2 injury will demonstrate flaccid paralysis. Developing an intermittent catheterization routine offers a way of manually draining the bladder, eliminating the need for an indwelling urinary catheter. With an injury at L1-L2, ambulation may be possible with long leg braces but not with short leg braces. Spasticity and autonomic dysreflexia are seen in clients with upper motor injuries above T6, not L1-L2 injuries.

The rehabilitation nurse is caring for a 25-year-old client who suffered extensive injuries in a motorcycle accident. During each interaction with the client, what action should the nurse perform most frequently? Evaluate the client's positioning. Complete a physical assessment. Plan nursing interventions. Assist the client to ambulate.

Correct response: Evaluate the client's positioning. Explanation: During each client contact, the nurse evaluates the client's position and assists the client to achieve and maintain proper positioning and alignment. The nurse does not complete a physical assessment during each client contact. Similarly, the nurse does not plan nursing interventions or assist the client to ambulate each time the nurse has contact with the client.

A nurse is communicating with a client who has aphasia after having a stroke. Which action should the nurse take? Talk in a louder than normal voice. Face the client and establish eye contact. Use one long sentence to say everything that needs to be said. Keep the television on while she speaks.

Correct response: Face the client and establish eye contact. Explanation: When speaking with a client who has aphasia, the nurse should face the client and establish eye contact. The nurse should use short phrases, not one long sentence, and give the client time between phrases to understand what is being said. Keeping extraneous and background noise such as the television to a minimum helps the client concentrate on what is being said. It isn't necessary to speak in a louder or softer voice than normal.

While performing an initial nursing assessment on a client admitted with suspected tic douloureux (trigeminal neuralgia), for which of the following would the nurse expect to observe? Facial pain in the areas of the fifth cranial nerve Ptosis and diplopia Hyporeflexia and weakness of the lower extremities Fatigue and depression

Correct response: Facial pain in the areas of the fifth cranial nerve Explanation: Tic douloureux (trigeminal neuralgia) is manifested by pain in the areas of the fifth (trigeminal) cranial nerve. Ptosis and diplopia are associated with myasthenia gravis. Hyporeflexia and weakness of the lower extremities are associated with Guillain-Barre syndrome. Fatigue and depression are associated with multiple sclerosis.

A client with a spinal cord injury says he has difficulty recognizing the symptoms of urinary tract infection (UTI). Which symptom is an early sign of UTI in a client with a spinal cord injury? Lower back pain Frequency of urination Burning sensation on urination Fever and change in urine clarity

Correct response: Fever and change in urine clarity Explanation: Fever and change in urine clarity as early signs of UTI in a client with a spinal cord injury. Lower back pain is a late sign. A client with a spinal cord injury may not experience a burning sensation or urinary frequency.

A client has been diagnosed as having global aphasia. The nurse recognizes that the client will be unable to perform which action? Form words that are understandable Speak at all Form words that are understandable or comprehend spoken words Comprehend spoken words

Correct response: Form words that are understandable or comprehend spoken words Explanation: Global aphasia is a combination of expressive and receptive aphasia and presents a tremendous challenge to the nurse to effectively communicate with the client. In receptive and expressive aphasia, the client is unable to form words that are understandable. The client who is unable to speak at all is referred to as mute. Chapter 62: pp. 2032-2033.

Which lobe of the brain is responsible for concentration and abstract thought? Frontal Parietal Temporal Occipital

Correct response: Frontal Explanation: The major functions of the frontal lobe are concentration, abstract thought, information storage or memory, and motor function. The parietal lobe analyzes sensory information such as pressure, vibration, pain, and temperature. The occipital lobe is the primary visual cortex. The temporal lobe contains the auditory receptive areas located around the temples. p. 1968.

A client is demonstrating an altered level of consciousness from a traumatic brain injury. Which assessment will the nurse use as a sensitive indicator of neurologic function? Cranial nerve function Mental status evaluation Glasgow Coma Scale Cerebellar function

Correct response: Glasgow Coma Scale Explanation: An altered level of consciousness (LOC) is present when the client is not oriented, does not follow commands, or needs persistent stimuli to achieve a state of alertness. LOC is gauged on a continuum, with a normal state of alertness and full cognition (consciousness) on one end and coma on the other end. LOC, a sensitive indicator of neurologic function, is assessed based on the criteria in the Glasgow Coma Scale: eye opening, verbal response, and motor response. Cerebellar function, cranial nerve function, and mental status evaluation are all elements of the neurologic assessment. Chapter 61:, p. 9.

While riding a bicycle in a race, a patient fell into a ditch and sustained a head injury. Another cyclist found the patient lying unconscious in the ditch and called 911. What type of concussion does the patient most likely have? Grade 2 concussion Grade 3 concussion Grade 4 concussion Grade 1 concussion

Correct response: Grade 3 concussion Explanation: There are three grades of concussion or mild traumatic brain injury defined by the American Academy of Neurology when the injury is sports related. Grade 1 concussion has symptoms of transient confusion, no loss of consciousness, and duration of mental status abnormalities on examination that resolve in less than 15 minutes. Grade 2 concussion also has symptoms of transient confusion and no loss of consciousness, but the concussion symptoms or mental status abnormalities on examination last more than 15 minutes. Grade 3 concussion, there is any loss of consciousness lasting from seconds to minutes

A 37-year-old mother of three has just been diagnosed with a grade I meningioma. As part of patient education, the nurse tells the patient that: Surgery, which can result in complete removal of the possible tumor, should be done as soon as possible. The tumor is malignant and aggressive. Growth is slow and symptoms are caused by compression rather than tissue invasion. The tumor will cause pressure on the eighth cranial nerve.

Correct response: Growth is slow and symptoms are caused by compression rather than tissue invasion. Explanation: A meningioma is benign, encapsulated, and slow-growing. Sometimes the patient has no symptoms because of the slow-growing nature of the tumor.

A 37-year-old mother of three has just been diagnosed with a grade I meningioma. As part of patient education, the nurse tells the patient that: Growth is slow and symptoms are caused by compression rather than tissue invasion. Surgery, which can result in complete removal of the possible tumor, should be done as soon as possible. The tumor will cause pressure on the eighth cranial nerve. The tumor is malignant and aggressive.

Correct response: Growth is slow and symptoms are caused by compression rather than tissue invasion. Explanation: A meningioma is benign, encapsulated, and slow-growing. Sometimes the patient has no symptoms because of the slow-growing nature of the tumor.

The sympathetic and parasympathetic nervous systems have a direct effect on the circulatory system. Stimulation of the parasympathetic nervous system (PNS) causes which of the following? Blood vessels in the heart muscle to dilate Blood vessels in the skeletal muscles to dilate Heartbeat to decrease Blood pressure to increase

Correct response: Heartbeat to decrease Explanation: The parasympathetic nervous system has a constricting effect on the blood vessels in the heart and skeletal muscles; the heartbeat and blood pressure will decrease.

A client is admitted to undergo lumbar laminectomy for treatment of a herniated disk. Which action should the nurse take first to promote comfort preoperatively? Administer hydrocodone (Vicodin) as ordered. Notify the physician of the client's pain. Help the client assume a more comfortable position. Provide teaching on nonpharmacologic measures to control pain.

Correct response: Help the client assume a more comfortable position. Explanation: The nurse should first help the client assume a more comfortable position. After doing so, the nurse may administer pain medication as ordered. Next, the nurse should assess the client's knowledge of nonpharmacologic measures to relieve pain and provide teaching as necessary. If the client's pain isn't relieved after taking these actions, the nurse should notify the physician of the client's pain issues.

A client is experiencing muscle weakness and an ataxic gait. The client has a diagnosis of multiple sclerosis (MS). Based on these symptoms, the nurse formulates "Impaired physical mobility" as one of the nursing diagnoses applicable to the client. What nursing intervention should be most appropriate to address the nursing diagnosis? Change body position every 2 hours. Use a footboard and trochanter rolls. Use pressure-relieving devices when the client is in bed or in a wheelchair. Help the client perform range-of-motion (ROM) exercises every 8 hours.

Correct response: Help the client perform range-of-motion (ROM) exercises every 8 hours. Explanation: Helping the client perform ROM exercises every 8 hours helps in promoting joint flexibility and muscle tone in a client with muscle weakness. Measures such as using pressure-relieving devices or changing the body positions every 2 hours prevents skin breakdown. The nurse should use a footboard and trochanter rolls to promote a neutral body position that will keep the body in good alignment.

Which is the most common cause of acute encephalitis in the United States? Herpes simplex virus (HSV) Lyme Disease Human immunodeficiency virus (HIV) Western equine bacteria

Correct response: Herpes simplex virus (HSV) Explanation: HSV-1 ( herpes simplex virus) is the most common cause of acute encephalitis in the United States. Fungal infections of the central nervous system occur rarely in healthy people. The Western equine encephalitis virus is one of four types of arboviral encephalitis that occur in North America is one of several fungi that may cause fungal encephalitis. Lyme disease leads to flu like symptoms and starts as a local infection which can systematically spread causing organ issues, however the incidence is rate, HIV leads to autoimmune disorders. Chapter 64: p. 2092.

The nursing instructor gives their students an assignment of making a plan of care for a client with Huntington's disease. What would be important for the students to include in the teaching portion of the care plan? How to exercise How to perform household tasks How to take a bath How to facilitate tasks such as using both hands to hold a drinking glass

Correct response: How to facilitate tasks such as using both hands to hold a drinking glass Explanation: The nurse demonstrates how to facilitate tasks such as using both hands to hold a drinking glass, using a straw to drink, and wearing slip-on shoes. The teaching portion of the care plan would not include how to exercise, perform household tasks, or take a bath. Chapter 65: p. 2130.

Which disease is a chronic, progressive, hereditary disease of the nervous system that results in progressive, involuntary dancelike movements and dementia? Creutzfeldt-Jakob disease Huntington disease Multiple sclerosis Parkinson disease

Correct response: Huntington disease Explanation: Because it is transmitted as an autosomal dominant genetic disorder, each child of a parent with Huntington disease has a 50% risk of inheriting the illness. Multiple sclerosis is a chronic, degenerative, progressive disease of the central nervous system (CNS) characterized by the occurrence of small patches of demyelination in the brain and spinal cord. Parkinson disease is associated with decreased levels of dopamine due to destruction of pigmented neuronal cells in the substantia nigra in the basal ganglia of the brain. Creutzfeldt-Jakob disease is a rare, transmissible, progressive and fatal disease of the CNS characterized by spongiform degeneration of the gray matter of the brain.

The nurse assesses initial skin redness in a patient who is at risk for skin breakdown. How should the nurse document this finding? Ischemia Eschar Hyperemia Anoxia

Correct response: Hyperemia Explanation: The initial sign of pressure is erythema (redness of the skin) caused by reactive hyperemia, which normally resolves in less than 1 hour. Unrelieved pressure results in tissue ischemia or anoxia. Eschar is a dry scab that forms over a healing ulcer.

Following a transsphenoidal hypophysectomy, a nurse should assess a client for which condition? Hypercalcemia Hypocortisolism Hyperglycemia Hypoglycemia

Correct response: Hypocortisolism Explanation: Although steroids should be given during surgery to prevent hypocortisolism, the nurse should assess the client for it. Abrupt withdrawal of endogenous cortisol may lead to severe adrenal insufficiency. Signs of hypocortisolism include vomiting, increased weakness, dehydration, and hypotension. After the corticotropin-secreting tumor is removed, the client shouldn't be at risk for hyperglycemia. Calcium imbalance and hypoglycemia shouldn't occur in this situation.

The nurse is caring for a client with a ventriculostomy. Which assessment finding demonstrates effectiveness of the ventriculostomy? Cerebral perfusion pressure (CPP) is 21 mm Hg. Increased ICP is 12 mm Hg. The pupils are dilated and fixed. The mean arterial pressure (MAP) is equal to the intracranial pressure (ICP).

Correct response: Increased ICP is 12 mm Hg. Explanation: A ventriculostomy is used to continuously measure ICP and allows cerebral spinal fluid to drain, especially during a period of increased ICP. The normal ICP is 0 to 15 mm Hg, so ICP measured at 12 mm Hg would demonstrate the effectiveness of the ventriculostomy. Dilated and fixed pupils are not a normal assessment finding and would not indicate an improvement in the neurologic system. Cerebral circulation ceases if the ICP is equal to the MAP. Normal CPP is 70 to 100. A CPP reading less than 50 is consistent with irreversible neurologic damage.

The nurse is caring for a client with Guillain-Barré syndrome. Which assessment finding would indicate the need for oral suctioning? Increased pulse rate, adventitious breath sounds Decreased pulse rate, abdominal breathing Decreased pulse rate, respirations of 20 breaths/minute Increased pulse rate, respirations of 16 breaths/minute

Correct response: Increased pulse rate, adventitious breath sounds Explanation: An increased pulse rate above baseline with adventitious breath sounds indicate compromised respirations and signal a need for airway clearance. A decrease in pulse rate is not indicative of airway obstruction. An increase of pulse rate with slight elevation of respirations (16 breaths/minute) is not significant for suctioning unless findings suggest otherwise.

Which nursing diagnosis takes the highest priority for a client with parkinsonian crisis? Impaired urinary elimination Ineffective airway clearance Risk for injury Imbalanced nutrition: Less than body requirements

Correct response: Ineffective airway clearance Explanation: In parkinsonian crisis, dopamine-related symptoms are severely exacerbated, virtually immobilizing the client. A client confined to bed during such a crisis is at risk for aspiration and pneumonia. Also, excessive drooling increases the risk of airway obstruction. Because of these concerns, the nursing diagnosis of Ineffective airway clearance takes the highest priority. Although Imbalanced nutrition: Less than body requirements, Impaired urinary elimination, and Risk for injury are also appropriate nursing diagnoses, they aren't immediately life-threatening.

A client has been brought to the ED with altered LOC, high fever, and a purpura rash on the lower extremities. The family states the client was reporting neck stiffness earlier in the day. What action should the nurse do first? Initiate isolation precautions. Apply a cooling blanket. Administer prescribed antibiotics. Ensure the family receives prophylaxis antibiotic treatment.

Correct response: Initiate isolation precautions. Explanation: The signs and symptoms are consistent with bacterial meningitis. The nurse should protect self, other health care workers, and other clients against the spread of the bacteria. Clients should receive the prescribed antibiotics within 30 minutes of arrival, but the nurse can administer the antibiotics after applying the isolation precautions. The nurse can use a cooling blanket to help with the elevated temperature, but this should be done after applying isolation precautions. Prophylaxis antibiotic therapy should be given to people who were in close contact with the patient, but this is not the highest priority nursing intervention.

The nurse working on a neurological unit is mentoring a nursing student. The student asks about a client who has sustained a primary and secondary brain injury. The nurse correctly tells the student which of the following, related to the primary injury? It refers to the difficulties suffered by the client and family related to the changes in the client. It results from inadequate delivery of nutrients and oxygen to the cells. It refers to the permanent deficits seen after the rehabilitation process. It results from initial damage to the brain from the traumatic event.

Correct response: It results from initial damage to the brain from the traumatic event. Explanation: The primary injury results from the initial damage from the traumatic event. The secondary injury results from inadequate delivery of nutrients and oxygen to the cells, usually due to cerebral edema and increased intracranial pressure. Chapter 63: Page 2056

A client with epilepsy is having a seizure. What intervention should the nurse do after the seizure? Keep the client on one side. Pry the client's mouth open to allow a patent airway. Place a cooling blanket beneath the client. Help the client sit up.

Correct response: Keep the client on one side. Explanation: The nurse will need to keep the client on one side to prevent aspiration. Make sure the airway is patent. On awakening, reorient the client to the environment. If the client is confused or wandering, guide the client gently to a bed or chair. If the client becomes agitated after a seizure (postictal), stay a distance away, but close enough to prevent injury until the client is fully aware. The client does not need a cooling blanket after a seizure. The client's temperature should not be elevated from the seizure. The nurse should not pry the client's mouth open after a seizure so that the airway remains open.

The nurse is called to attend to a patient having a seizure in the waiting area. What nursing care is provided for a patient who is experiencing a convulsive seizure? Select all that apply. Restraining the patient to avoid self injury Opening the patient's jaw and inserting a mouth gag Positioning the patient on his or her side with head flexed forward Providing for privacy Loosening constrictive clothing

Correct response: Loosening constrictive clothing Positioning the patient on his or her side with head flexed forward Providing for privacy Explanation: During a patient's seizure, the nurse should do the following. Loosen constrictive clothing. If possible, place the patient on one side with head flexed forward, which allows the tongue to fall forward and facilitates drainage of saliva and mucus. If suction is available, use it if necessary to clear secretions. Provide privacy, and protect the patient from curious onlookers. (The patient who has an aura [warning of an impending seizure] may have time to seek a safe, private place.) The nurse should not attempt to pry open jaws that are clenched in a spasm or attempt to insert anything. Broken teeth and injury to the lips and tongue may result from such an action. No attempt should be made to restrain the patient during the seizure, because muscular contractions are strong and restraint can produce injury.

The diagnosis of multiple sclerosis is based on which test? Cerebrospinal fluid (CSF) electrophoresis Magnetic resonance imaging (MRI) Evoked potential studies Neuropsychological testing

Correct response: Magnetic resonance imaging (MRI) Explanation: The diagnosis of MS is based on the presence of multiple plaques in the central nervous system observed on MRI. Electrophoresis of CSF identifies the presence of oligoclonal banding. Evoked potential studies can help define the extent of the disease process and monitor changes. Neuropsychological testing may be indicated to assess cognitive impairment. Chapter 64: pp. 2094-2095.

The most important nursing priority of treatment for a patient with an altered LOC is to: Prevent dehydration and renal failure by inserting an IV line for fluids and medications. Maintain a clear airway to ensure adequate ventilation. Position the patient to prevent injury and ensure dignity. Stabilize the blood pressure and heart rate to ensure adequate perfusion of the brain.

Correct response: Maintain a clear airway to ensure adequate ventilation. Explanation: The first priority of treatment for the patient with altered LOC is to obtain and maintain a patent airway. The patient may be orally or nasally intubated (unless basilar skull fracture or facial trauma is suspected), or a tracheostomy may be performed. Until the ability of the patient to breathe on his or her own is determined, a mechanical ventilator is used to maintain adequate oxygenation and ventilation. Chapter 63: p. 2064.

The nurse in the neurologic ICU is caring for a client who sustained a severe brain injury. Which nursing measures will the nurse implement to help control intracranial pressure (ICP)? Position the client in the supine position Maintain cerebral perfusion pressure from 50 to 70 mm Hg Administer enemas, as needed Restrain the client, as indicated

Correct response: Maintain cerebral perfusion pressure from 50 to 70 mm Hg Explanation: The nurse should maintain cerebral perfusion pressure from 50 to 70 mm Hg to help control increased ICP. Other measures include elevating the head of the bed as prescribed, maintaining the client's head and neck in neutral alignment (no twisting or flexing the neck), initiating measures to prevent the Valsalva maneuver (e.g., stool softeners), maintaining body temperature within normal limits, administering O2 to maintain PaO2 greater than 90 mm Hg, maintaining fluid balance with normal saline solution, avoiding noxious stimuli (e.g., excessive suctioning, painful procedures), and administering sedation to reduce agitation.

The nurse is performing the physical examination of a client with a suspected neurologic disorder. In addition to assessing other parts of the body, the nurse should assess for neck rigidity. Which method should help the nurse assess for neck rigidity correctly? Moving the head and chin toward the chest Gently pressing the bones on the neck Moving the head toward both sides Lightly tapping the lower portion of the neck to detect sensation

Correct response: Moving the head and chin toward the chest Explanation: The neck is examined for stiffness or abnormal position. The presence of rigidity is assessed by moving the head and chin toward the chest. The nurse should not maneuver the neck if a head or neck injury is suspected or known. The neck should also not be maneuvered if trauma to any part of the body is evident. Moving the head toward the sides or pressing the bones on the neck will not help assess for neck rigidity correctly. While assessing for neck rigidity, sensation at the neck area is not to be assessed

A client is admitted to an acute care facility for treatment of a brain tumor. When reviewing the chart, the nurse notes that the client's extremity muscle strength is rated 1/5. Which assessment finding should the nurse anticipate? Muscle contraction is palpable and visible. Muscle contraction or movement is undetectable. Muscles move actively against gravity alone. Normal, full muscle strength is present.

Correct response: Muscle contraction is palpable and visible. Explanation: Muscle strength is assessed and rated on a five-point scale in all four extremities, comparing one side to the other. Palpable, visible muscle contraction on the affected side and normal, full muscle strength on the unaffected side indicate a rating of 1/5. Normal, full muscle strength on both sides is rated 5/5. Active muscle movement against gravity alone on the affected side with normal, full muscle strength on the unaffected side is rated 3/5. Undetectable muscle contraction or movement on the affected side with normal, full muscle strength on the unaffected side is rated 0/5.

Which of the following is the initial diagnostic in suspected stroke? Magnetic resonance imaging (MRI) Noncontrast computed tomography (CT) CT with contrast Cerebral angiography

Correct response: Noncontrast computed tomography (CT) Explanation: An initial head CT scan will determine whether or not the patient is experiencing a hemorrhagic stroke. An ischemic infarction will not be readily visible on initial CT scan if it is performed within the first few hours after symptoms onset; however, evidence of bleeding will almost always be visible.

A client newly diagnosed with Huntington diease asks for information concerning management of symptoms. Which action would the nurse first take to address this request? Provide a referral for the client to a Huntington disease multidisciplnary team. Recommend the immediate use of levodopa by the client. Perform a focused assessment on the client's needs and capabilities. Establish the client's willingness to adhere to prescribed treatments.

Correct response: Perform a focused assessment on the client's needs and capabilities. Explanation: Nurses must look beyond the disease to focus on the patient's needs and capabilities first. While the client will benefit from the expertise of a Huntington disease multidisciplinary team, it is important to first establish their goals and individual needs. Only after a full assessment should medications be recommended by anyone on a health care team. In clients who present with rigidity, some temporary benefit may be obtained from antiparkinson medications, such as levodopa. Establishing a client's willingness to adhere to treatments is premature until the treatment plan is established. Huntington disease is characterized by a triad of symptoms that include motor dysfunction (the most prominent being chorea), impaired cognition, and behavioral features such as a blunted affect. Huntington disease is a progressive and terminal illness at this time. The focus for this client is optimizing quality of life with available medications and supportive treatments.

A nurse conducts the Romberg test on a client by asking the client to stand with the feet close together and the eyes closed. As a result of this posture, the client suddenly sways to one side and is about to fall when the nurse intervenes and saves the client from being injured. How should the nurse interpret the client's result? Positive Romberg test, indicating a problem with equilibrium Negative Romberg test, indicating a problem with body mass Negative Romberg test, indicating a problem with vision Positive Romberg test, indicating a problem with level of consciousness

Correct response: Positive Romberg test, indicating a problem with equilibrium Explanation: If the client sways and starts to fall during the Romberg test, it indicates a positive result. This means the client has a problem with equilibrium. The Romberg test is used to assess the client's motor function, including muscle movement, size, tone, strength, and coordination.

A health care provider needs help in identifying the precise location of a brain tumor. To measure brain activity, as well as to determine structure, the nurse expects the health care provider to order which of the following tests? MRI Positron-emission tomography (PET) Computed tomography (CT) Computer-assisted stereotactic biopsy

Correct response: Positron-emission tomography (PET) Explanation: A PET scan is most diagnostic for brain activity, as well as for assessment of tumor size. It can also be useful in differentiating a tumor from scar tissue or radiation necrosis.

How can the nurse prevent continuous moisture on the skin of a patient who is at risk for developing skin breakdown? Practice meticulous hygiene measures. Place an indwelling catheter in the patient. Administer vitamin B12 to the patient. Apply powder.

Correct response: Practice meticulous hygiene measures. Explanation: Continuous moisture on the skin must be prevented by meticulous hygiene measures. It is important to pay special attention to skin folds, including areas under the breasts, arms, and groin, and between the toes. Perspiration, urine, stool, and drainage must be removed from the skin promptly. soiled skin should be washed immediately with mild soap and water and blotted dry with a soft towel. The skin may be lubricated with a bland lotion to keep it soft and pliable. Drying agents and powders are avoided. Topical barrier ointments (e.g., petroleum jelly) may be helpful in protecting the skin of patients who are incontinent. Placing an indwelling catheter and administering vitamin B12 would not be effective measures in preventing continuous moisture.

A patient sustained a head injury and has been admitted to the neurosurgical intensive care unit (ICU). The patient began having seizures and was administered a sedative-hypnotic medication that is ultra-short acting and can be titrated to patient response. What medication will the nurse be monitoring during this time? Lorazepam (Ativan) Phenobarbital Propofol (Diprivan) Midazolam (Versed)

Correct response: Propofol (Diprivan) Explanation: If the patient is very agitated, benzodiazepines are the most commonly used sedative agents and do not affect cerebral blood flow or ICP. Propofol (Diprivan), on the other hand, a sedative-hypnotic agent that is supplied in an intralipid emulsion for intravenous (IV) use, is the sedative of choice. It is an ultra-short acting, rapid onset drug with elimination half-life of less than an hour. It has a major advantage of being titratable to its desired clinical effect but still provides the opportunity for an accurate neurologic assessment Lorazepam (Ativan) and midazolam (Versed) are frequently used but have active metabolites that my cause prolonged sedation, making it difficult to conduct a neurologic assessment.

Which is a component of the nursing management of the client with new variant Creutzfeldt-Jakob disease (vCJD)? Administering amphotericin B Initiating isolation procedures Providing supportive care Preparing for organ donation

Correct response: Providing supportive care Explanation: vCJD is a progressive fatal disease, and no treatment is available. Because of the fatal outcome of vCJD, nursing care is primarily supportive and palliative. No treatment is available for vCJD. Prevention of disease transmission is an important part of providing nursing care. Although client isolation is not necessary, use of standard precautions is important. Institutional protocols are followed for blood and body fluid exposure and decontamination of equipment. Organ donation is not an option because of the risk for disease transmission. Amphotericin B is used in the treatment of fungal encephalitis.

The nurse is caring for a client immediately after a spinal cord injury. Which assessment finding is essential when caring for a client in spinal shock with injury in the lower thoracic region? Numbness and tingling Pain level Pulse and blood pressure Respiratory pattern

Correct response: Pulse and blood pressure Explanation: Spinal shock is a loss of sympathetic reflex activity below the level of the injury within 30 to 60 minutes after insult. In addition to the paralysis, manifestations include pronounced hypotension, bradycardia, and warm, dry skin. Numbness and tingling and pain are not as high of a concern at this time due to the cord injury. Because the level of impairment is below the first thoracic vertebrae, respiratory failure is not a concern.

A client with a T4-level spinal cord injury (SCI) is experiencing autonomic dysreflexia; his blood pressure is 230/110. The nurse cannot locate the cause and administers antihypertensive medication as ordered. The nurse empties the client's bladder and the symptoms abate. Now, what must the nurse watch for? Spinal shock Urinary tract infection Rebound hypertension Rebound hypotension

Correct response: Rebound hypotension Explanation: When the cause is removed and the symptoms abate, the blood pressure goes down. The antihypertensive medication is still working. The nurse must watch for rebound hypotension. Rebound hypertension is not an issue. Spinal shock occurs right after the initial injury. The client is not at any more risk for a urinary tract infection after the episode than he was before. Chapter 63: p. 2078.

A nurse is developing a plan of care for an 85-year-old woman who is bedridden following a stroke. Which of the following would the nurse be least likely to include in the plan of care for this patient to reduce her risk for pressure ulcers? Repositioning the patient about once a shift Lubricating the skin with a non-irritating lotion Using a static support device on the patient's bed Lifting rather than sliding the patient when repositioning her

Correct response: Repositioning the patient about once a shift Explanation: Turning should occur every 1 to 2 hours — not once a shift — for patients who are in bed for prolonged periods. The nurse should apply a non-irritating lotion, use static support devices to relieve pressure, and lift rather than slide the patient when repositioning to reduce shearing forces.

What nursing intervention is appropriate for a client with receptive aphasia? Frequently reorient the client to time, place, and situation. Encourage the client to repeat sounds of the alphabet. Explore the client's ability to write. Speak slowly and clearly.

Correct response: Speak slowly and clearly. Explanation: Nursing management of the client with receptive aphasia includes speaking slowing and clearly to assist the client in forming the sounds. Nursing management of the client with expressive aphasia includes encouraging the client to repeat sounds of the alphabet or to explore the client's ability to write. Nursing management of the client with cognitive deficits, such as memory loss, includes frequently reorienting the client to time, place, and situation.

What nursing intervention is appropriate for a client with receptive aphasia? Speak slowly and clearly. Frequently reorient the client to time, place, and situation. Explore the client's ability to write. Encourage the client to repeat sounds of the alphabet.

Correct response: Speak slowly and clearly. Explanation: Nursing management of the client with receptive aphasia includes speaking slowing and clearly to assist the client in forming the sounds. Nursing management of the client with expressive aphasia includes encouraging the client to repeat sounds of the alphabet or to explore the client's ability to write. Nursing management of the client with cognitive deficits, such as memory loss, includes frequently reorienting the client to time, place, and situation.

Guillain-Barré syndrome is an autoimmune attack on the peripheral myelin sheath. Which of the following is an action of myelin? Acts as chemical messenger Represents building block of nervous system Carries message to the next nerve cell Speeds nerve impulse transmission

Correct response: Speeds nerve impulse transmission Explanation: Myelin is a complex substance that covers nerves, providing insulation and speeding the conduction of impulses from the cell body to the dendrites. The axon carries the message to the next nerve cell. The neuron is the building block of the nervous system. A neurotransmitter is a chemical messenger.

Guillain-Barré syndrome is an autoimmune attack on the peripheral myelin sheath. Which of the following is an action of myelin? Acts as chemical messenger Speeds nerve impulse transmission Carries message to the next nerve cell Represents building block of nervous system

Correct response: Speeds nerve impulse transmission Explanation: Myelin is a complex substance that covers nerves, providing insulation and speeding the conduction of impulses from the cell body to the dendrites. The axon carries the message to the next nerve cell. The neuron is the building block of the nervous system. A neurotransmitter is a chemical messenger.

A patient with Bell's palsy says to the nurse, "It doesn't hurt anymore to touch my face. How am I going to get muscle tone back so I don't look like this anymore?" What interventions can the nurse suggest to the patient? Tell the patient to smile every 4 hours. Suggest massaging the face several times daily, using a gentle upward motion, to maintain muscle tone. Suggest applying cool compresses on the face several times a day to tighten the muscles. Inform the patient that the muscle function will return as soon as the virus dissipates.

Correct response: Suggest massaging the face several times daily, using a gentle upward motion, to maintain muscle tone. Explanation: After the sensitivity of the nerve to touch decreases and the patient can tolerate touching the face, the nurse can suggest massaging the face several times daily, using a gentle upward motion, to maintain muscle tone. Facial exercises, such as wrinkling the forehead, blowing out the cheeks, and whistling, may be performed with the aid of a mirror to prevent muscle atrophy. Exposure of the face to cold and drafts is avoided. Chapter 64: p. 2110.

The nurse is providing discharge instructions for a client who was admitted to the oncology unit due to dehydration and anorexia after chemotherapy treatment. What information should the nurse provide to the client to promote improve the client's nutritional intake at home? Avoid any oral care prior to eating Eat uninterrupted by others to eliminate distractions Prepare the eating area with a pleasant room spray Take prescribed pain medication prior to commencing a meal

Correct response: Take prescribed pain medication prior to commencing a meal Explanation: The client needs to be clean, comfortable, and free of pain for meals, in an environment that is as attractive as possible. Ensuring adequate pain relief in advance of commencing a meal will make the experience more pleasant and tolerable. Pain is correlated with lack of appetite.

An adult client's current goals of rehabilitation focus primarily on self-care. What is a priority when teaching a client who has self-care deficits in ADLs? To ensure that the client is able to perform self-care without any aid from caregivers To provide an optimal learning environment with minimal distractions To help the client become aware of the requirements of assisted-living centers To describe the evidence base for any chosen interventions

Correct response: To provide an optimal learning environment with minimal distractions Explanation: The nurse's role is to provide an optimal learning environment that minimizes distractions. Describing the evidence base is not a priority, though nursing actions should indeed be evidence based. Assisted living facilities are not relevant to most clients. Absolute independence in ADLs is not an appropriate goal for every client.

A client with a malignant brain tumor comes to the clinic for a follow up. During the visit, the client asks the nurse, "Why am I so tired all the time?" When responding to the client, which information would the nurse include as possible causes? Select all that apply. Metastasis Tumor Treatment being used Effects of increased intracranial pressure Stress

Correct response: Tumor Treatment being used Stress Explanation: Fatigue is a symptom experienced by clients with both malignant and nonmalignant brain tumors. Etiology of fatigue can be multifactorial. The tumor itself, surgery, medications, chemotherapy, and radiation may all contribute to increased fatigue. Clients may report a constant feeling of exhaustion, weakness, and lack of energy. It is also important to identify underlying conditions, such as stress, anxiety, and depression, which may play a role in fatigue. Metastasis and increased intracranial pressure are not usually associated with fatigue. Chapter 65: p. 2118.

A client diagnosed with Huntington disease is on a disease-modifying drug regimen and has a urinary catheter in place. Which potential complication is the highest priority for the nurse while monitoring the client? Severe depression Choreiform movements Emotional apathy Urinary tract infection

Correct response: Urinary tract infection Explanation: Because all disease-modifying drug regimens for Huntington disease can decrease immune cells and infection protection, it is most important for the nurse to assess for acquired infections such as urinary tract infections, especially if the client is catheterized. Severe depression is common and can lead to suicide. Symptoms of Huntington disease develop slowly and include mental apathy and emotional disturbances, choreiform movements (uncontrollable writhing and twisting of the body), grimacing, difficulty chewing and swallowing, speech difficulty, intellectual decline, and loss of bowel and bladder control. Assessing for these other conditions is appropriate but not as important as assessing for urinary tract infection in the client on a disease-modifying drug regimen with a urinary catheter in place.

The nurse is providing discharge teaching to a client who has had surgery for partial removal of a spinal tumor to decompress the spinal cord. Preoperatively, the client had lost sensation to the lower legs. When instructing the client regarding pain management strategies, the nurse should include which information? Use assistive devices Sleep in the recumbent position Use heat to decrease back pain Perform moderate exercise

Correct response: Use assistive devices Explanation: To prevent falls and pain due to fatigue and overuse, the client should be encouraged to use assistive devices such as canes, walkers and/or wheelchairs when ambulating. When the nurse is providing discharge teaching to a client after spinal surgery, the nurse should ensure that, for a client with residual sensory involvement, the client is aware that extreme temperatures should not be applied to the skin. The client should be alerted to the dangers of heating devices (e.g., hot water bottles, heating pads, space heaters). Sleeping in the recumbent position (three quarters prone) can increase pain. Thus, the client should be encouraged to sleep flat with the head of the bed slightly elevated or closely follow instructions for sleep position provided by the surgeon. Although maintaining muscle strength is important in promoting ability to carry out activities of daily living, moderate exercise may not be possible. The client should follow the rehabilitation plan prescribed by the allied health professional responsible for this aspect of the client's care (e.g., the physiotherapist).

A nurse is performing a neurologic assessment on the client and notes a positive Romberg test. This test for balance is related to which of the following cranial nerves? X VII VIII III

Correct response: VIII Explanation: CN VIII is the acoustic nerve. It has to do with hearing, air and bone conduction, and balance. CN X is the vagus nerve and has to do with the gag reflex, laryngeal hoarseness, swallowing ability, and the symmetrical rise of the uvula and soft palate. CN III is the oculomotor and has to do with pupillary response, conjugate movements, and nystagmus. CN VII is the facial nerve and has to do with symmetry of facial movements and the ability to discriminate between the taste of sugar and salt.

A nurse is performing a neurologic assessment on the client and notes a positive Romberg test. This test for balance is related to which of the following cranial nerves? III X VIII VII

Correct response: VIII Explanation: CN VIII is the acoustic nerve. It has to do with hearing, air and bone conduction, and balance. CN X is the vagus nerve and has to do with the gag reflex, laryngeal hoarseness, swallowing ability, and the symmetrical rise of the uvula and soft palate. CN III is the oculomotor and has to do with pupillary response, conjugate movements, and nystagmus. CN VII is the facial nerve and has to do with symmetry of facial movements and the ability to discriminate between the taste of sugar and salt.

A nurse is caring for a client with a history of severe migraines. The client has a medical history that includes asthma, gastroesophageal reflux disease, and three pregnancies. Which medication does the nurse anticipate the physician will order for the client's migraines? Amiodarone (Cordarone) Verapamil (Calan) Metoprolol (Lopressor) Carvedilol (Coreg)

Correct response: Verapamil (Calan) Explanation: Calcium channel blockers, such as verapamil, and beta-adrenergic blockers, such as metoprolol, are commonly used to treat migraines because they help control cerebral blood vessel dilation. Calcium channel blockers, however, are ordered for clients who may not be able to tolerate beta-adrenergic blockers, such as those with asthma. Amiodarone and carvedilol aren't used to treat migraines.

The nurse is providing information about strokes to a community group. Which of the following would the nurse identify as the primary initial symptoms of an ischemic stroke? Vomiting and seizures Severe headache and early change in level of consciousness Foot drop and external hip rotation Weakness on one side of the body and difficulty with speech

Correct response: Weakness on one side of the body and difficulty with speech Explanation: The main presenting symptoms for an ischemic stroke are numbness or weakness of the face, arm, or leg, especially on one side of the body; confusion or change in mental status; and trouble speaking or understanding speech. Severe headache, vomiting, early change in level of consciousness, and seizures are early signs of a hemorrhagic stroke. Foot drop and external hip rotation are things that can occur if a stroke victim is not turned or positioned correctly.

The nurse is performing a neurologic assessment on a client diagnosed with a stroke and cannot elicit a gag reflex. This deficit is related to which of the following cranial nerves? VIII VII III X

Correct response: X Explanation: CN X is the vagus nerve and has to do with the gag reflex, laryngeal hoarseness, swallowing ability, and the symmetrical rise of the uvula and soft palate. CN VII is the facial nerve and has to do with symmetry of facial movements and the ability to discriminate between the tastes of sugar and salt. The inability to close one eyelid indicates impairment of this nerve. CN VIII is the acoustic nerve. It has to do with hearing, air and bone conduction, and balance. CN III is the oculomotor nerve and has to do with pupillary response, conjugate movements, and nystagmus.

The nurse has documented a client diagnosed with a head injury as having a Glasgow Coma Scale (GCS) score of 7. This score is generally interpreted as most responsive. minimally responsive. least responsive. coma.

Correct response: coma. Explanation: The GCS is a tool for assessing a client's response to stimuli. A score of 7 or less is generally interpreted as a coma. The lowest score is 3 (least responsive/deep coma); the highest is 15 (most responsive). A GCS between 3 and 8 is generally accepted as indicating a severe head injury. No category is termed "least" responsive.

When the nurse observes that the client has extension and external rotation of the arms and wrists and plantar flexion of the feet, the nurse records the client's posture as decorticate. decerebrate. flaccid. normal.

Correct response: decerebrate. Explanation: Decerebrate posturing is the result of lesions at the midbrain and is more ominous than decorticate posturing. The client's head and neck arch backward, and the muscles are rigid. In decorticate posturing, which results from damage to the nerve pathway between the brain and spinal cord and is also very serious, the client has flexion and internal rotation of the arms and wrists, as well as extension, internal rotation, and plantar flexion of the feet.

An osmotic diuretic such as mannitol is given to the client with increased intracranial pressure (ICP) to reduce cellular metabolic demand. dehydrate the brain and reduce cerebral edema. control fever. control shivering.

Correct response: dehydrate the brain and reduce cerebral edema. Explanation: Osmotic diuretics draw water across intact membranes, thereby reducing the volume of brain and extracellular fluid. Antipyretics and a cooling blanket are used to control fever in the client with increased ICP. Chlorpromazine may be prescribed to control shivering in the client with increased ICP. Medications such as barbiturates are given to the client with increased ICP to reduce cellular metabolic demands. Chapter 61: p. 2003.

While repositioning an immobile client, a nurse notes that the client's sacral region is warm and red. Further assessment confirms that the skin is intact. Based on these findings, it's most appropriate for the nurse to: give the client a donut ring to reduce pressure on the affected area. do nothing; the client's skin is intact. document the condition of the client's skin. contact the client's family.

Correct response: document the condition of the client's skin. Explanation: The client's warm, red skin is consistent with a stage I pressure ulcer. Documenting the findings will provide a permanent record of the condition. If the nurse fails to take action, the client may experience further skin trauma. Donut rings reduce circulation to the sacral area when the client sits on them; they're contraindicated in this instance. There's no reason for the nurse to contact the client's family at this time; doing so might violate the client's right of privacy.

A nurse observes that decerebrate posturing is a comatose client's response to painful stimuli. Decerebrate posturing as a response to pain indicates: dysfunction in the brain stem. risk for increased intracranial pressure. dysfunction in the cerebrum. dysfunction in the spinal column.

Correct response: dysfunction in the brain stem. xplanation: Decerebrate posturing indicates damage of the upper brain stem. Decorticate posturing indicates cerebral dysfunction. Increased intracranial pressure is a cause of decortication and decerebration. Alterations in sensation or paralysis indicate dysfunction in the spinal column.

After a plane crash, a client is brought to the emergency department with severe burns and respiratory difficulty. The nurse helps to secure a patent airway and attends to the client's immediate needs, then prepares to perform an initial neurologic assessment. The nurse should perform an: evaluation of the corneal reflex response. evaluation of bowel and bladder functions. examination of the fundus of the eye. assessment of the client's gait.

Correct response: evaluation of the corneal reflex response. Explanation: During an acute crisis, the nurse should check the corneal reflex response to rapidly assess brain stem function. Other components of the brief initial neurologic assessment usually include level of consciousness, pupillary response, and motor response in the arms and legs. If appropriate and if time permits, the nurse also may assess sensory responses of the arms and legs. Emergency assessment doesn't include fundus examination unless the client has sustained direct eye trauma. The client shouldn't be moved unnecessarily until the extent of injuries is known, making gait evaluation impossible. Bowel and bladder functions aren't vital, so the nurse should delay their assessment. p. 1984.

Three hours after injuring the spinal cord at the C6 level, a client receives high doses of methylprednisolone sodium succinate (Solu-Medrol) to suppress breakdown of the neurologic tissue membrane at the injury site. To help prevent adverse effects of this drug, the nurse expects the physician to order: famotidine (Pepcid). naloxone (Narcan). nitroglycerin (Nitro-Bid). atracurium (Tracrium).

Correct response: famotidine (Pepcid). Explanation: Adverse effects of methylprednisolone sodium succinate and other steroids include GI bleeding and wound infection. To help prevent GI bleeding, the physician is likely to order an antacid or a histamine2-receptor antagonist such as famotidine (Pepcid). Naloxone, nitroglycerin, and atracurium aren't used to prevent adverse effects of steroids. Naloxone, an endogenous opioid antagonist, has been studied in animals for its action in inhibiting release of endogenous opioids after spinal cord injury. (Endogenous opioids are thought to contribute to secondary damage to spinal cord tissue by reducing microcirculatory blood flow.) Nitroglycerin is used to dilate the coronary arteries. Atracurium is a nondepolarizing muscle relaxant.

The nurse is caring for a comatose client. The nurse knows she should assess the client's motor response. Which method may the nurse use to assess the motor response? observing the reaction of pupils to light using the Romberg test assessing the client's sensitivity to temperature, touch, and pain observing the client's response to painful stimulus

Correct response: observing the client's response to painful stimulus Explanation: The nurse evaluates motor response in a comatose or unconscious client by administering a painful stimulus. This action helps determine if the client makes an appropriate response by reaching toward or withdrawing from the stimulus. The Romberg test is used to assess equilibrium in a noncomatose client. Pupils are examined for their reaction to light to assess sensitivity in the third cranial (oculomotor) nerve. Sensitivity to temperature, touch, and pain is a test to assess the sensory function of the client and not motor response.

A client with epilepsy is having a seizure. During the active seizure phase, the nurse should: place the client on his side, remove dangerous objects, and protect his head. place the client on his side, remove dangerous objects, and insert a bite block. place the client on his back, remove dangerous objects, and hold down his arms. place the client on his back, remove dangerous objects, and insert a bite block.

Correct response: place the client on his side, remove dangerous objects, and protect his head. Explanation: During the active seizure phase, the nurse should initiate precautions by placing the client on his side, removing dangerous objects, and protecting his head from injury. A bite block should never be inserted during the active seizure phase. Insertion can break the teeth and lead to aspiration. Placing the client on his back and holding down the arms could cause injury to the client and the nurse.

A nurse in a rehabilitation facility is coordinating the discharge of a client who is tetraplegic. The client, who is married and has two children in high school, is being discharged to home and will require much assistance. Who would the discharge planner recognize as being the most important member of this client's care team? home care nurse chaplain spouse physical therapist

Correct response: spouse Explanation: The client's spouse and family would need to be involved in the everyday care of the client; without their support, it is unlikely that the client would be able to manage at home.

A client with hypercholesterolemia is receiving Lipitor (atorvastatin) to prevent high cholesterol and stroke. The order is for Lipitor 40 mg PO daily. The medication is supplied in 80 mg tabs. How many tabs will the nurse administer to the client? Enter the correct number ONLY. tabs

Correct response: 0.5 Explanation: 40 mg/80 mg = 0.5 tabs.

Cerebral edema peaks at which time point after intracranial surgery? 12 hours 24 hours 48 hours 72 hours

Correct response: 24 hours Explanation: Cerebral edema tends to peak 24 to 36 hours after surgery.

If warfarin is contraindicated as a treatment for stroke, which medication is the best option? Ticlodipine Dipyridamole Clopidogrel Aspirin

Correct response: Aspirin Explanation: If warfarin is contraindicated, aspirin is the best option, although other medications may be used if both are contraindicated.

Which of the following types of skull fractures may be evident by Battle's sign? Depressed Simple Comminuted Basilar

Correct response: Basilar Explanation: A clinical manifestation of a basilar skull fracture is the Battle's sign (an area of ecchymosis may be seen over the mastoid). A simple (linear) fracture is a break in continuity of the bone. A comminuted skull fracture refers to a splintered fracture line. When bone fragments are embedded into the brain tissue, the fracture is depressed. Chapter 63:p. 2057.

A patient is admitted to the hospital for management of an extrapyramidal disorder. Included in the physician's admitting orders are the medications levodopa, benztropine, and selegiline. The nurse knows that most likely, the client has a diagnosis of: multiple sclerosis. seizure disorder. Parkinson's disease. Huntington's disease.

Correct response: Parkinson's disease. Explanation: Although antiparkinson drugs are used in some clients with Huntington's disease, these drugs are most commonly used in the medical management of Parkinson's disease. The listed medications are not used to treat a seizure disorder. The listed medications are not used to treat MS.

The nurse educator is teaching nursing students about various types of brain tumors. The instructor recognizes that teaching has been effective when students correctly identify a client whose lab work indicates excessively high levels of thyroid stimulating hormone would most likely be diagnosed with which type of tumor? Neuroma Angioma Glioblastoma Pituitary adenoma

Correct response: Pituitary adenoma Explanation: Pituitary adenomas can increased production of several hormones including TSH, ACTH, growth hormone and prolactin. Excessive hormone production is not characteristic of the brain tumors identified in the alternate options.

A client who is disoriented and restless after sustaining a concussion during a car accident is admitted to the hospital. Which nursing diagnosis takes the highest priority in this client's care plan? Impaired verbal communication Dressing or grooming self-care deficit Disturbed sensory perception (visual) Risk for injury

Correct response: Risk for injury Explanation: Because the client is disoriented and restless, the most important nursing diagnosis is Risk for injury. Although Disturbed sensory perception (visual), Dressing or grooming self-care deficit, and Impaired verbal communication may all be appropriate, they're secondary because they don't immediately affect the client's health or safety.

The nurse is performing a neurologic assessment and requests that the patient stand with eyes open and then closed for 20 seconds to assess balance. What type of test is the nurse performing? Watch-tick test Weber test Romberg test Rinne test

Correct response: Romberg test Explanation: The Romberg test is a neurologic assessment of the patient's balance in which the patient is instructed to stand with eyes open and then closed for 20 seconds. The Weber (including the watch-tick) and Rinne tests assess hearing. Chapter 60: p. 1981.

Neurological level of spinal cord injury refers to which of the following? The best possible level of recovery The lowest level at which sensory and motor function is normal The level of the spinal cord transection The highest level at which sensory and motor function is normal

Correct response: The lowest level at which sensory and motor function is normal Explanation: "Neurologic level" refers to the lowest level at which sensory and motor functions are normal.

The nurse has developed an evidence-based plan of care for a patient requiring rehabilitation after a total hip replacement. Ultimately, who should approve the plan of care? The nurse The patient The physician The physical therapist

Correct response: The patient Explanation: The evidence-based plan of care that nurses develop must be approved by the patient and family and is an integral part of the rehabilitation process.

When developing a care plan for a client who has recently suffered a stroke, a nurse includes the nursing diagnosis Risk for imbalanced body temperature. What is the rationale for this diagnosis? A decreased body temperature will signal the need to cover the client. An elevated temperature indicates cerebellum malfunction. The stroke may have impacted the body's thermoregulation centers. An elevated body temperature indicates infection.

Correct response: The stroke may have impacted the body's thermoregulation centers. Explanation: The body's thermoregulation centers are located in the hypothalamus. A stroke may impair their functioning. A decreased body temperature isn't necessarily an indication to cover the client. Although an elevated temperature may indicate cerebellum malfunction or infection, these factors aren't the focus of the risk described in the nursing diagnosis.

A patient is treated for a neurologic dysfunction affecting facial expressions. The affected cranial nerve originates in the: midbrain. cerebral hemisphere. pons. medulla.

Correct response: pons. Explanation: Cranial nerves V through VIII connect to the brain in the pons. Cranial nerve VII (facial nerve) affects facial expressions and muscle movements.

A nurse is assisting with the assessment of a client with suspected brain abscess. Which of the following findings would be consistent with such an abscess in the frontal lobe of the brain? Select all that apply. Nystagmus Changes in vision Hemiparesis Seizures Expressive aphasia

Hemiparesis Seizures Expressive aphasia Explanation: Signs and symptoms of a frontal lobe abscess include hemiparesis, expressive aphasia, seizures, and frontal headache. Vision changes are associated with a temporal lobe abscess. Nystagmus is a sign of a cerebellar abscess.


Kaugnay na mga set ng pag-aaral

INTB 3355 SmartBook Module 5: Political Forces

View Set

ASQ - Ch 18: Supply Chain Management (P. 490 - 512)

View Set

EMPA Mid Term 1 of 2 (Combined MEM6210-13 Sets)

View Set

MTA 98-349 Windows Operating System Fundamentals, MTA Operating System Fundamentals, MTA Operating System (1-10), MTA Operating System (81-90), MTA Operating Systems (91-100), MTA Operating System (111-120), MTA Operating System (131-140), MTA Operat...

View Set